You are on page 1of 208

IBRARY HOMC

H ANOI O PEN MATHEMATICS


COMPETITION 2019

(S CIENTIFIC S EMINAR )

H ANOI , 02-06/04/2019
HOMC 2019 - Scientific seminar, Hanoi, April 3,
2019

PREFACE
Founded in 2004 by the Hanoi Mathematical Society, Hanoi Open
Mathematics Competition (HOMC) is a mathematics competition held every
year for school students, and it has two sections: contestants with 14 and 16 of
age (equivalently, those in 8 and 10 grade by the educa- tion classification in
Viet Nam). As the original regular of HOMC, all questions, problems, and
contestants presentation should be presented in English. Through those fourteen
events, HOMC were attracted par- ticipation of thousands of school students as
well as attention of a great number of high and secondary schools in Viet Nam.
From 2013 to 2016, hosting by the Department of Education and Training of Ha
Noi City, the HOMCs were posited in three cities: Ha Noi, Dac Lac, and Cao
Lanh with the participation of thousands of students coming from 50 high and
secondary schools in Viet Nam. Taking some interaction between students as a
part of competition, the 2017 HOMC took place of Hanoi city and time of two
days with 900 contestants from 30 schools in Viet Nam.
Supporting by the administrative Committee of Hanoi city, HOMC
2018 has been organized with two groups: Group A is the International HOMC
which had eight international teams: China-Zheng Huan, Ghana, Indonesia,
Malaysia, Myanma, Philippines, Poland, Thailand together with two teams of the
Hanoi capital; Group B is the Home HOMC which had more than forty teams
from provinces and schools in Viet Nam.
This proceedings is published and hosted by the Department of Edu- cation
and Training of Ha Noi City, which can be seen as a registered part among the
activities HOMC 2019 by opening and sharing with the pur- poses of HOMC.
Hopefully, every participant of each one in the orga- nized HOMC’s will find
their contribitions, and new comers in HOMC 2019 will take part their new
cooperation with pleasure. We would like to express our special thanks to Prof.
Nguyen Minh Tuan, Prof. Nguyen Huu Dien who gave their valuable
suggestions and some proofreading.

1
Contents

Preface...................................................................................................................1

Tran Nam Dung, Gabriel Dospinescu, Mixing variables...................................4

Nguyen Thanh Hong, On the counter-example about parallelogram..............18

Nguyen Van Loi, The separation of elements into two parts of equal weight23

Nguyen Van Mau, On a class of functional equations in plane geometry.......36

Nguyen Van Ngoc, On the Rearrangement Inequality....................................41

Dam Van Nhi, Some new combinatorial identities..........................................57

Nguyen Van Nho, Menelaus’s theorem and its applications in some Grade 8
problems..............................................................................................................67

Vu Tien Viet, Proofs and generalizations of one inequality at the 36th IMO 78

Nguyen Dang Phat, Some International Olympiad problems in geometry 88

Tran Ngoc Nam, Selected problems from Mathematics and Youth Magazine
2018.....................................................................................................................99

Nguyen Ngoc Han, Selected via mail competition questions from Children’s Fun
Maths Journal...................................................................................................117

Nguyen Hung Son, Invariant in Tiling problems...........................................124


Le Quy Thuong, Nguyen Minh Tuan, On the Shapiro’s cyclic inequality and
some relative issues...........................................................................................132
MIXING VARIABLES
Tran Nam Dung
Hochiminh City University of Natural Science and
Gabriel Dospinescu
Romania
One of the main properties of most inequalities, especially polynomial ones is the
fact that equality holds when all or some variables are equal. Mixing variables
method is based on this property and its purpose is to decrease the number of
variables, reducing the problem to an easier form, which can be proved using
factorization, AM-GM, one-variable calculus or by mathemat- ical induction.

1 Some examples
To prove the inequality

we can try to show that


f ( x 1 , x 2, . . . , x n ) ≥ 0 (1)

f (x1, x2, . . . , xn) ≥ f ((x1 + x2)/2, (x1 + x2)/2, . . . , xn) (2)


or that
√ √
f (x 1, x2, . . . , x n ) ≥ f ( x 1 x 2 , x 1 x 2 , . . . , x n ) (3)
And then,
proving if we succeded in proving this, we reduce proving (1) to
the inequality
2, x3 . . . , x n ) = g ( x 1 , x3 , . . . , x n ) ≥ 0
(x1, xvariables.
which has onef less Of ,course, inequalities (2) and (3) can (4)
be totally wrong or can be true only with some additional condition. In
(2) and (3) we change only two variables, so usually it is not hard to verify them.
The following example will show that this technique can be used both for easy
inequalities and hard ones.
Example 1 (Kvant). Let a, b, c > 0. Show that we have following inequal-
ity:

2 ( a 2 + b2 + c2 ) + 3 3 a 2 b 2 c2 ≥ ( a + b + c ) 2
Solution. Let’s consider the function


f ( a, b, c) = 2( a + b + c ) + 3
2 2 2
3 a2b2c2 − (a + b + c)2.
The expression of f itself√suggests us to mix two variables into their ge-
ometric mean (to keep 3 3 a2b2c2 unchanged). We have
√ √ √
f (a, b, c) − f (a, bc, bc) = 2(b + c − 2bc) + (a + 2 bc)2 − (a + b + c)2
2 2

√ √ 2 √ √
= ( b − c) [( b + c)2 − 2a].
So, if a = min{a, b, c} (and we can obviously suppose that) then we
have √ √
f (a, b, c) ≥ f (a, bc, bc).
Thus, to prove the initial inequality, it is sufficient to show that f (a, b, b) ≥
0 for all a and b. But this inequality is equivalent to

2( a + 2b ) + 3
2 2
3 a2 b4 ≥ ( a + 2b)2 ,
or

a2 + 3 3 a2b4 ≥ 4ab,
√3 √3 a2b4.
2
which is true by AM-GM for a , 3 a 2 b4 , 3 a 2 b4 ,
√3
3
The following example is much more difficult:

Example 2 (Vietnamese IMO Team selection Test, 1996). Let a, b, c be ar-


bitrary real numbers. Prove that

47
F(a, b, c) = (a + b)4 + (b + c)4 + (c + a)4 − (a4 + b4 + c4) ≥ 0
Solution. We will show that with some additional condition
F(a, b, c) ≥ F(a, (b + c)/2, (b + c)/2).
Indeed, long but easy computations show that we have

F(a, b, c) − F(a, (b + c)/2, (b + c)/2)


4
= ( a + b ) 4 + ( b + c )4 + (c + a) 4 − (a + b + c )
4 4 4

7
4
− 2(a + (b + c)/2)4 − (b + c)4 + (a4 + 2((b + c)/2)4
7
4
= (a + b)4 + (c + a)4 − 2(a + (b + c)/2)4 + c((b 7+ c)4/8 − b4 − c4)
3
= a(4b3 + 4c3 − (b + c)3) + 3a2(2b2 + c2 − (b + c)2 ) + ( b + c − (b + c) /8)
4 4 4

7 2 + 10bc]
= 3a(b + c)(b − c)2 − 3a2(b − c)2 + (3/56)(b − c)2[7b2 + 7c
3
= 3a(a + b + c)(b − c)2 + (b c)2[7b2 + 7c2 + 10bc].
56

The last term is nonnegative. If a, b, c have the same sign, then the
from c has
a, b, is
inequality the If
trivial. same
a, b,sign
c have + b + c.signs
as adifferent WLOG,thensuppose that From
at least one it is a.the
initial
last
equality we have

F(a, b, c) ≥ F(a, (b + c)/2, (b + c)/2).


Thus, it is now sufficient to show4 that F(a, b, b) ≥ 0 for all a, b, or 2(a +
b)4 + (2b)4 − (a4 + 2b4) ≥ 0.
If b = 0 then inequality is trivial. If b =ƒ 0, then dividing the two sides of the
inequality by b4 and putting x = a/b, we reduce it to the equivalent form:

7
47 4
2(x + 1) + 16 − (x + 2) ≥ 0
4

The last one can be proved by one-variable calculus. Consider

4
f (x) = 2(x + 1)4 + 16 − (x4 + 2).
7
We have
16
f J ( x ) = 8( x + 1)3 − x3. f J ( x) = 0
7
3 2
⇔ x+1=. 7x

Thus ⇔ x = −2.9294.

fmin = f (−2.9294) = 2(−1.9294)4 + 16 − (4/7)(−2.9294)4 − 8/7 = 0.4924.Q


For free of conditions inequalities in general we can mix variables into
whatever we want: arithmetic mean, geometric mean, harmonic mean, quadratic
mean ... and in some cases, we can even define special means. A beautiful
example is the following famous (due to its difficulty) prob- lem:

Example 3 (Iran Mathematical Olympiad 1996). Let a, b, c be positive re- als.


Prove that:

1 1 1 9
( ab + bc + ca). Σ≥
( a + b )2 (b + c) 2 ( c + a )2 4
Analysis.
We can mix a, b into (a + b)/2, but then both terms in L.S.H will be
one term and
changed unchanged. Forwill
the result example, webulky.
be very will keep ab + bcwe
Naturally, + cawant
unchanged.
to keep
For this goal, we mix b and c into t such that

ab + bc + ca = at + t2 + ta.
If we solve this equation (in t) we get

t = − a + .( a + c)( a + b).
Let

f (a, b, c) = 1/(a + b)2 + 1/(b + c)2 + 1/(c + a)2.


Our goal is to show that

f (a, b, c) ≥ f (a, t, t).

sumption, forweexample
this inequality, holds aonly=withmin
see that it for {a, as-b,
some c}. Investigating

Solution. WLOG, we can assume that a = min{a, b, c}. Let

t = − a + .( a + c)( a + b),

g(a, b, c) = ab + bc + ca
and
f (a, b, c) = 1/(a + b)2 + 1/(b + c)2 + 1/(c + a)2.
Then
g(a, b, c) = g(a, t, t).
We will show that
g(a, b, c) ≥ (a, t, t). (5)
Note that

a + t = .( a + c)( a + b),

.( a + c)( a + b) ≤ t ≤ (b + c)/2
Also, (5) is equivalent to
( 1/
can also be written( a + b ) − 1/ ( a + c )) 2
≥ 1/4t 2
− 1/ ( b + c ) 2
, which

(b − c)2/(a + b)2(a + c)2 ≥ (b + c − 2t)(b + c + 2t)/4t2(b + c)2.


Now,
.
b + c − 2t = (a + b) + (a + c) − 2 (a + c)(a + b)
√ √
=( a+b− a + c) 2
√ √
= ( b − c) / ( a + b + a + c) 2
2
and it sufficient to show that
√ √
( a + b + a + c)24t2(b + c)2 ≥ (a + b)2(a + c)2(b + c + 2t) But
this is trivial because (a + c)(a + b) 2t b + c.
√ is to prove that
Thus, all we have to do now

g(a, t, t) f (a, t, t) ≥≤9/4.


This can be done by easy computation:

g(a, t, t) f (a, t, t) ≥ 9/4


⇔ 4(2at + t2)(2/(a + t)2 + 1/4t2) ≥ 9
⇔ (2at + t2)(8t2 + (a + t)2) ≥ 9t2(a + t)2
⇔ (2at + t2 )(9t2 + a2 + 2at) ≥ 9t2 (a2 + 2at + t2)
2
⇔ at(a − t) ≥ 0.Q
Mixing variables can be used not only for free of conditions inequalities,
but also for inequalities with additional conditions. In these cases, when we mix
variables, we have to be sure that the conditions still hold. So, we have less
variants to choose.

Example 4 (Vietnamese Mathematical Olympiad 2002). Prove that for ar-


bitrary real numbers x, y, z, satisfying

x2 + y2 + z2 = 9,
the inequality

holds. 2(x + y + z) ≤ xyz + 10

Solution.
Let
f (x, y, z) = 2(x + y + z) − xyz.
We have to show that

f (x, y, z) ≤ 10 when x2 + y2 + z2 = 9.
To
intoleave
their the condition
quadratic x2 +Consider
mean. y2 + z2 = 9 unchanged, we will mix two variables

f ( x, .(y2 + z2 )/2, .(y2 + z2 )/2) − f ( x, y, z)

= 2( x + 2.(y2 + z2 )/2) − x (y2 + z2 )/2 − 2( x + y + z) + xyz

= 2(.(y2 + z2 ) − y − z) − x (y − z)2 /2

= (y − z)2 [2/(.2(y2 + z2 ) + y + z) − x/2].


If x, y, z > 0, we consider two cases. The first one:
1 ≤ x ≤ y ≤ z.
Then


2( x + y + z) − xyz ≤ 2.3( x2 + y2 + z2 ) − 1 = 6 3 − 1 < 10.
The second one: 0 < x ≤ 1. Then

2( x + yJ + z) − xyz < 2( x + y + z) ≤ 2x2 + 2.2(y2 + z2 ) = 2x + 2.2(9 − x2 ) = g( x ). We


have g ( x ) = 2 2 2x/(9 x ) > 0, so it follows that g( x )
g(1) = 10.
thatIfxamong numbers x,√
< 0. Then y, z there is a negative number, WLOG, suppose
− −
f (x, .(y2 +to
reduce our problem z2)proving z2)/2) − f (x, y, z) ≥ 0 and we
/2, .(y2 + that

f ( x, .(y2 + z2 )/2, .(y2 + z2 )/2) ≤ 10


or
.
2x + 2 2(9 − x2) − x(9 − x2)/2 ≤ 10,
or .

h(x) = x3 − 5x + 4 2(9 − x2) ≤ 20.


We have:
hJ ( x ) = 3x2 − 5 − 4x .2/(9 − x2 ).

Solving the equation hJ ( x ) = 0 (where x < 0), we get x = −1. It is


maximal point of h, thus

h(x) ≤ h(−1) = 20.

Example 5 (Gabriel Dospinescu). Prove that if a, b, c > 0 verify a2 + b2 +


c2 = 3, then

Solution. a3(b + c) + b3(c + a) + c3(a + b) ≤ 6.


Again, thewepurpose it todifference
preserve the relation given in the problem.
That is why study the
b +c
f (a, b, c) − f (a, t, t), where t = . 2 2.
It is easy to compute the difference. It is
2
3 −
−a (2t − b − c) + a(b3 + c3 3 ) + t2 (2bc − 2t2).
2t
Let us take a = min{ a, b, c}. Then

b3 + c3 − 2t3 = (b + c)(b2 − bc + c2) − 2t3 ≤ 2t(b2 − bc + c2) − 2t3 = t(b − c)2.


So, the difference is at most
3 ≤ 0,
−a (2t − b − c) + at(b − − t (b −
2
c )2 c) 2
since 2t − b − c ≥ 0 and a ≤ t. Thus, it remains to prove that if

a2 + 2t2 = 3, then f (a, t, t) ≤ 6.


This can be written

a3t + at3 + t4 ≤ 3
2 4
⇔ at(3 − t ) ≤ 3 − t
⇔ (3 − 2t2)t2(3 − t2)2 ≤ (3 − t4)2
and this one is true since it is equivalent to

3(t2 − 1)2(t4 − 3t2 + 3) ≥ 0.


Thus, it isn’t always trivial to show the inequality for two numbers. We
will see that there is an easier solution for this problem, thought this is the most
natural solution.
We finish with an inequality proposed for the final round of the ro- manian
mathematical olympiad in 2003:

Example 6 (Mircea Lascu and Marian Tetiva). Prove that if a, b, c > 0 have
product 1, then the following inequality holds:

a2 + b2 + c2 + 3 ≥ a + b + c + ab + bc + ca.
Solution. Now it’s obvious that we should consider the function:

f (a, b, c) = a2 + b2 + c2 + 3 − a − b − c − ab − bc − ac.
It is also easy to deduce that

√ √ √ √ √ √
f (a, b, c) − f (a, bc, bc) = ( b + c) (( b + c)2 − 1 − a).
2

So, we would like to have the last term positive. Naturally, we choose

a = mon {a, b, c}. By AM-GM we have that


√ √ √ 4
( b + c)2 ≥ 4
bc = √ ≥ 4 ≥ 1 + a, since a ≤ 1.
a
Thus, it is enough to prove that
√ √
f (a, bc, bc) ≥ 0
and this is in fact 2 √
1
a2 + . Σ

√ −1 +2≥ a+2 a
and it is obviously true since a


a + 2 a ≤ 2a + 1 ≤ a2 + 2.
In considered examples, we mix variables into their means. We do so,
because equality holds when these two variables are equal. But for some
inequalities, this property does not hold, and we have to choose other expression
to mix into, rather means. Usually, in these cases we use boundary values (look
at the ends priciple).
Example 7 (Gabriel Dospinescu, Gheorghe Vranceanu Contest, 2003). Let n
be a natural number, n ≥ 2. Find the greatest value of the expres-
sion + (bc)n + (ca)n
n
1(ab)ab 1 bc 1 ca
− −
when a, b, c are
Thearbitrary nonnegative
investigation reals case (n =a2+) shows
− satisfying
of the easiest b + c =that
1. Analysis.
the
maximum value is not achieved in the point (1/3, 1/3, 1/3), but in the point (1/2,
1/2, 0). This fact suggests that we should mix a and b into a + b and 0. But in
additional condition that c = max a, b, c . When one variable is equal to 0,
the function becomes a simple one-variable
function.
Solution. Let’s assume that a ≤ b ≤ c. Put {
(ab)n (bc)n (ca)n }
f
+ + .
We will show that (a, b, c) = 1 − ab 1 − bc 1 − ca

This is equivalent to f (a, b, c) ≤ f (0, a + b, c).


(ab) n
(bc)n (ca)n (a + b )nc n
(6)
+ + 1 − ac − bc
1 − ab 1 − bc 1 − ca

We write consequently

(a + b )nc n (an + nabn−1 + bn)cn


1≥− ac − bc 1 − ac − bc

≥ ancn + anbn + bncn


1 − ac − bc 1 − ac − bc 1 − ac − bc
anb n anc n bncn
≥ + + .
1 − ab 1 − ac 1 − bc
and we get (6).
All we have to do now is to find the maximum value of
(bc)
n where b + c = 1.
f (0, b, c) = 1 − bc
This is an easy exercise, since the function

h ( x ) = x n / ( 1 − x)

is an increasing function on (0, 1) and

bc ≤ (b + c)2/4 = 1/4,
so max
f = h(1/4) = 1/(3.4n−1).
1
when a = b = and c = 0. This is the
answer to the problem, since the value is attained for example
2
As it can be easily deduced from the above, mixing variables is most
useful for 3-variable inequalities. The reason is simple: if we can mix variables
for a 3-variable inequality, then we get 2-variable one, which, in most of cases
can easily reduce to 1-variable (for example, a homogenus 2-variable is
equivalent to 1-variable inequality). When the expression which needs to be
evaluated has more than 3 variables, using this method meets more difficulty,
because every mixing does the expression more complicated. Especially in the
case of a general inequality (with n vari- ables), the difficulties are more than
obvious: mixing variables change the general form and we cannot use induction,
which is the most ap- propriate approach. Mixing variables will be more effective
if we use it together with other arguments, for example, the properties of continu-
ous functions on a compact.
Example 8 (Gabriel Dospinescu). Prove that for positive reals x1, x2, . . . , xn, whose
product is 1 we have
1 1 1
x1 + x2 + · · · + xn
+ +···+ ≤ .

1 + x1 1 + x2 1 + xn 4
Analysis. Let
x+1 1
f ( x) = − .
4 x+1
We have to show that if
x1.x2 . . . .xn = 1 then f (x1) + f (x2) + · · · + f (xn) ≥ 0.
√ √
The condition x1.x2 . . . .xn = 1 forces us to mix x1, x2 into x1, x2, x1, x2. But
after that? We can mix some pairs of variables, but if the number of
k
ables brings us to result iff n = 2
is odd? More detailed consideration . What we can do
shows thatin other
mixingcases?
vari-Wecan
variables
work
backward!
If
min{show
we will ) + f ( x2 ) + · · · + f ( xn )} = f ( x1∗ ) + f ( x2∗ ) + · · · + f ( xn∗ ),
f ( x1that
x1∗ = x2∗ = · · · = xn∗ = 1,

and we get that min = 0. It is trivial if we can prove that if



x1∗ ƒ= x2∗ then f ( x1∗ ) + f ( x2∗ ) > 2 f ( x1∗ x2∗ ).
But why does this minimum exist? Sometimes, we ignore this fact,
considering it trivial. But it is one of main question in this problem.
Some properties of continuous functions will help us.
Solution. Let
x+1 1

We will show that if − .


f (x) = 4 x+1

x1.x2 . . . xn = 1 then f (x1) + f (x2) + · · · + f (xn) ≥ 0.


First, we will show that there exists a minimal value of

S(x1, . . . , xn) = f (x1) + f (x2) + · · · + f (xn)


on the surface

{(x1, . . . , xn) ∈ (R+)n|x1.x2 . . . xn = 1}.


Note that
f (1)if+ · · · + f (1) = 0 and 1/(1 + x1) + 1/(1 + x2) + · · · + 1/(1 + xn) < n.
So,

xi > 3n for some i then S(x1, . . . , xn) > 0.


Similarly, if

and xi < 1/(3n)n then x1 . . . xn/xi > (3n)n,

and x1 + · · · + xn − xi > (n − 1)(x1 . . . xn/xi)1/n−1 > 3n

S(x1, . . . , xn) > 0.


Thus, we can find the minimal value in the compact:

C = {(x1, . . . , xn)|1/(3n)n ≤ xi ≤ 3n, x1 . . . xn = 1}.

S(x1, . . . , xn) is continuous on C, so the first thing is done.


Secondly, let

min{ f ( x1 ) + f ( x2 ) + · · · + f ( xn )} = f ( x1∗ ) + f ( x2∗ ) + · · · + f ( xn∗ ).


We will show that
x1∗ = x2∗ = · · · = xn∗ = 1.
Indeed, if it is not so, then we can find x1∗ ƒ= x2∗ . We will show that if


x ƒ= y then f (x) + f (y) > 2 f ( xy), (7)
which leads to a contradiction. But
√ √
(7) ⇔ (x + y − 2 xy)/4 > 2/(1 + xy) − 1/(1 + x) − 1/(1 + y)
√ √ √ √
⇔( x− y)2 > [2(1 + x)(1 + y) − (1 + xy)(2 + x + y)]/(1 + xy)(1 + x)(1
√ √ √ √ √ √
⇔ ( x − y)2/4 > ( x − y)2(1 − xy)/(1 + xy)(1 + x)(1 + y).
which is true. Finally, we conclude that

f (x1) + f (x2) + · · · + f (xn) ≥ f (1) + · · · + f (1) = 0.Q


2 Studied Problems
Problem 1 (After Kvant problem). Prove that for arbitrary real numbers
x1, x2, . . . , xn we have
n 2
(n − 1)( x21 + x2 2+ · · · + x2 n) + n . x2 x1 2 2. . . x2 n≥ ( x1 + x2 + · · · + xn ) .
Problem 2 (PTNK Team selection test 2002). Find all positive integer k such
that inequality

2(ak + bk + ck ) + 3 abc ≥ 9

holds for all positive reals a, b, c satisfying a + b + c = 3.


reals x3 Matematica
satisfyingGazeta
(Vasile Cartoaje, + y3 + A).z3Let =x, y,3.z beShow
positive that Problem 3
x4y4 + y4z4 + z4x4 ≤ 3.
0Problem 1 then: for 2000 Moscow Olympiad). Prove that if x, y, z >
4 (Proposed
have product

x2 + y2 + z2 + x + y + z ≥ 2(xy + yz + zx).
Problem
are 5 (1993
nonnegative IMO
reals Shortlist,
which add up proposed by Vietnam). Prove that if a, b, c, d
to 1, then

1 + 176abcd
abc + bdc + cda + dab ≤ .
27
a2 + b2 + c2 + 2 + that
Prove ≥ all
abc for a +
a, bb, +
c>c 0+ we + bc the
ab have + ca. Problem
following 6.
inequality:
Problem 7. For positive real numbers a, b, c prove that

(a2 + b2)(b2 + c2)(c2 + a2)(ab + bc + ca)2 ≥ 8a2b2c2(a2 + b2 + c2)2


Problem 8. For positive real numbers a, b, c prove that
a3 3 + b33 + 3c≥
3
3
(a + b) (b + c) (c + a) 8
Vietnamese
Problem 9.Team selectionreal
For arbitrary testnumbers
2005 a, b, c, show that
16
( a + b )6 + (b + c) 6 + ( c + a )6 ≥ ( a 6 + b6 + c 6 ) .
63
ON THE COUNTER-EXAMPLE ABOUT PARALLELOGRAM
Nguyen Thanh Hong, Hanoi University of Education
Combined the properties of a parallelogram, we propose some problems, which seems
useful on proving quadrilaterals are parallelograms. . .

1 Introduction
1.1 Definition
A parallelogram is a quadrilateral with both pairs of opposite sides parellel.

1.2 Several properties of parallelograms

Theorem 1. If a quadrilateral is a parallelogram, then its opposite sides are


congruent.

Theorem 2. If a quadrilateral is a parallelogram, then its opposite angles are


congruent.
Theorem 3. If a quadrilateral is a parallelogram, then its consecutive angles
are supplementary.

Theorem 4. If a quadrilateral is a parallelogram, then its diagonals bisect each


other.

1.3 Proving quadrilaterals are parallelograms


Corresponding to the 4 above theorems, we have the following well- known
ways to prove that a quadrilateral is a parallelogram.

Theorem 5. If both pairs of opposite sides of a quadrilateral are congruent, then


the quadrilateral is a parallelogram

Theorem 6. If both pairs of opposite andgles of a quadrilateral are congruent,


then the quadrilateral is a parallelogram

Theorem 7. If an angle of a quadrilateral is supplementary to both of its con-


secutive angles, then the quadrilateral is a parallelogram.

Theorem 8. If the diagonals of a quadrilateral bisect each other, then the


quadri- lateral is a parallelogram.

2 A counter example about parallelogram?


Combined the properties of a parallelogram, we propose the follow- ing
problem, which seems useful on proving quadrilaterals are parallel- ograms.

Problem 1. Given a quadrilateral ABCD such that AB = CD and ∠BAD =


∠BCD. Does this quadrilateral must be a parallelogram?
We present below our proof.
to
LetAD,
H, K areSince
BC. ∠BAD the
respectively = ∠foot
BCD,of we see that either
perpendiscular K, B,
from HD belong to Proof.
segments AD,
BC, or A, C belong to segments DH, BK respectively.
The two triangles ABH and CDK are congruent, then DK = HB and

CK = AH.
Consequently, the two triangles BHD and DKB are congruent, hence
BK = DH.
Thus, we ABCD
quadrilateral must=beAD,
obtain BC combining with the hypothesis AB = CD,
a parallelogram. Q
Unfortunately, the following construction will show a counter-example of the
above problem.

E acrossand
trapzoid) the side
DE BD. = Then
DC. weIt have AB =
shows (diagonals
thatDE AB = CD.of an isoscelec
Given an
isoscelec trapzoid ADBE. Let C be the symmetric point of
On the
∠BED =∠ other
DAB.hand, from
Then = ∠BCD.we have ∠BCD = ∠BED. Moreover,
the symmetry
∠BAD
Finally, the quadrilateral ABCD satisfies the condition of the above problem,
but it is not a parallelogram.
3 Selected problems
Problem 2 (Iranian Geometry Olympiad 2015). In rectangle ABCD, the points
M, N, P, Q lie on AB, BC, CD, DA respectively such that the area of triangles
AQM, BMN, CNP, DPQ are equal. Prove that quadrilateral MNPQ is
parallelogram.
Problem 3 (Estonian 2005). Let ABCD be a parallelogram, M the mid- point
of AB and N the intersection of CD and the angle bisector of
∠ ABC. Prove that CM ⊥ BN iff AN is the angle bisector of ∠ DAB.
Problem 4 (JBMO 2001). Let ABCD be a parallelogram centered at O. Let
M and N be the midpoints of BO and CD. Prove that if the triangles ABC and
AMN are similar, then ABCD is a square
Problem 5 (JBMO 2001). Consider the triangle ABC with ∠ A = 90◦ and

∠B ƒ= ∠C. A circle (O, R) pass through B and C and intersect the sides
AB and AC in D and E, respectively. Let S be the foot of the perpen-
dicular from A to BC and let K be the intersection point of AS with the segment
DE. If M the midpoint of BC, prove that AKOM is a parallelo- gram.

Problem 6 (Iranian Geometry Olympiad 2016). Suppose that ABCD is a


convex quadrilateral with no parallel sides. Make a parallelogram on each two
consecutive sides. Shows that among these 4 new points, there is only one point
inside the quadrilateral ABCD.
Problem 7. Consider a parallelogram ABCD such that ∠ ABC = 80◦ and
∠ ACD =the
intersects 20◦line
. A line
AC passing through
in the point A meets
R. B the line
line passing AB at an
through

C angle 20 and
meets the line AC at an angle of 30◦ and intersects the line AB in the point T.
Find the measure of the angle determined by the lines TR and DC.

Problem 8. Let ABCD be a parallelogram. The circle c with diameter


AB passes through the midpoint of the side CD and through the point
D. Determine the measure of the angle ∠ABC.
Problem 9. Consider a convex quadrilateral ABCD and let M, N, P, Q
( MN + PQ
2midpoints of )the
= AB
sides+ AB,
BC +BC, + DA,
CDCD, DAthen ABCD is Prove
respectively. a parallelogram. be the
that if Problem
10. Given a parallelogram ABCD. N is the intersection point
of CD and the angle bisector of ∠ ABC. Prove the following assertions are
equivalent
1) BN AN

2) AB = 2BC
3) AN is angle bisector of ∠ DAB
with
Problem∠ DAB = 60◦. Let
11 (Estonian L be points 2002).
K, competitions
math on its Let
sidesABCD
AD and
be aDC and Ma
rhombus
point on the diagonal AC such that KDLM is a parallelogram. Prove
that triangle BKL is quilateral.
Problem 12 (Estonian 2004-final round). Consider a convex quadrilat- eral
ABCD. Choose a point O in the plane and let K, L, M, N be the cir- cumcenter
of triangles AOB, BOC, COD, and DOA, respectively. Prove that there exists
exactly one point O in the plane such that KLMN is a parallelogram.

References
[1]D. Branzei, L. Serdean, V. Serdean Junior Bankal Mathematical Olympiads,
Plus Publishing House, 2003, 146 pages.
[2]R. Geretschlage, J. Kalinowski, J. Svrcek, A Central European Olympiad,
the Mathematical Duel, vol.7, World Scientific Publishing, 2018, 281 pages
[3]website: Art of Problems Solving.
THE SEPARATION OF ELEMENTS INTO TWO PARTS OF
EQUAL WEIGHT
Nguyen Van Loi, Budapest - Hanoi
When dealing with sets where elements are attached distinct weights, the question
naturally arises: what options are there to divide the weights? This question also
has great relevance to everyday life. In this article, I will con- tinue the research
we began with Peter Erben [4] and then continued with
N. H. Dang and D. Th. Khang [3], placing the emphasis on a clearer presen-
tation, simpler forms of proof, as well as potential utilization.

1 The structure of elements with weight and its effect on


the distribution of weight
Basic knowledge can be found in [2] and [5].
Let A be a set with elements e1, e2, , ek, k is the number of elements. Each
element is assigned ei. Accordingly, the weight assigned to ei is denoted
s(ei), leaving no room for misunderstanding. To make calculations eas-
ier, we list the elements (weights) in a monotonous order.
s1, s2, ,all
mentsarticle st from A, the
weights are sum s1 +integers
positive s2 + + stand
is the
all sum of s(finite.
sets are s1) + sFor
(s2)ele-
+ In this
+ s(st). th
the m = sis +
s2 weights
+ + nst, if for any m s n isthere the
is e1, eweight
2 , , et Aofso that 1
where ei. Given n
≤ can be expressed
positive integer and set A. Seti A up to
If S is the sum of the elements in set A and set A can be expressed until S,
we can say that S is fully expressible. Note, if a set is expressible until a
point (> 0), then it obviously contains the weight unit. Thus, a set which is
fully expressible can be regarded as a measuring system in which we can
measure any weight between 1 and S.
Theorem 1. Given are set A and N weight. If the weight of elements in A are
not more than N and set A is expressible up to N, then set A is fully expressible.
Proof.
taken asLet A = to{a1their
identical ; a2;weights),
; ak bea aisset of theasweights
denoted a = a1 +(here
a2 + the
+ elements are
ak. By induction according
} to N, we shall prove the Theorem.
If N =
trivial. 1, then all elements in set A weight one unit, thus the statement is

ment is true
with the weight + 1.
to N of N If+ A1,can be by
then expressed upof
the rules to induction
N and there
theare no elements
Assuming the
statement is true up to N, we need to show that the state-
with
true. Thus,the weight that set(NA has a +few elements
we can assume 1). Let statement is

N + 1 = a1 = a2 = .... = ai > ai+1 ≥ ai+2 ≥ .... ≥ ak


and b = athe
one with i+1 + ai+2 + .... + a k . It is obvious that if we are looking to ex- contain
weight (N + 1). It follows that B = a ; ai+2; ; ak press a weight s
with elements from set A, then these elementsi+1 cannot is a set which is
expressible up to N and the total weight of its elements
is no greater than N. Let us take an arbitrary 0 < s a. ≤
If s { b, then s
exist r, q N so that s b = q (N + 1) r where 0 }≤ r < N. can be
expressed with weights contained in B. If b < s a, then there
∈ − −
If q i + 1 s (i +≤1)(N + 1) + b r = a + ( N + 1) r > a, which
≥ with
expressed ⇒q pieces
≥ of (N + 1) weights.
≤ −At ≤ the same time, r can be would be
contradictory.−Thus, 0 q i, which means b−r can be expressed with elements
from B. These two sets are disjoint, thus s is
expressible with elements from A. Thus, A is fully expressible.

If A and B are two fully expressible sets, then the element with the largest
weight can either be found in A or B, thus the theorem is proved.

Corollary 1. If A and B are two fully expressible subsets of set S, then C =


A ∪ B is also fully expressible.

least up which
be a set to 1. contains
Let us denote P(A
the unit ) as a fully
weight, thus expressible subset atof set Let A
A is expressible
A. Then P(A) is fully expressible and is the largest among the fully
expressible subsets. From this we can conclude:
Corollary
largest fully2.expressible
Let A be subset
a set that
of A contains the of
and the sum unit element.
elements of PLet (Ah.) be
(A)Pis If the
the
weight of any element in A is no larger than h, then either A = P(A) or a ≥ h +
2, provided there is an a ∈ A\ P( A).

Proof. If there is an s weight where s ∈ A and s = h + 1, then P(A) ∪ {s


is }
thealso fullysuch
largest expressible,
subset. which would contradict the statement that H is

In the following, let us denote A(M, k) a set where the total weight is M
and has k elements.
M M
Proposition 1. Let A(M, k); k ≥ 4 + 2 and ai ≤ 2. Then the average
of the weights is smaller than 4 3 elements with
. If A contains a maximum of
weight less than 4, then
a)a 1 = a2 = a3 = 1, a4 = a5 = . = ak−1 = 4 and ak = 5; or
b)a 1 = a2 = 1, a3 = 2, a4 = a5 = . = ak = 4.

In these cases A(M, k) is fully expressible.


Note: Since M > k, it is easy to prove that M ≥ 3.
M M
Proof. M < 4( + 2) ≤ 4k, then < 4, which means the average of
4 k
the weights of its elements is smaller than 4.
On
withthe otherless
weight hand,
thanlet4.usInassume thatAthere
this case, has are at most
at least k −33elements
elementsinwith
A
M
weight over 3. Thus, M ≥ a1 + a2 + a3 + 4(k − 3) ≥ 3 + 4( + 2 − 3) =

M 1. Since a1 4
− ≤ a2 ≤ a3 ≤ ≤ a k (the weights are in ascending order),
it follows that
a2 = 1,aa1 3==
Either
It follows
2 = a3 = 1, a4 = a5 = . = ak−1 = 4 and ak = 5 Or a1 =
a2,
from 4 = a5 = . =
aTheorem ak = A(M,k)
1 that 4. is fully expressible.
N
Theorem 2. Given A(2N, k), s(ai) = ai ≤ N and k ≥ + 2. Let us denote the
largest fully expressible subset of A as P(A). If P (A) is 2expressible up to at
least 3, then:
N
1)Let S be the total weight of P (A), then S ≥ ;
2
2)A is either fully expressible or A has exactly two elements with weight no
N
less than S + 2, thus no less than (
2 + their
into two subsets of equal weight (meaning
2), and set A can be separated
weight is N).

Proof. 1) LetASisbefully
P(A), then the weight of P (and
expressible A) (obviously S isseparated
so A can be a positiveinto
integer). =
If Aof
two parts
equal weight.
Let us assume that A is not fully expressible.
4
3, elements with weight
P(A) contains less thanwith
every element P( A) no
4. Asweight canlarger
be expressed at least
than 4. Thus, a) up to
First
we need to see that N > 4. Due to Proposition 1, set A has at least
which
statementmeans
thatthey is(A
are kin) P
A( M, ). fully
not So P(expressible.
A) = A, which is a contradiction
Thus, N > 4. if 2Nof the 8,
then the≤ weight of the elements in A can be no larger than 4,
b) If N > 4, let B = A\ P( A) = b1; b2; ; b p . We can identify
= s(bj).2,Then
bCorollary
j
S +=2 Sfor+ every
bj ≥ 2N + j· ≤
b1 + b02 < ·{ ·p.+Ifbpthe
and p > 0.of According
number the elementsto
the elements of B with their weights without } misunderstanding, that is in set
P( A) is q, then k = q + p and S ≥ q. Then:
2N = S + b1 + b2 + .... + bp ≥ S + p(2S + 2) = S + (k − q)(S + 2) Since S

=⇒ S⇒2 −
q 2N(k≥−S 1+)S(k−
− 2k + 2)>=0.−Looking
S)(+S2N S + (k − )S quadratic
at 1the + 2k polynomial:
f (x) = x2 − (t + 1)x + 2t − 4
f (2) = 22 − (t + 1) · 2 + 2t − 4 = −2 < 0
f (t − 1) = (t − 1)2 − (t + 1)(t − 1) + 2t − 4 = −2 < 0
This means that on the [2, t − 1] interval f (x) < 0. Since S > 3 and
N N
f (S) > 0, thus S ≥ t and T = ⇒ S≥ .
2 2
2) Obviously if set A is fully expressible then it can be separated into two
subsets of equal weight. Thus we only need to look at the case when set
N fully expressible. It follows that A ƒ= P(A). In this case, due to
A is not
S≥ ,
2

N N N
2N ≥ S + p(S + 2) ≥ 2 + p( 2 + 2) = 2( p + 1) + 2p
where p is a positive integer. From this we can find that 4N − 4p ≥
4p
N · (p + 1) ⇒ 3 − > p.
N
As p is an integer, p 2.

a) If p = 1 ⇒ b1 = 2N − S ≥ 2N − (b1 − 2) ⇒ b1 N + 1 it would

contradict the fact that the weight of the elements of A is no larger than
N. Thus, p ƒ= 1. N

b) If p = 2, then b1; b2 ≥ S + 2 ≥ + 2. Furthermore


2
N N
N − b1 ≤ N − ( 2 + 2) < 2 − 2 ≤ S
N N
Since P (A) is fully expressible, because of − b ≤ S, − b1 can be
2 1
expressed with the elements of P(A). Thus there exist h1, h2, , hl so2 that

N − b1 = h1 + h2 + + hl meaning N = b1 + h1 + h2 + + hl .
This means that set A can be separated into two subsets of weight N.
N
Corollary 3. Given A(2N, k) where N ≥ 2, s(ai) = ai ≤ N and k ≥ + 2,
then set A can be separated into two subsets the difference between the weights
of which is 0 or 2. 2
Proof. Let us add two elements of unit weight to set A and call the
N
new set A. The total weight of set A is 2N + 2 and has k + 2 ≥ +2+
2
N+1
2 > + 2 elements, and the weight of its elements is smaller than
2
(N + 1). Theorem 2 can be applied to prove that A can be separated into
two subsets of equal weight. Taking out the two elements we added, we
can prove the statement.
Examples
1)1) Let A = {1, 1, 1, 86, 6}, then the weight of A is 16. The number
of elements is 6 = + 2. A cannot be fully expressed (as 5 is not
2
expressible) and 1 + 1 + 6 = 1 + 1 + 6.
Counter-examples include instances when the criterion of express-
ibility up to 3 is not satisfied.

2)Let N be an odd number. K = N then A = { 2; 2; 2; s; 2 }


˛¸. The total
x of 2
N pieces
weight of set A is 2N. In this case set A is not expressible.

3)When N = 6m + 2(2N = 12m + 4), k = 4m + 2, let A = {1; 3;


s 3;˛¸....; 3 }.
4m+1 x
pieces of 3
In this is
which case set A can
divisible by 3only
andbe separated
another into
which is anot.
subset the weight of

4)When N = 6m + 4(2N = 12m + 8), k = 4m + 3, let A = {2; s 3;˛¸...; 3 }.


3;
4m+2 x
pieces of 3
In this is
which case set A can
divisible by 3only
andbe separated
another into
which is anot.
subset the weight of

5)When N = 6m(2N = 12m), k = 3m + 1, let A = { s 2;˛¸...; 2 ; 3; 6m −


2;
1}. In this case A cannot be separated into two equal x either.
3m−1 pieces
parts of 2

6)If N is odd, then A = { 2; 2; ...; 2 }. A cannot be separated into two


s ˛ ¸ x
n pie ce s of 2
equal parts.
From the above examples it can be seen that expressibility up to 3 is very
important.
Proposition 2. Given A(2N, k) where N ≥ 2, s(ai) = ai ≤ N and k ≥
N + 2. Furthermore, let a1 = 2. Then, the sum of the weights of any two
elements
2 in set A are not larger than N.

weight. This
Let way it is enough
us order to prove
the elements in ascending ak ≤ N,
that ak−1 + order which can
according be Proof.
to their
seen through:

N
a1 + a2 + .... + ak−2 ≥ 2(k − 2) ≥ 2 · 2= N
From this it follows that ak−1 + ak ≤ N.
Theorem 3. Given A(2N, k) where N is even, s(ai) = ai ≤ N and k ≥
+ 2. Furthermore, let a1 = a2 = 2. Then, set A can be separated into two subsets2
of
N equal weight.
Proof. Letis usL order
the other = l1the
; l2;elements in two subsets,
; lv , containing with theThis
odd weights. beingAC == Cwhile
firstway L
and C L, u + v = k. Since 2N is an even number, v also c1; c2; ; cu ,
{
containing elements the weights of which are even numbers, needs to be even.
Let
} v = 2t and consider the (u + t) set.
{
B = b1; b2; ; b(u + t) where the } bj elements are composed in the follow-
ing way: ∪
ci
{ b =
2 where i = 1, 2, , u, and
i
}
lj − lv−j
bu+j = 2 where j = 1, 2, , t.
Due to Proposition 1, a3 ≤ a4 3. The two first elements of set B are
1, 1 and its third element is not larger than 3. It follows that B can be expressed
up to 3.
On the other hand, due N to Proposition 2, the weight of elements in B
cannot be larger than , and the total weight of B cannot be larger than
N either. 2
N
It can be proved that u + t ≥ + 2 in the following way:
4
u+v− 2 u− 2
v ≥ 2+ +
u+t=2+u− 2+ 2 2
2

12 N
2 u −2 2 N4 u − 22
≥ 2+ · + =N
2+ +
N u− 2
From this: u + t ≥ 2 + 4
+ 2
≥ 2+ 4.
All criteria of the Theorem 2: are satisfied, thus B can be separated into two
subsets of equal weight. Restoring the original values, we will find proof of the
separability of set A.

2 Applications
This section will focus on the quantity of elements in a set, looking into how
the process of separation can be made easier by increasing the quantity of
elements.
Proposition 3 (In
(m is a positive case NGiven
integer). divided
A(by 6 yields
2N, a remainder
k) where s(ai) = aiof 2). andNk =4m
N Let 6m++3,2
then set A can be separated into two subsets of equal weight and the value of k
cannot be reduced. ≤

Proof. Since k = 4m + 3 ≥ 3m + 3 = + 2, due to Proposition
N
1, a3 3. If a3 = 3 a3 + a4 + .... + ak 2 3 (k 2) = 12m + 3
≤ ⇒ ≥ − ⇒
a1 + a2 ≤ 1, we get a contradiction. From that we can conclude that
a3 ≤ 2 ⇒ a1 ≤ a2 2. Applying Theorem 2 and 3, we can show that set
shows ≤that the value k = of
of subsets + 3 weight.
4k equal cannot Example
be reduced. A can
in fact be separated into two 3
Proposition 4 (Ininteger).
4 (m is a positive case N divided
Given by 6 yields
A(2N, a remainder
k) where s(ai) = of
ai 4).NLet
andN k= 6m
4m =
+
4, then set A can be separated into two subsets of equal weight and the value of
k cannot be reduced. ≤
≥ N
Proof. Since k = 4m + 4 ≥ 3m + 4 = + 2, due to Proposition 1,
2
a3 ≤ a4 ≤ 3.
If a3 = 3 a3 + a4 + .... + ak 3( k 2) = 12m + 6 a1 + a2 2, then
⇒ ≥ − ⇒

a1 = a2 = 1.
Applying Theorem 2 and 3 (when a3 ≤ 2), we can show that set A can
value of k = 4k
be separated + two
into 4 cannot be of
subsets equal weight.QExample 4 shows that the
reduced.
Proposition
integer). Given 5 A
(In(2N,
casek)Nwhere
is divisible
s(ai) = aby 6). = 6m
LetkN 3m
N. Es + 2,(mthen
is set
a positive
A can
i
be separated into two subsets of equal weight and the value of k cannot be
reduced. ≤
6m N
Note: k = 3m + 2 = + 2 = + 2.
Proof.
2 2 ≤ a ≤ a ≤ a ≤ 3. There are the
Due to Proposition 1, a1 2 3 4
following scenarios:
a)IftoA3,begins (1, Theorem
withcase
in which 1) or (1, 22) can
values, then A(2N, k) can be ex- pressed up
be used.
b)If it begins with (2, 2) values, then Theorem 3 can be used.
c)There
cases are
a2 =two
a3 =cases
a4 = left
3 iswhen it beginsofeither
true because with (1,1.3In
Proposition ) or (2,case
this 3). In
weboth
can
classify the elements based on their weight.
C = {e | es weight is divisible by 3} = {c1; c2; ; cu}

K1 = {a | as weight yields a remainder of 1 when divided by 3} =


{k1; k2; ; kv}

K2 = {a | as weight yields a remainder of 2 when divided by 3} =


{h1; h2; ; hw}
This way A or=inC KK ∪ 1 K2 and C K1 ∩ = C K1 = K +K22. =From
∅ (the element
K2 wea
1 or K∪2). Here k = u + v + w =∩
1
belongs in C 3m K1 and
can create a fourth set B so that bi = (ki + hi) until we ran out of either v or w.
For instance, in case v is ∩ larger, then we can
take one element each from K1 and K2 until we ran out of K2 elements,
element.
take threeSince 2N =from
elements 12mK(divisible by 3), the task can be done without then we
1 at the same time to create a new any elements left
over.
can beThus, we by
divided get3,sets
andCCand B, the atelements
contains of which
least 3 pieces of 3. The number of
v− w
elements in set B is w + (a non-negative integer). Let
3
e
E = { | e either from C or B}
3 of elements with integers forv weight.
E is a set − w Leg the number
v 2ofv ele-v 2w
ments in set E be r, then r = u + w + =u+ + + + =
3 3 3 3 3
v 2w
u+ + .
3 3
u
Since u has at least 3 elements, u ≥ 2 + . We get:
u 3v 3m
v 2w w
r = u + 3+ 3 ≥ 2+ 3 =2+m
≥ 2 + 3 + 3 +1 3
At the same time the weight of E is exactly of the weight of A, thus
3
s(E)need
We = 4m.
to show that the elements of E are not heavier than 2m. For this
it is enough to prove that ak ak−2 + ak−1 + ak
−1
+ a k ≤ 6m + 2 ⇔ ≤
2 3
2m + .
3 −2 + ak

Due
than to
2m.the construction, the elements are integers, so their weight is less

Truly: a1 + a2 + .... + ak−3 1 +≥


3(k 4) = 1 −
+ 3(3m 2) = 9m 5. −
All conditions of Theorem 2 are met. Thus, − E can be separated into two
subsets of equal weight. We restore the original weights and disassem- ble the
elements from sets K1 and K2 then set A can be separated into two subsets of
equal weight, proving the Theorem.
Example 5 shows that the value of k = 3m + 2 cannot be reduced.
≥.
N
Note: This explains why we began with k + 2Σ.
2
Theorem
= ai ≤ N, 4. Letset
then NA beisa fully
positive odd integer. Given A(2N, N + 1) where s(ai)
expressible.
Proof. Apply Theorem 2 and 1, utilising the fact that the number of elements is
rather big (the average weight is smaller than 2). Let the detailed proof be
homework.
Summary:

Theorem 5. Given
into two subsets A(2N,
of equal k) where
weight if: s(ai) = ai N. Set A can be separated

1)K = N + 1 if N is odd;
2)K = 4m + 3 if N = 6m + 2;

3)K = 4m + 4 if N = 6m + 4;

4)K = 3m + 2 if N = 6m.
Lastly, let us mention the problem that began this whole paper:
Problem. There are 35 positive integers, none larger than 50. Their sum is 100.
Can they be separated into two different groups with equal sums.
Answer. Indeed they can, this is a case where N = 50 and k = 4 8 + 3·=
35.

3 Exercises
I found the following exercises a very good opportunity to craft prob- lems for
mathematics competitions as well as bases for further research.

1)There are 100 objects with weights expressed as positive integers, with the
total weight of 198. If n is an arbitrary number not larger than 100, can a
few objects be picked so that their total weight is n?

2)Given n positive integers, prove that it is always possible to pick a few (in
some cases even one) so that their sum is divisible by n.
3)Prove thatsoif that
integers you their
cannot pick
sum a few numbers
is divisible (n 1) all
fromthese numbers
by n, then positive

yield the same remainder when divided by n.
N and k
4)Given A(2N, k), s(ai) = ai ≤ + 2. Denote P(A) as
2
N
≥ 2
the largest fully expressible subset of set A. Prove that if P(A) is
expressible up to 3, then A is fully expressible!
N
5)Given A(2N, k) where N ≥ 2, s(ai) = ai ≤ N and k ≥ + 2.
Furthermore, let a2 2
= 2. Prove that set A can be separated into two
subsets of equal weight.
6)There are 20 objects the weights of which can be expressed as pos- itive
integers that are not larger than 28. Their total weight is 56. Prove that they
can be separated into two groups of equal weight.
7)There are 30 objects with the weights being positive integers not larger than
36 kg. Their total weight is 80 kg. Prove that they can be separated into
two groups of equal weight.
8)There are 20 objects with the weights being positive integers not larger than
36. Their total weight is 72. Can they be separated into two groups of equal
weight?
9)There are 30 objects the weights of which are positive integers but do not
weigh more than 24. Their total weight is 72. Prove that they can be
arranged into three groups of equal total weight!
10) There are 24 objects that weigh positive integers but do not weigh more than
24 kg. They weigh 72 in total. Can they be separated into three groups of
equal weight?

4 Afterword
When in 2014 I could not find a detailed proof for this problem, I sub- mitted it
to Komal Forum [4] (Problem 3906 07/2014) to help me find a source for the
solution. However, it was not meant to be. Later on, me and Erben Peter
worked out a solution for the problem through painstaking work through the
Forum.
I also submitted the problem to the Bai Toan Hay Loi−Giai Dep on- line
mathematics forum [1], where it garnered great interest. The most tenacious
colleagues working on it (they were only students at the time) were Nguyen Hai
Dang and Nguyen Thanh Khang. We have already published our joint work in
the publication of the Ha Noi Mathematics Association [3] (2015).
That publication concentrated on the quantitative summary of our re- search
results to provide direct assistance to students who are either looking to continue
our research or are working on similar topics.
The publication in your hands contains results that have been reworked and
proofs that are simpler and more compact.
I would like to seize this opportunity to once again thank my colleagues for their
co-operation.

References
[1] Online Forum Bai Toan Hay Loi Giai Dep......(2014)

[2] Erds Bull.
Theory. Pl, Abraham
ResearchGinzburg
Council,,Israel,
Abraham10F;Ziv Theorem
41-43; 1961.in additive num- ber
[3] HMS Proceeding Lai Chau Seminar, (2015), 73 88
[4] Komal Forum 3906 problema

(2014) jul, Online Forum.

[5] Lovsz
beyond, L., Pelikn J., Vesztergombi
Springer-Verlag, 2003. K Discrete mathematics. Elementary and

Ha Noi, 20/3/2019.

N. V. Loi
Email: loiscenter@gmail.com.
ON A CLASS OF FUNCTIONAL EQUATIONS IN PLANE
GEOMETRY

Nguyen Van Mau Hanoi


University of Science

Let Γ be the set of all triples of positive numbers (A, B, C) such that

A + B + C = π,
i.e. every of
all triples triple (A, B,
positive Γ isb,the
C) ∈ (a,
numbers c) angles of a triangle, and let F be the set of
satisfying

|b − c| < a < b + c,
i.e.
Ourevery
main triple
purpose(a, is
b, c) F is the side lengths of a triangle.

- Determine all function f : (0, π) → (0, π) such that ( f (A), f (B), f (C)) ∈ Γ
for all (A, B, C) ∈ Γ.
- Determine all function f : (0, ∞) → (0, ∞) such that ( f (a), f (b), f (c)) ∈ F

for all (a, b, c) ∈ F.


1 Introduction

thatInAthe B + C =letπ,Γ i.e.


+ sequel, be the set triple
every of all triples
(A, B, ofC)positive
Γ is thenumbers (A,a B,
angles of (A,C) such
B,
C) such that A + B + C = π. triangle, and denote by Γ0
the set of all triples of non-negative numbers ∈
Let F be the set of all triples of positive numbers (a, b, c) such that

|b − c| < a < b + c,

i.e. every triple (a, b, c) ∈ F is the side lengths of a triangle.


The main purpose of the paper is to find the general solutions of the
following functional equations.
Question 1. Determine all functions f : (0, π) → (0, π) such that ( f (A), f (B), f (C)) ∈
Γ for all (A, B, C) ∈ Γ.
Question 2. Determine all functions f : (0, ∞) → (0, ∞) such that ( f (a), f (b), f (c)) ∈

F for all (a, b, c) ∈ F.


2 General solution of functional equations involving an-
gles of triangles
In this section, we consider Problem 1. We start by finding all differ- entiable
solutions.
Proposition
f satisfies ( f (1.
A),Suppose
f (B), f (that
C)) ∈f :Γ (for )→
0, πall 0, πC)) is
(A,(B, ∈differentiable
Γ. Then thereand that
exists
a ∈ [−2 1 , 1] such that
π(1 − a)
f (x) = ax + .
3
Proof. We have

f (x) + f (y) + f (π − x − y) = π, ∀x, y, x + y ∈ (0, π). (1)


Taking derivative in x we obtain

f J ( x ) − f J (π − x − y) = 0, ∀ x, y, x + y ∈ (0, π ). (2)

It follows that f J is constant in (0, π ), hence f ( x ) = ax + b for some constants


a and b. Since f (x) + f (y) + f (π − x − y) = π we deduce that b = π(1−a).
3

Since f only takes values in (0, π) we must have a ∈ [− 1 2, 1].


We now show that the Proposition still holds without the differentiability
requirement.

Theorem 9. Suppose that f : (0, π) → (0, π) satisfies ( f (A), f (B), f (C)) ∈

Γ for all (A, B, C) ∈ Γ. Then there exists a ∈ [− 1 ,2 1] such that


π(1 − a)
f (x) = ax + .
3
Proof. Let g : (− π3, 2π3 ) → (− π3 , 2π3 ) be defined by
π π
g(x) = f (x + ) − .
3 3
By (1),
g ( x ) + g( y) + g (z ) = 0 if x + y + z = 0.

We deduce that g(0) = 0 and hence,

π 2π π
g ( x ) + g ( y ) = g ( x + y) for x, y ∈ (−3 , 3 ), x + y < 3
Since, g is also bounded from below, it follows that g must be linear on
(− π , 2π ) (see [1]-[3]). Therefore, g(x) = ax for some a and hence,
3 3

π(1 − a)
f (x) = ax + .
3

Since f only takes values in (0, π) we must have a ∈ [− 1 , 1].


3 General solution of functional equations involving side
lengths of triangles
In this section we turn our attention to Problem 2. We first have some easy
observations.

• f (x) = αx + β satisfies ( f (a), f (b), f (c)) ∈ F for all (a, b, c) F if


and only if α, β “ 0 and α + β > 0.
1
• f ( x) = satisfies ( f (a), f (b), f (c)) ∈ F for all (a, b, c) ∈ F if
αx + β
and only if α = 0, β > 0.
Let O be a cirle with diameter length 1, and M be the set of all tri-
angles inscribed in the cirle O. We next consider the case f maps M to
M.
Theorem 10.allAny
(c)) ∈ M for function
(a, b, c) ∈ Mf is of : (the ) → (0, 1) such that ( f (a), f (b), f
0, 1form
π (1 − a )
f ( x ) = sin .a arcsin x + Σ, (3)
1
2
Proof. By the sine theorem, α, β and γ are the3angles of a triangle if
and only if sin α, sin β, and sin γ are the side-lengths of a triangle in M.
Appyling Theorem 15 to the function
for some a ∈ [− , 1].
obtain the conclusion. g(x) = arcsin( f (arcsin x)), we
We now formulate our main result.

Theorem
(c)) ∈ F for 11.allAny c) ∈ Ffis: of(0,the
function
(a, b, → (0, ∞) satisfying ( f (a), f (b), f
∞)form
(1 − α)π 1
f ( x ) = u sin .α arcsin θ ( x ) + Σ, − ™ α ™ 1, u > 0, (4)
where θ ( x) is difined as follows; for every given triple (a, b, c) F with
x
the circumscribed radius R then of θ ( x) =3 . Proof.2 Applying the
2R
above additional discussion and theorem 10, it is easy to obtain the form ∈
(4).
Remark 1. Some other types of functional equations in geometry were
considered by S. Galab [4].

References
[1]T. Acze’l, Lectures on functional equations and their applications, Aca-
demic Press, New York/San Francisco/London, m1966.
[2]M. Kuczma, B. Choczewski, R. Ger, Interative Functional Equa- tions,
Cambridge University Press, Cambridge/New York/Port
Chester/Melbourne/Sydney, 1990.
[3]P.K. Sahoo, T. Riedel, Mean Value Theorems and Functional Equations,
World Scientific, Singapore/New Jersey/London/HongKong, 1998.
[4]S. Galab, Functional equations in geometry, Prace Mat., NoCCXXIII,
Zeszyt 14, 1969.
ON THE R EARRANGEMENT I NEQUALITY
Nguyen Van Ngoc Thang
Long University, Hanoi

Rearrangement inequality or permutation inequality is an elementary in- equality


and can be used to prove many famous inequalities. In this note we reveal a nice
result that provides a very simple but powerful inequality that can be used for
proving many inequalities.

1 Triples of real numbers


1.1 Difinition

Definition 1. Let’s
bers. The triples (a1, consider
a2, a3) andtwo , b2, b2)(aare
(b1triples 1, a2, a3) and (b1, b2, b2) of real num-
said to be:
similarly arranged if both are increasing (i. e. a1 a2 a3 and b1 b2

b3) or both are decreasing (i. e. a1 ≥ a2 ≥ a3 and b1 ≤ ≥ b2 ≤ b3). ≤
• oppositely arranged if one is increasing≥and the other ≤ is
decreasing.
• Let triples (a1,and
a2, a3) and c(b)1,beb2any , b2)permutation
be similarly of arranged or bboth be oppo-
sitely a1b1 + a2b2 + a(c3b1,3 cis2, called
(i) S =arranged, 3
the Sorted numbers.1, b2, b3.
sum of thenumbers
(ii) R = a1b3 + a2b2 + a3b1 is called the Reversed sum of the numbers.

(iii) P = a1c1 + a2c2 + a3c3 is called the Permutated sum of the numbers.
Example 9.
1. (1, 3, 5)arranged.
oppositely and (2, 4, 6) are similarly arranged while (1, 3, 5) and (6, 4, 2) are
2. If 0 < a ≤ b ≤ c, then (a, b, c) and ( 1 , 1 , 1 ) are oppositely arranged, while

(a, b, c) and ( 1b+c , 1c+a, 1a+b


) are similarly
abc
arranged.
3. m Ifm0 < a ≤ b c and m is a positive real number, then (a, b, c) and
(a , b , cm) are similarly arranged while (a, b, c) and ( 1 , 1 , 1 ) are oppo-
sitely arranged. am bm cm
4. 0< a
ar-Ifranged. b c, then (a, b, c) and (log a, log b, log c) are similarly

1.2 The Rearrangement Inequality

Theorem 12.
S ≥ P ≥ R. (5)

Proof.
and let Let’s
(c1, c2take
, c3) two
be atriples (a1, a2of
, a3()band (b1, b2, b3) increasingly
c1 ≥ ar-
c2. ranged
permutation 1, b2, b3). Assume that

Let Q = a1 c1 + a2 c2 + a3 c3 and QJ = a1 c2 + a2 c1 + a3 c3. We have


Q J − Q = a 1 c2 + a 2 c 1 − a 1 c 1 − a 2 c 2
= a2 ( c1 − c 2 ) − a1 ( c1 − c 2 )

= (c1 − c2)(a2 − a1) ≥ 0.


value ≥
of the sum Q. Therefore, if we interchange all pairs (ci , c j ) so that Hence
QJ Q. That is, interchanging c1 and c2 can only increase the that
corresponds to (b1, b2, b3 ), that is S = a1 b1 + a2 b2 + a3 b3 . ci c j for
i < j, the sum can only get larger. The lagest sum is that one

The above argument works similarly if (a1, a2, a3) and (b1, b2, b3) are
both decreasing.

The equality (5) occurs if and only if b1 = b2 = b3 or a1 = a2 = a3.


1.3 Problems
Problem 1. Let a, b, c be real numbers. Prove the inequality

a5 + b5 + c5 ≥ a4b + b4c + c4a.



Solution. Without loss of generality we may assume that a b c, and
4 4 4
clearly that a ≥ b c . Now by the rearrangement inequality we get the
required inequality. Equality occurs iff a=b=c.
Problem 2. Let a, b, c, ∈ R. Then

(i) a2 + b2 + c2 ≥ ab + bc + ca,

(ii) an + bn + cn ≥ an−1b + bn−1c + cn−1a for every even positive integer n.


Since the triplets (a, b, c) case (an−1
and of , bn−1show
, cn−1(ii).
) are similarly ≤
Solution. (i) is a particular (ii).Let’s Assume a b arranged by
c. using (5)
we get

aan−1 + bbn−1 + ccn−1 ≥ abn−1 + bcn−1 + can−1,


which is just (ii).
Problem 3. If a, b, c >, 0 then

a+b+c 1 1 1
(i ) abc + + ,
a2 b2 c2

a2 b2 c2 b c a
(ii) + + ≥ + + ,
ba22 c2b2 a2 c2 a b c
(iii) + + a ≥ a + b + c.
Solution. We may assume that b ac≤ a ≤ b.
1 1 1 1 1 1
(i) The triplets . , , Σ and . , , Σ are similarly arranged. Thus
a b c abc
11 11 11 11 11 11
+ + ≥ + + ,
that is (i). aa bb cc ab bc ca
a b c a c
(ii) The triplets . , , Σ and . Σ are similarly arranged. Hence
b
, ,
b c a bca
aa bb cc ab bc ca
+ + ≥ + + ,
which is (ii). bb cc a a b c c a a b
.1 1 1 Σ
2 2 2
(iii) The triplets (a , b , c ) and , , are opposetely arranged. Hence
1 1 a1 b c 1 1 1
a +b +c
2 2 2
≤a +b +c , 2 2 2
a b c b c a
that is, just (iii).
Problem 4. Let a,2b, c 2be positive
2 2
real numbers.
2 2
Prove the inequality
a +c b +a c +b

+ + ≥ 2 ( a + b + c) .
b c a
Solution. Since the given inequality is symmetic, without loss of gener-
ality we may assume that a ≥ b ≥ c. Then clearly
1≥1
1 b c.
a2 + b2 ≥ c2 and ≥ a
By the Rearrangement inequality we have
a2 b2 c2
21 1 1 21 1 1
+b2 +c2 + b 2 + c 2 = a + b + c, (6)
+ b c ≥ a b c
a
and b2 + =a
a c a
21 1 21 1 1
b 2 c2 +b2 +c 1 + b 2 + c 2 = a + b + c. (7)
2
+ c a ≥ a b c
c + =a b
a
Adding (1)a andb (2) yields the required inequality.
Problem 5. (Neshbit’s inequality). If a, b, c > 0, then
a
+ b
+ c
≥ . 3
b + c c + a a + b1 1 2
Solution. The triplets (a, b, c) and ( b 1 , , ) are similarly arranged
(we may assume a ≤ b ≤ c). Thus
+c c+a a+b
1 1 1
1 1 1 ≥a +b +c ,
a +b +c

and also
b+c c+a a+b c+a a+b b+c
1 1 1

b +1 c c1+ a a1+ b ≥ a a + b + b b + c + c c + a .
a +b +c
Adding these two inequalities we fine

.b +a c c b+ a c b Σ
a+ c +a a a+
bb +b b+c= c3.
2 + + + a +
≥a + b +b + c c++ a c
if a
required inequality =
is oblained. = only c. Therefore, the
b holds if and
Equality
Problem 6. (Neshbit’s generalized Inequality). If a, b, c > 0, and n is a
positive integer, then
an + b n + c n≥ an−1 + bn−1 + cn−1 .
b+c c+a a+b 2

Solution.
1 1Assume 1 that a b c. Then tripltes (an, bn, cn) and
. , , , Σ are similarly arranged. Hence,
b+c c+a a+b
b +1 c c1+ a n a1+ b c 1+ a a1+ b n b1+ c
an + bn +c ≥ an + bn +c
1 1 1
n a1+ b ≥ a + bn + cn
n
nb +1c n c1+ a .
a +b +c a+b b+c c+a
Adding these two inequalities we fine
2. an c +b
bn cn Σ a n + cn bn + an n n. (8)
+ + ≥ + +
b+c c+a a+b c+a a+b
We have b+c
an + bn
1 n−1 n−1
−n−1
a+b + bn−1 ) ⇔ (a − b)(a b ) ≥ 0, ∀a, b > 0, n ≥ 1.
2

(a
Therefore the right side ofn (8) nbecomes
an + cn b +a cn + bn
+ + n−1
. (9)
c+a a+b ≥
b+c +b n−1
+c n−1

a
From (8) and (9) follows the our result. Equality holds if and only if

a = b = c.
Problem 7. Suppose a, b, c are the lengths of the sides of a triangle. Prove that
a b c

Solution. Suppose that the triplet (a, b, c) is increasing. Then the triplet
1 1 1 is increasing also. By the Rearrangement
, ,
b+c− ac+a− ba+b− c
Inequality we have
a + b + c ≥ b + c + a ,
b+c− a c+a− b a+b− c b+c− a c+a− b a+b− c
a + b + c c + a + b .
b≥+ c a c + a b a+b c b+c− a c+a− b a+b− c
− −
Put
− a b c
Q= + + .
b + c −a
Adding these inequalities we get c+a− b a+b− c
b+c c +a a+b
2Q ≥ + +
b+c− a c+a− b a+b− c
b+c− a+a c+a− b+b a+b− c+c
= − + − + −
b+ca a c+a c a+b c

− −
=3+ + bb − + = 3 + Q.
b+c a c+a b a+b c
which is equivalent to the desired inequality.
Problem 8. Let x, y, z > 0 be real numbers. Prove the inequality
x2 z2 y2 x2 z2 y2
− +
y+z − + −
z+x
≥ 0.
x+y
Solution. We need to prove that
x2 + y2 + z2≥ z+2 +x2 . y2
y+z z+x x+y y+z z+x x+y
Without loss of generality we may assume that x ≥ y ≥ z(since the
given innequality is cyclic we also will consider case z ≥ y ≥ x). Then
clear 1 1 1
≥ .
x2 ≥ y2 ≥ z2 and + z+x x+y
y z

By the Rearrangement inequality we have
x2 + y2 + z2≥ z+2 +x2 , y2
y+z z+x x+y y+z z+x x+y

as required.
If we assume z 2≥ y ≥ x, then 1
z y2 x2 and 1 1

≥ ≥ .
≥ x+z z+y
x+y
By the Rearrangement inequality we obtain
x2 y2 z2 2 1 2 1 2 1
+ + =z +x +y
y+z z+x x+y 2 x +
1 y y +
1 z
≥z +x2 +y z+x
2 1
x+y z+x
2 2 2
z+x
= + + . z x y
y+z z+x x+y

Equality occurs if and only if x = y = z.


Problem 9. Let x, y, z > 0. Prove that
x3 y3 z3

+ + ≥ x + y + z.
yz zx xy is symmetric we may assume that
Solution. Since the given inequality
x ≥ y ≥ z. Then
1 1
x3 ≥ y3 ≥ z3 and ≥ 1
y zx xy.
By the Rearrangement inequality we havez ≥

x3 y3 z3 3 1 3 1 1
+ + =x +z3
yz zx xy + y zx xy
yz x2 y2 z
1 31 2. (10)
≥ x yz+ y yz+ z 3 zx
1
3
= x
We will prove that + +
y z
2 2 2
x y z
+ + ≥ x + y + z. (11)
y z x1 1 1 (since the inequality
Let x ≥ y ≥ z. Then x2 ≥ y2 ≥ z2 and ≥ ≥ x
z case
(11) is cyclic we also need to consider the y z ≥ y x). By the
Rearrangement inequality we obtain
x2 y2 z2
x2 y2 z2

+ + ≥
y z x + + = x + y + z.
x y z
The case when z ≥ y x is analogus to the previous case. Now by (10) and
(11) we have
x3 y3 z3
+ + ≥ x + y + z.
yz zx xy
The equality occurs if and only if x = y = z.

Problem 10. Let a, b, c, d be positive real numbers such that a + b + c + d = 4.


Prove the inequality
a2bc + b2cd + c2da + d2ab ≤ 4.

Solution. Let (x, y, z, t) be a permutation of (a, b, c, d) such that x ≥y


z ≥ t. Then
inequality clearly xyz ≥ xyt ≥ xzt
we obtain yzt. By the Rearrangement

Since AMx.xyz
≥ GM+ y.xyt + z.xzt + t.yzt ≥ a2bc + b2cd + c2da + d2ab.
we deduce (12)
t.yzt
x.xyz + y.xyt + z.xzt + = (xy + zt)(xz + yt) (xy + xz + yt + zt)2
4 .


(13)
Since
(x + y + z + t )2

xy + xz + yt + zt = (x + z)(y + t) ≤ =4
by (13) we deduce that 4

x.xyz + y.xyt + z.xzt + t.yzt ≤ 4.


Finally by (12) we obtain

a2bc + b2cd + c2da + d2ab ≤ 4


and we are done. Equality holds iff a = b = c = d = 1 or a = 2b = c =

1, d = 0(up to permutation).
Problem 11. (IMO, 1995). Let a, b, c be positive real numbers such that abc=1.
Prove that + 1 + 1≥ . 3
a3(b 1+ c) b3(c + a) c3(a + b) 2
1 1 1
Solution. Set x = , y = x = , z = x = . Since abc = 1 we obtain
xyz = a b c
1. With these new notations the required inequality transforms
into
x2 + y2 + z2≥ . 3
y+z x+z x+y 2
In virtue of the Neshbit’s generalized and Am-GM inequalities, we hve
x2 y2 z2 x+y+z √ xyz 3
3
+ + ≥
y+z x+z x+y
≥ = .
2 2 2
Equality holds if and only if x = y = z ⇔ a = b = c.
Problem 12. (Chbyshev’s
ilarly arranged, then inequality). If (a1, a2, a3) and (b1, b2, b3) are sim-
≥. Σ. Σ.

a1b1 + a2b32 + a3b3 a1 + a23 + a3 b1 + b32 + b3


Solution. Using the Rearrangement inequality we get:

a+1ba1 b+ a+2ba2 + a3b3 = a1b1 + a2b2 + a3b3, a1b1


2 2 3b3 ≥ a1b2 + a2b3 + a3b1, a1b1 +
a2b2 + a3b3 ≥ a1b3 + a2b1 + a3b2.
Adding these inequalities we obtain

3(a1b1 + a2b2 + a3b3) ≥ a1(b1 + b2 + b3) + a2(b1 + b2 + b3) + a3(b1 + b2 + b3)

= (a1 + a2 + a3)(b1 + b2 + b3),

which is the desired result. Equality holds if a1 = a2 = a3 or b1 = b2 = b3.


Problem 13. (1974, USA Math Olympiad). If a, b, c > 0, then
a+b+c
a b c
abc abc .
≥( ) 3

Solution. We may assume that a ≤b


c. Then (a, b, c) and (log a, log b, log c)
are similar arranged. Using the Chebyshev’s inequality, we have
a+b+c
a log a + b log b + c log c ≥ (log a + logb + log c),
3
or

log(aabbcc) ≥ log(abc) a+b+c


3
,
which is the desired result.
c − a) 1964).
a2(b + (IMO, + b2(c Let −bband
+ a a, ) + cc2(be b−
a +the ≤ 3abc.
c) of Problem 14.
Solution. We may assume that a b sides
c. Wea first
triangle.
proveProve
that cthat
(a + b
cc) a≥ b(cThe
) 0. +a − b) ≥
second a(inequality
b + c a− ). Note
can be c(a + bin cthe
thatproved ) − (c +−a manner.
bsame b) = (bByc)(bthe
+
Rearrangement−inequality, we have
− −
≥ ≥
a2≥
(b + c − a) + b2(c + a − b)−+ c2(a + b − c)
a≤
2 ba ( b + c − a)2 + cb (c + a − b)2 + ac ( a + b − c),
( b + c − a) + b ( c + a − b) + c ( a + b − c)

≤ ca(b + c − a) + ab(c + a − b) + bc(a + b − c).


Adding these two inequalities, the right side simplifies to 6abc. The de-
sired inequality now follows.
a2b(a − b1983). b −
) + b2c(Let a, bc)and − sides
+ c2ca(bec the a) ≥of 0. Problem
that a15. (IMO,
Solution. Wemay assume that a b, c. If aa triangle.
b Prove
c, we have (b + c
a) b(c + a b) c(a + b c) as in Problem 14. By the Rearrangement inequality,
≤ − ≤
1 1 1
a(b + −
c − a) + b(c + a − b) + c(a + b − c)
≥ ≥ ≥
c a b
1 1 1 −

≤ a(b + c − a) + a(c + a − b) + a(a + b − c) = a + b + c.


a b c
1 1 1
This simplifies to a(b − a) + b(c − b) + c(a − c) ≤ 0, which is equiv-
c a b
alent to the desired inequality. If a ≥ c ≥ b, then a(b + c − a) ≤

c( a + b − c) ≤ b ( c + a − b ) .
Problem 16. Let a, b, c > 0. Prove that
a8 + b8 + c8 1 1 1

a 3b 3c 3 + + .
a b c
Problem 17. Leta ≥ b ≥ c and 0 ≤ x ≤ y ≤ z. Prove that
a b
c
a+b+c
≥ √ xyz .a+b+cΣ
x + y + z 3
≥ 3 x+y+z .
Problem 18. Let a, b, c > 0. Prove that
√ √ √
ab + bc + ca ≥ a bc + b ac + c ab

Problem 19. Let a, b, c > 0 such that a2 + b2 + c2 ≥ 3. Show that


a2 + a2 + a2≥ . 3
b+c b+c b+c 2
Problem 20. If a, b, c > 0, then
.
a+b+c an + b n + cn
3 .

2 Generality
n

3
2.1 Definition
Definition 2. Let a1 ≤ a2 ≤ · · · ≤an and b1 ≤ b2 ≤ · · · bn be any real
number sequences. ≤
(a) S(n) = a1b1 + a2b2 + · · · + anbn is called the Sorted sum of the numbers.
bers.R(n) = a1bn + a2bn−1 +
(b)
··
+ anb1 is called the Reversed sum of the num-
bers,P(where
(c) n) = ac1c,1 c+, a. 2.c.2 c+ be any·+permutation
1 2 n ··
ancn is calledofthe
thePermutated
numbers b1sum
, b2, .of. .the
, bnnum-
.
·
2.2 The Rearrangement Inequality

Theorem 13.
S( n) ≥ P( n) ≥ R(n). (14)

The equality (14) occurs if and only if b1 = b2 = . . . bn or a1 = a2 = · · · = an.


Proof.
(i) S(n) ≥ P(n) : S(1) P(1) is obviously true. Assume S(k) P(k) is true for
some k N. For P(k + 1),≥since cJs are the permutations of the bJs, suppose bk+1
= ci and ck+1 = bj. ∈ ≥

(ak+1 − ai )(bk+1 − bj) ≥ 0 ⇔ a ia j + ak+1bk+1 ≥ aibk+1 + ak+1bj


⇒ aibj + ak+1bk+1 ≥ aici + ak+1ck+1.
So k+1 to get a possibly large sum. Af- P (k ).
So, inS(P(k+1),
k + 1) we≥ may
P(k switch and ctrue.
+ 1) isci also By the principle of mathemat- ical
induction, S( n) ≥ P(n) is true ∀n ∈ N. ter switching
of these terms, we come up with the inductive hypothesis
(ii) The inequality P(n) ≥ R(n) follows easily from S(n) ≥ P(n) by
replaing b1 ≤ b2 ≤ . . . bn by −bn ≥ −bn−1 ≥ · · · ≥ −b1.
Corollary 1. Let a1, a2, . . . , an be real numbers and c1, c2, . . . , cn be its permu-
tation. Then

a21 + a22+.........+ a2n ≥ a1c1 + a2c2 + · · · + ancn.


Corollary 2. Let a1, a2, . . . , an be positive real numbers and c1, c2, . . . , cn be its
permutation. Then
c c
a1 1 + a22 ≥ n.
an
+···+

cn

2.3 Classical Inequalities


2.3.1 Arithmetic Mean-Geometric Mean Inequality AM ≥ GM

Let x1, x2, . . . , xn be positive numbers. Then


x1 + x2 + n· · · + xn
≥ x1x2 . . . xn. (15)
√n

Equality holds if and only if x1 = x2 = ·· =x .


n
Proof. Let ·
√n x1
G= x1x2 . . . xn, a1 = , a2 =
x1 x2 x1 x2 . . . xn
,..., an = = 1.
G G G
By Corollary 2,
a1 a2 an x1 x2 xn
n≤ + +... = + +... ,
an a1 an−1 G G G
which is equivalent to the inequality (15).

2.3.2 Geometric Mean-Harmonic Mean Inequality GM ≥ HM

√npositive numbers. Then n


Let x1, x2, . . . , xn be
x x ...x . (16)
≥1 2 n
x11 x2 x
+ 1 + · · · + 1n
Equality holds if and only if x1 = x2 =
·· =x .
· as in nthe proof of AM-GM. By
Proof. Let G and a1, a2, . . . , an similarly
Corollary 2,
a1 a2 an G G G
n≤ + +... = + +... ,
a2 a3 a1 x1 x2 xn
which is equivalent to the inequality (16).

2.3.3 Root Mean Square-Arithmetic Mean Inequality RMS ≥ AM

Let x1, x2, . . . , xn be real numbers. Then


2
.x + x 2 + · · · + x2 x1 + x2 + · · · + xn

1 n 2
. (17)
n ·· n
Equality holds if and only if x1 = x2 = · = xn. Proof. By Corollary 7, we
have

x21 + x22+ · · · + x2n= x1x1 + x2x2 + · · · + xnxn,


x21 + x22+ · · · + x2n≥ x1x2 + x2x3 + · · · + xnx1,
x21 + x22+ · · · + x2n≥ x1x3 + x2x4 + · · · + xnx2,
........................≥ .................................
1 2 n
x2 + x2 + · · · + x2 ≥ x1xn + x2x1 + · · · + xnxn−1.
Adding all inequalities together, we have

n(x21+ x2 2+ · · · + x2 n) ≥ (x1 + x2 + · · · + xn)2,


which is equivalent to (17). The equality holds if and only if x1 = x2 =

· · · = xn.
2.3.4 Cauchy-Bunyakowskii- Schwarz Inequality (CBS-Inequality)
Let a1, a2, . . . , an and b1, b2, . . . , bn be real number sequences. Then
(a1b1 + a2b2 + · · · + anbn)2 ≤ (a2 + a2 + · · · + a2 )(b2 + b2 + · · · + b2 ),
1 2 n 1 2 n
(18)
with
or bi equality if and
= kai, for 1 ≤onlyi if≤forn.some
Proof. 1 2 · =1 a≤
constantIf ak, a=i a= =kb·i, ·for i ≤ n
n = 0 or

2 = · · · =2n band
.ab211 +=a2b2+ n =Q 0, =theb .result is trivial. Otherwise, define P =
· · · a+i a i 21
bfor+ 1b2≤
2+ · · · + b2n. Since both are non-
i ≤ n. By Corollary 7,
zero, we let xi = and xn+i =
P Q
a2 + a2 + · · · + a2 b2 + b2 + · · · + b2
2
2 P2 n 2 Q2 n
= 1 + 1
= x21+ x2 +2 · · · + x2 n
≥ x1 xn+1 + x2 xn+2 + · · · + xnx2n + xn+1 x1 + xn+2 x2 + · · · + x2nxn
2(a1b1 + a2b2 + · · · + anbn)
which=is equivalent toPQ ,
the desired result. Equality holds if and only if

xi = xn+1 for 1 ≤ i ≤ n, or aiQ = biP for 1 ≤ i ≤ n.


2.3.5 Chebyshev’s Inequality

Let x1 ≤ x2 ≤ . . . xn and y1 ≤ y2 . . . yn be any numbers sequences.


Then ≤

x1y1 + x2y2 + · · · + xnyn x1 + x2 + · · · + xn y1 + y2 + · · · + yn


n ≥ n . n
≥ x1yn + x2yn−1 + · · · + xny1
n . (19)
x1y1 + x2y2 + · · · + xnyn = x1y1 + x2y2 + · · · + xnyn Proof. By
Rearrangement inequality, we
≥ xcyclically
y + x yrotate + xyiy, ,
+ · x·i ·and
1 n 2 n−1 n 1
x1y1 + x2y2 + · · · + xnyn ≥ x1y2 + x2y3 + · · · + xny1

≥ x1yn + x2yn−1 + · · · + xny1,


. . . . . . . . . . . . . . . . . . . ≥ . . . . . . . . . . . . . . . . . . . . . .,
x1y1 + x2y2 + · · · + xnyn ≥ x1yn + x2yn−1 + · · · + xny1

=x1yn + x2yn−1 + · · · + xny1.


Adding up the inequalities and divide by n2, we get our result.

2.4 Problems
Problem 21 (IMO, 1975). Let x1 x2 xn and y1 y2 yn
≤ ≤···≤ ≤ ≤···
be real numbers. Let (z1, z2, . . . , zn) ≤
be a permutation of (y1, y2, . . . , yn).
Prove that

(x1 − y1)2 + (x2 − y2)2 + · · · + (xn − yn)2

≤ (x1 − z1)2 + (x2 − z2)2 + · · · + (xn − zn)2.


Solution. Note that we have y2 +1y2 + 2· · · + y2 =nz2 + z12 + ·2· · + z2 . After
n
expansion and simplification, the desired inequality is equivalent to
x 1y 1 + x 2y 2 + · · · + x n y n ≥ x 1 z1 + x 2 z2 + · · · + x n z n ,
which follows from the Rearrangement inequality.

Problem 22 (IMO, 1978). Let a1, a2, . . . , an be distinct positive integers. Prove
that
a1 a2 an 1 1 1
12 + 22 + · · · + n2 ≥ 1+ 2+ · · · + n.

Solution. Let (c1, c2, . . . , cn) be the permutation of (a1, a2, . . . , an), such
that c1 ≤ c2 ≤ · · · ≤ cn. Then ci ≥ i for 1 ≤ i ≤ n. By the Rearrangement
inequality,
a1 a2 an cn
+ +···+ c1 c2 +··· +
≥ +
12 22 n2 12 22 n2
c1 2 n
≥ + +···+
112 122 1 n2

= + +··· + .
References 1 2 n
[1] Zdravko Cvetkovski (2012), Inequalities: Theorems, Techniques and Se-
lected problems. Springer-Verlag Berlin Heidelberg.
[2] Samin Riasat (2008), Basic of Olympiad Inequalities.
[3] https://www.qc.edu.hk/math/Resource/AL/RearrangementRearrangement Inequality.
Yue Kwok Choy.
[4] https://www.math.ualberta.ca/pi/issue2/page21-23.pdf The Rear- rangement.
Inequality by Dragos Hrimiuc.
SOME NEW COMBINATORIAL IDENTITIES

Dam Van Nhi


Hanoi University of Education

We introduce some classes of systems of linear equations can be solved by combinatorial


identities.

1 Some systems of linear equations and combinatorial


identities

Example 1. Assume that α1, α2, . . . , αn ∈ R and αi + j = 0 with i, j =


1, 2, . . . , n. Solve the following linear system:
 x
1
x xn
2 +xx2α1 + · · · + n α1 = 1
1 + α1 + 2 n
=1
 + + · · ·
1 + α2
1 +
... 2 + α2 n x+ α2

 x1 x2 xn
+ = 1.
1 + αn 2 + αn + · · · n + αn
x1 xn p (x )
Proof. Consider f (x) = x2 +···+ −1= ,
+ ∏
n
1+x 2+x n+x i=1 (i + x)
where p(x) is a polynomial of degree n. Since f (αi) = 0, therefore p(αi) =

0, i = 1, . . . , n. In view of this result, we get


(x − α1)(x − α2) . . . (x − α n )
f ( x) = − .
x1 (x + 1)(x + 2) . . . (x + n)
From x2 (x − α1) . . . ( x −
αn)
+ x 1 = we de-
+ ·· + −
n
1+x 2+x · n + x (x + 1) . . . (x + n)
duce

x1(x + 2)(x + 3) . . . (x + n) + x2(x + 1)(x + 3) . . . (x + n)


+ x3(x + 1)(x + 2)(x + 4) . . . (x + n) + x4(x + 1) . . . (x + n)
+ ···
+ xn(x + 1)(x + 2) . . . (x + n − 1) − (x + 1)(x + 2) . . . (x + n)
= −(x − α1)(x − α2) . . . (x − αn).
Upon direct computation, we get the solution of system of equations
n

 (− i=1 (1 + αi) by x = −1
n−1
1)

x1 =
( n − 1) !
( 1)n−2 ∏n (2 + αi)
x 2 = − by x = −2
i=1

 1!(n 2)!

n

( 1)n−3 ∏ (3 + αi)
above i=1
by x = −3

x 3 = 2!(n − 3)!

.
..
n
 (n −
(− ∏ (n + α i) by x = −n.
1)!
 xn i=1

=
Corollary
ties: 3. Set ϕ(x) =
∏ (x + αi). Then, there are the following identi-
i=1
i
n (−1)i(n)ϕ(i) = (−1)nn!.
(i) ∑
i=0 (−1)i(n)i in = (−1)nn!.
(ii) n
∑ i n n+i n
(iii) n (−1) ( )( ) = (−1) .
i=0
∑ i i
Proof.i=0
n(i) We invoke by Example 1 to deduce that:
(−1)n−i ϕ(i) (x − α1)(x − α2) . . . (x − αn)
∑ (x + i)(i − 1)!(n − i)!
−1=−
(x + 1)(x + 2) . . . (x + n)
.
i=1
By x
0 we get n (−1)n−i ϕ(i) (−1)n+1 ϕ(0) n
1 i n
−1 = or ∑( i
= ∑ i!(n − i)! n!
(−1)nn!. i=0
− ) ( )ϕ( ) =
i=1 i

(ii) Substituting α1 n
= · · · = αn = 0 in ∑ (−1)i(n)i ϕ(i) = (−1)nn! we get
i=0

n
i n n
i=0
(−1) ( )i = (−1) n!.
n

(iii) By
α i (−1)i(n)(n+i) = (−1)n.
n
i
= i, i = 0, 1, . . . , n we obtain ∑ i i
i=0
Example 2. Solve the following system of linear eqations:
∑ n sx k + yk 1
k=1
=
s2 + k2 s

s = ±1, . . . , ±n; xk, yk ∈ R, k = 1, . . . , n
n


n
∏ ( s2 + k 2 )
s= 1
(k2 2 2 .
∏ − s )(1 + k )
and evaluate the sum S = k=1
1 sƒ=k
Proof. Consider f (x) = − + n x k x + yk p (x )
∑ = , where p(x)
x k=1 x2 + k2 n
x ∏ ( x2 + k 2 )
is a polynomial of degree ™ 2n. Since f (s) = 0 therefore
k=1 p(s) = 0 when

s = ± 1, . . . ±n. We obtain f (x) = a(x2 − 12)(x2 − 22) . . . (x2 − n2)


, . Since
n 2 2
x ∏ (x + k )
k=1
1 n
xkx + yk
2 2 = a(x2 − 12)(x2 − 22) . . . (x2 −
− +∑ x +k n
we deduce the iden-
x
n 2)
x ∏ (x2 + k2)
k=1
tity k=1 2 2 2 2 2
2 2 2 2 2
− (x + 1 ) . . . (x + n ) + x(x1x + y1)(x + 2 )(x + 3 ) . . . (x + n )
+ x(x2x + y2)(x2 + 12)(x2 + 32) . . . (x2 + n2) + · · ·
+ x( xn x + yn)(x2 + 12)(x2 + 22) . . . (x2 + (n − 1)2)

= a(x2 − 12)(x2 − 22) . . . (x2 − n2)


 2 2
n ∏ (1 + k )

 a = −(−a1
(−) n
by
1 ) x
n
 − x1 + n
k=1 by x = i
iy1 = −
2
∏ (k 1) 2
 − k=2n n 2
and obtain the solution = 0− x2 + iy2 = a( 1) ∏ (2 + k ) by x = 2i

n k=1


... (k2 − 22)
 k=1,kƒ=2 n
∏ (n2 + k2)
− xn + a(−1)n k=1 by x = ni.
iyn = −
(k2 −
 n n∏1

k=1
∏ (r + k 2 )
2

Hence y1 = · · · = yn = 0 and xr = k= 1 with r = 1, . . . , n.


n

2 2 k=1,kƒ=r
(k2 − r2)
xkx − 2 − 2
2 2
Since x 2 + k2 = a(x 1 )( x 2 ) . . . − n ) we have the
1 n
− +∑ (x n 2 2
x k=1 x ∏ (x + k )
k=1
n (s2 + k2)

n xk
sum ∑ = 1 or n s=1 = 1 by replacing x = 1.
∑ n 2
∏ (k − s2)(1 + k2)
2
k=1 1 + k k=1 sƒ=k

Corollary 4. There is the following identity


n ∏
n

s=1 (s
2
+ k2)( 2n )
2 ∑ (−
n−k n−k = (2n)!.
2
k=0
1) k
2 2
n (s + k ) 1 + k2
n
∏ n
Proof. Sine ∑ n s=1− = 1 by Example 2 and ∏ (k2 − s 2) =
∏(k2 s2)(1 + k2)
sƒ=k
k=1
s

k
n
∏ n
(n − k ) ! ( n + k ) !
(k − s) ∏ (k + s) = ( −1)n−k we get
2k2
sƒ=
sƒ=

k k

n
2k2 ∏n (s2 + k2)
s =1

= 1.
k=1 (−1)n−k(n − k )!( n + k )!( 1 + k2)
(s2 + k2)( 2n )
n−k 2
(−1) k ∏ n
n s=1 n−k

By multiplicating (2n)! we obtain 2 k=1 1 + k2 = (2n)!.


Example 3. Solute the following system of linear equations:

∑n n yk 1
k=1 xk +∑
s− k=1 =
k s+k s

s = ±i, . . . , ±ni; xk, yk ∈ R
n

∏ (s 2 + k 2 )
and evaluate the sum S = n s= 1

k=1 k2 (1 ∏ (k2 − s 2 ) .
+ k2)

1 sƒ=k
yk p( x)
Proof. Consider f (x) = − + n xk +
n
=
∑ ,
x− ∑ x2
x k=1 k x+k n k2)
k=1
x ∏ (−
k=1
where p(x) is a polynomial of degree ™ 2n. Since f (s) = 0 therefore

p(s) = 0 when s = ±i, . . . , ±ni. It is easy to show that


2 2 2 2 2 2
f (x) = a(x + 1 )(x + 2 ) . . . (x + n ) .
n

x ∏ (x2 k2)
1 k=1
Since n xk 2 2 2 2 2 2
nyk = a(x + 1 )(x + 2 ) . . . (x + n ) there
∑ ∑ x+k
n
+ x ∏ (x2 k 2)
− + k=1 x − k k=1 −
x
k=1
is:

− (x2 − 12 ) . . . (x2 − n2 ) + x1x(x + 1)(x2 − 2 )(


2 x 2 − 3 2 ) . . . ( x2 − n2 )

+ x2x(x + 2)(x2 − 12)(x2 − 32) . . . (x2 − n2) + · · ·


+ xn x( x + n)(x2 − 12)(x2 − 22) . . . (x2 − (n − 1)2)
+ y1 x(x − 1)(x2 − 22)(x2 − 32) . . . (x2 − n2)
+ y2 x(x − 2)(x2 − 12)(x2 − 32) . . . (x2 − n2) + · · ·
+ y nx (x − n)(x2 − 12)(x2 − 22) . . . (x2 − (n − 1)2)

= a(x2 + 12)(x2 + 22) . . . (x2 + n2).


n
(x2 + k2) and ψ(x) = 2 2
n (x − k ). Then, we obtain xk
∏ and
Put ϕ(x) = ∏
yk as: k=1
k=1

a = −(−1)n by x = 0
n

a ∏ (12 + k2)
x1 = k =1 by x = 1
n
2.12 ∏ (1 − k 2 )
k=2
n

a ∏ (22 + k2)
x2 = k=1n
∏ by x = 2
2.22

... k=1,kƒ=2 (2
2− k2)
n

a ∏ ( n2 + k 2 )
k= 1
n∏1
xn = by x = n
− ( n2 −
2n2 k=1 2k

n
)a ∏ (12 + k2)
y1 = k= 1 by x = −1
n
2
2.12 ∏ (1 − k )
k=2
n

a ∏ (22 + k2)
y2 = k= 1
n by x = −2

2.22
... k=1,kƒ=2 (2
2− k2)
n

a ∏ (n 2 + k 2 )
k =1
n∏1
yn = 2n2 − (n2 − 2k by x = −n.
k=1

1 n ) yk
From = a(x2 + 12)(x2 + 22) . . . (x2 + n2)
− +∑ x x +
n k n and
x x −k k + ∑ x ∏ (x2 − k 2)
k=1 k=1 k=1
2 ∑
n
a(x + 1 )(x2 + 22) . . . (x2 + n2)
2 2
xk .
x2 − k2 =
n
xk = yk it follows −1 + 2x k=1 ∏ (x2 k2)

k=1
n
∏ ( s2 + k 2 )
By replacing x = i we obtain ∑ n s= 1 = 1.
k2 (1 + k2) ∏ (k2 − s2)
k=1

sƒ=k
Corollary 5. There is the following identity
n ∏
n

s=1 (s
2
+ k2)( 2n )
2 ∑ (− n−k = (2n)!.

1 + k2
k=1
n
n
Proof. Since ∏ (s2 + k 2 )
s=12 = 1, by the Example 3 and
2 2 2
∑ k ( 1 + k ) ∏ (k − s )
k=1
sƒ=k
n
∏ n n

( k 2 − 2s ) k
− (k s) ∏ −
sƒ= sƒ=k
sƒ=k
=
(sk) =
+ ( − k )!( n + k )!
we have
n 2k2
2 2
n 2 ∏ (s + k )
s =1

= 1.
k=1 (−1)n−k(n − k)!(n + k)!(1 + k2)


n
n (s2 + k2)( 2n )
We obtain 2 ∑ (− s=1 n−k = (2n)! by multiplicating with

(2n)!. k=1 1 + k2
Example 4. Solute the following system of lineat equations and evaluate the
following sum:
1
n sxk + yk

(i) 
k=1 s2 + k2 =
s(s2 + 12)(s2 + 22) . . . (s2 + n2)

s = ±1, . . . , ±n; xk, yk ∈ R.

n xk
(ii) T = ∑ .
k=1 n2 + k2
x k x + yk 1 p( x)
n − n =
Proof. Consider f (x) = ∑ x 2 + k2 n

k=1
x ∏ (x2 + k2) x ∏ ( x2 + k 2 )

k=1 k=1

with polynomial p(x) of dgree ™ 2n. Because f (s) = 0 therefore p(s) = 0


n
by replacing s = ±1, . . . , ±n. Hence f (x) = a(x2 − 12)(x2 − 22) . . . (x2 − n2)
.

x ∏ ( x2 + k 2 )

k=1
Since ∑

a(−

n
x k x + yk
− n
=
x 2 + k2

a(x2 − 12) . . . (x2 − n2)


we obtain
1


=
(n!)2
n

k=1 (22 + k2)
a(− 1
n
∏ + n

− x2 + iy2 = by x = 2i
2− 22) (k2 − 22)
... k=1,kƒ=2 (k
k=1,kƒ=2

n
(n 2 + k 2 )
a(−
− xn + iyn = k=1 1 by x = ni.
n−1 +
∏ (k2 − n2)
n−1
∏ ( k2 n 2)
k=1 k=1

n
∏ ( k2 + s 2)
We have yk = 0, xk = s=1 n n 1 with k =

∏ ∏
1, . . . ,
n
(n!)2 s=1,sƒ=k (s − s=1,sƒ=k (s −
2 k2) 2 k2)
n
and obtain the identity:
xkx 1
− = (−1)n−1(x2 − 12) . . . (x2 − n2)
∑ x2 + k2 n
x ∏ (x2 + k2)
n
∏ (x2 + k 2)
.
k=1 k=1 (n!)2 k=1

x
n
xk 1
n2 + k2 = .
By x = n there is ∑ n2 ∏n (n2 + k2)
k=1 k=1
Corollary 6. There is the following identity:
n
2 2 Σ (−1)k−12k2( 2n ) (2n)!
∑n Σ ∏ (k + s ) = n .
s =1 1 + k2 n+k
− ∏ (1 + k 2 )
k=0 (n!)2
k=0

Proof. By Example 4 we have ∑ n


xk 1
= n or the identity
k=1 ∏ (1 + k2)
1+ k2
k=1
n
n Σ ∏ ( k2 + s2 ) Σ
s =1 1 1 1
∑ ∏
n ∏
n 2
=
− 1+k n .
2− 2−
∏ (1 + k2)
k=1
(n!)2 s=1,sƒ=k (s k2) s=1,sƒ=k (s k2)
k=1
Since n
2 2 (−1)k−1 k! n k ! and T 1
s k n we
s
∏ ( − )= = n
∏ (1 + k2)
(
obtain =1,sƒ=k 2k2 − )( + )
k=1
n
n Σ 2 2 Σ
=1 ( k + s )
s∏ 1 1
T −
= ∑ (n!)2
n
∏ (s2 − k2)
n
∏ (s2 − k2) 1 + k2
k=1 s=1,sƒ=k s=1,sƒ=k

= Σ ∏n
( k2 + s2 ) −
n Σ k22k2
∑ (n!)2(s=n1 − k)!(n + k)! (n − k)!1(n + k)! (−11+
)k−1
k=1n Σ n
s=1(k

2
+ s2)( 2nn+k) (n+k2n) Σ (−1)k−12k2
= ∑ −
(n!)2(2n)! (2n)! .
k=1
n (k2 + s2)
1 + k2

Hence n Σ +k
Σ (−1)k−12k2(n2n
s =1 ) (2n)! .
∑ −1 = n
∏ (1 + k2)
k=1
(n!)2 1 + k2
k=1
References
[1]D. Faddeev et I. Sominski, Recueil D’Exercices D’Agebre Superieure,
Editions Mir-Moscou 1977.
[2]R. Merris, Combinatorics, PWS publishing company 20 Park Plaza,
Boston, MA 02116-4324.
[3]J. Rivaud, Exercices D’Algebre 1, Paris Librairie Vuibert 1964.
MENELAUS’S THEOREM AND ITS APPLICATIONS IN
SOME GRADE 8 PROBLEMS

Nguyen Van Nho


Danang city
We would like to start with a problem in [1].
Problem 1 ([1], Example 41, pp. 90). Let ABC be a triangle with AC > AB.
Arbitrary points D, E are taken on sides AB, AC, respectively, such that BD
= CE. Let K be the intersection
KE point of the two lines DE, BC.
Prove that the ratio does not depend on how to choose points D
KD
and E.
Vu Huu Binh [1] gave three methods for solving this problem.

Method 1. For making the ratio equal to KE : KD, we draw straight line DG
parallel to AC (G on BC). According to Thales’s theorem, we have

KE KE
KC
= = EC .
KD , KD DG
KG EC BD KE BD
=
Since BD = CE, replacing by , hence KD , which is
AB DG DG DG
equal to , this happens because DG ǁ AC. Consequently, KE : KD
AC
does not depend on how to choose points D and E.

Method 2. Drawing EH ǁ AB (H on BC), we have


KE EH EH
= AB
= = AC .
KD BD EC
Method 3. Drawing DM ǁ BC (M on AC), we have

= KE
ED EC = =
KE AB
MC MC
EM EC BD AC .
⇒ =
KD

In these three ways, we used Thales’s theorem and its consequence, after
drawing the appropriate parallel lines.
However, after drawing the shape according to the problem with- out having
to draw the necessary parallel lines, if we are familiar with Menelaus’s theorem
then the result will be seen immediately, with the attention that BD = CE. We
will continue to discuss this after introducing the Menelaus’s theorem.
In fact, this theorem is proved by Thales’s theorem and the relation of sides
of similar triangles, and in excellent student exams, you have the right to use it
without proof. Now, I would like to introduce this theorem.
2 Menelaus’s theorem
2.1 Observation
A transversal may meet two sides of a triangle and the third side which is
produced (Fig.4), or all three sides which are produced (Fig.5).

P, Q, R are called the traces of transversal PQR on the sides BC, CA, AB of
the triangle ABC. The trace (P) of the transversal on a side (BC) and the two
vertices (B, C) lying on that side determine two segments (PB, PC). The content
of Menelaus’s theorem gives the beautiful rela- tionship about the six segments
(AR, RB, BP, PC, CQ, QA).

2.2 Menelaus’s theorem


Theorem 14. Given triangle ABC. On straight lines containing the sides BC,
CA, AB, we take the corresponding points P, Q, R so that each point does not
coincide with vertices of the triangle, and there are no more than two points on
the sides of the triangle. Then, three points P, Q, R are collinear if and only

BR AQ PC
. . = 1. (1)
AR QC BP

Proof.
(a) Let P, Q, R are collinear (in other words, P, Q, R are the traces of
transversal PQR on the sides BC, CA, AB of the triangle ABC). Let the parallel
through A to the opposite side BC meet PQR in L. From the two pairs of similar
triangles ALQ and CPQ, BRP and ARL we have
AL AQ BR BP
CP.AQ BR
= , = . = 1.
CP ⇔ AL = CQ AR AL

AL AR
BP
CQ
Replacing AL in the first equation into the second equation is what
must be proved.
* Remark. In the second case of the transversal PQR, you can rely on Figure
7 to proceed similarly.

(b) Reversely, suppose (1) occurs. Call Q’ as the intersection point of RP and
side AC. Then, according to the above, we also have:

B R AQJ PC
. . = 1. (2)
AR Q C BP
J

AQ AQ
From (1) and (2) we deduce j
QJ C = , that means Q ≡ Q’, which
QC
proves the reverse proposition.

2.3 Remark
There are several different proofs of Menelaus’ theorem. In what fol- lows,
we give briefly the second proof for (a) by using ratio of two trian- gles.
Denoted the area of the triangle XYZ by [XYZ], we have
BR [ PBR ] AQ [PAR] PC [ PCR ]
= , = [PCR] and = [PBR] ,
AR ] QC BP
BR [AQPARPC
hence . . = 1.
AR QC BP

2.4 Solving Problem 1 by using Menelaus’s theorem

Going back to Problem 1, we see that, the transversal BCK of the tri- angle
ADE meet all three sides which are produced (Fig.9, like Fig.5). Therefore, we
have
DB AC = 1, and the conclusion is inferred immediately, since
EK . .
AB CE KD
DB = CE.

3 Applications of Menelaus’s theorem in some grade-8-


problems
Problem 2 (Hanoi Open Mathematical Competition 2016, Junior Section,
Q.11). Let be given a triangle ABC, and let I be the middle point of BC. The
straight line d passing I intersects AB, AC at M, N, respectively. The
straight line d0 (= ƒd) passing I intersects AB, AC at Q, P, respectively.
Suppose M, P are on the same side of BC and MP, NQ intersect BC at E
and F, respectively. Prove that IE = IF.

Hints. Applying Menelaus’s theorem to the cases: transversals EMP, FNQ,


MIN, QIP of the same triangle ABC (Fig.10), we get the following
equations in turn:
EB PC = 1, (3)
MA. .
EC PA MB
FC QB NA = 1, (4)
. .
FB QA NC
MB NA IC = 1,
. . (5)
MA NC IB
PA IC QB = 1. (6)
. .
PC IB QA
EB FC
Using (3), (4), (5), (6) and the fact IB = IC, you can deduce = ,
then IE = IF. EC FB

Problem 3 (Exam for excellent students of grade 8 at Nguyen Khuyen school,


Danang City, 2007). The diagonals MP and NQ of a quadrilat- eral MNPQ meet
at O in such a way that MO = OP and QO = 2ON. Suppose that H and K are
points on OP and NP respectively such that
MP NK
=
OH = 3. Show that the points Q, H and K are collinear.
Solution.KP
From the assumption QO = 2ON we have (Fig.11)
= 2
QN . (7)
OQ 3
1 MP
Since MO = OP (PO = MP) and
= 3, hence
2 OH
1 MP 1

H
PH PO −HHO = 2. H − 1 = 2. (8)
O = O O
Now, consider triangleNK ONP and the points Q, H and K. By (7), (8)
and using the assumption = 3, we obtain
KP
NK
. PH 12
KP OQ .HO = 3. 2 . 3 = 1.
QN
Therefore, by the converse of Menelaus’ theorem, Q, H and K are
collinear.

Problem 4 (From [2], Problem 1.12, pp. 28). Let ABC be a triangle and three
points A1, B1, C1, respectively be on the three sides BC, CA, AB such that the
lines AA1 , BB1, CC1 intersect at O. Suppose that three pairs of lines AB and
A1B1, BC and B1C1, CA and C1 A1 alternately intersect at three points C2, A2,
and B2. Prove that C2, A2, and B2 are collinear.

Solution. Applying Menelaus’s Theorem to triangles and points:


* OAB and A1, B1, C2, we have
AA1 OB1 BC2
. . = 1. (9)
OA1 BB1 AC2
* OBC and B1, C1, A2, we have
OC1 BB1 = 1. (10)
CA 2 . .
CC1 OB1 BA2
* OAC and A1, C1, B2, we have
OA1 CC1 AB2
. . = 1. (11)
AA1 OC1 CB2
By multiplying the relations (9), (10), and (11) side by side, we obtain
BC2 CA 2 = 1.
AB2 . .
AC2 BA2 CB2
Hence, by the converse of Menelaus’ theorem, C2, A2, and B2 are collinear.

Problem 5 (Problem 4, from the Entrance exam for Vnh Phc specialized
high school ). Let ABC be an acute triangle, AB < AC. Let D, E, F be the
foot of the height of the triangle dropping from A, B, C, respectively. Call P
the intersection point of the lines BC and EF. Straight line through D which
is parallel to EF
PBcuts straight lines AB, AC, CF at Q, R, S, re- spectively.
D
Prove that B and D is the midpoint of QS.
PC =
DC
Note. That is the content of sentence b) in the problem. I ignore the
two sentences a) and c) because the content is in grade 9 knowledge, which is
not appropriate here.

Solution. Since AB < AC, Q is on the opposite ray of ray BA and R is


inside of line segment CA, so, Q, C are on the same side to the straight line
BR (Fig.13).

From pairs of similar triangles DHB and EHA, DHC and FHA we
DB HB DC HC
have
= , = .
AE HA HA AE HB AE
Hence DB
. = . FB
AF = AF HC AF . (12)
DC EC
Applying Menelaus’s theorem to the triangle ABC with the transver-
sal PEF, we obtain

PB CE PB AE FB
AF . . =1⇔ = . . (13)
(12) and (13) give PC EA FB PC AF EC

PB DB
=
PC . (14)
BD DC CD
DQ
We have QR ǁ EF, so: P
DS = . Combining these with
C
F = , P
(14) we obtain DQ = DS, that means D isPthe midpoint of QS.
B
Problem 6 (6th 30-4 Mathematical P Olympiad,
F April 2000, Vietnam). Let

the triangle ABC have an area S0 = 1. Points M, N, P are on sides


MB NC PA
BC, CA, AB, respectively, such that
MC = k1 , = k2, = k3,
NA

PB
(k1 , k2 , k3 < 1).
Calculate the area of the triangle which is created by three intersec-
tion points of the three line segments AM, BN and CP.

Solution. Let EIF be the triangle which is created by three intersection points
of the three line segments AM, BN and CP (Fig.14), we have
C
N k2
[BCN] = k2 + 1 .
S0
=
CA

k2
Having S0 = 1, so [ BCN ] = .
k2 + 1
[ BCF] BF BF k2
We also have = , hence [BCF ] = .
[ BCN ] BN BN k2 + 1
(Denoted the area of the triangle XYZ by [XYZ]). Applying Menelaus’s
theorem to the triangle ABN with the transversal PCF, we obtain
FB CN
PA . . = 1, hence
FN CA PB
1 + k2 1 + k2
= k2k3 =
FB ⇒ BF 1 + k2 + k2k3 ,
F
N BN
therefore k2 k2
. =
1 + k2
[BCF]= k2 + 1 1 + k2 + k2k3 .
1 + k2 + k2k3 k3 k1
Similarly, we have [ ACI ] = , [ ABE ] = .
Therefore, 1 + k3 + k1k3 1 + k1 + k1k2
k2
[ EIF ] = 1 .− k3 k1 Σ
+ + .
1 + k2 + k2k3 1 + k3 + k1k3 1 + k1 + k1k2
Finally, we give some exercises for good students of grade 8 to solve
themselves. Hopefully, through this article, you will master how to ap- ply
Menelaus theorem to solve appropriate problems.
Problem 7. Let ABC be a triangle. On the opposite ray of ray CB, we get the
point M such that BM : MC = 3. On the line segment CA we take the point N so
that NA : CN = 3. Let P be the intersection point of MN and AB. Prove that P is
the midpoint of AB.

Problem 8. Let MNP be a triangle with angle N = 90◦, NP = 3cm and MN =


4cm. S is a point on the segment MP such that MS = 1cm, and T is the midpoint
of MN. The line ST meets the line NP at H. Find the length of the segment NH.
Problem 9. Prove that the three external angle bisectors of a triangle intersect
their opposite sides at three collinear points.

References
[1]Vu Huu Binh, Some development topics of Geometry 8, Vietnam Educa- tion
Publishing House, 2000, Vietnamese.
[2]Nguyen Van Nho, The theorems in Plane Geometry through Olympic
exams, Vietnam Education Publishing House, 2004, Vietnamese.

Nguyen Van Nho Danang,


March, 2019
PROOFS AND GENERALIZATIONS OF ONE INEQUALITY
AT THE 36 IMO
TH

Vu Tien Viet
Hanoi Mathematical Society

We introduce some solutions and generalizations of one International Olympiad


problem (36th IMO).

1 Second problem of IMO 1995


It is known that the 2nd problem given at the 36th IMO held at Toronto
(Canada, 1995) was formulated as follows:

Problem 1. Let a, b, c be positive real numbers with abc = 1. Prove that


1 + 3 1 + 3 1 ≥ 3
3
a ( b + c ) b ( c + a) c ( a + b )
· (1)
2 Proofs 2

Proof 1. It follows from the condition abc = 1 that


1 = b2c2 , 3 1 = c2 a2 , 3 1 = a2b2
3
a ( b + c) b (c + a) c (a + b )
·
a ( b + c) b(c + a) c ( a + b)

Noe, the inequality (1) is equivalent to


b2c2
ca22 a2b2 3
+ + ≥ ·
a(b + c) b(c + a) c(a + b) 2
By Cauchy-Schwarz inequality
1 2 3 1 2 3
(a2 + a2 + a2)(b2 + b2 + b2) ≥ (a1b1 + a2b2 + a3b3)2,
we have 2 2
Σ bc c2 a2 a2b2
+ +
a(b + c) b(c +2 a) c(a + b)
[a(b + c) + b(c + a) + c(a + b)] Σ

≥ (ab + bc + ca) ,
or
b2c2 c2 a2b2 1
+ a +
2 ≥
a ( b + c ) b ( c + a) c ( a + b )
By AM-GM inequality, we have
(ab + bc + ca).
2 3
b2c2 + c2 a2 + a2b2≥ 1 3 √3 2 2 2

a ( b + c) b ( c + a ) c ( a + b) abc
(ab + bc + ca) ≥ = ·
2 2 2
Proof 2. It follows from the condition abc = 1 that 2 2
1 b2c2 , 1 c2 a2 , 1 ab
= = =
a3(b + c) b3(c + a) c3(a + b) ·
a ( b + c) b(c + a) c ( a + b)
Noe, the inequality (1) is equivalent to
b2c2 ca22 a2b2 3

+ + ≥ ·
Let a(b + c) b(c + a) c(a + b) 2
2 2 +
bc c2 a2 + a2b2= b2c2 + c2 a2 + a2b=
2 S.
a ( b + c) b ( c + a ) c ( a + b) ab + ac bc + ba ca + cb
By Cauchy-Schwarz inequality
(a2 + a2 + a2)(b2 + b2 + b2) ≥ (a1b1 + a2b2 + a3b3)2,

we have 1 2 3 1 2 3

[(ab + ac) + (bc + ba) + (ca + cb)]S ≥ (ab + bc + ca)2

= (ab + bc + ca)(ab + bc + ca).


By AM-GM inequality, we have

2(ab + bc + ca)S ≥ (ab + bc + ca)(ab + bc + ca)



≥ ( ab + bc + ca)3 3 a2 b2 c2 = 3( ab + bc + ca).

Hence S ≥
3 The desired conclusion follows.

1 1
Proof 3. Note that for α > 0 we have α + ≥ 2, or α ≥ 2 − . We thus
α α
have
1 1 2 1. a 2 ( b + c) 1 ab + bc

Σ
=
a3(b + c)
Similarly · ≥ 2−
2a a2(b + c) 2a 2 = − ·
a 4
1 1
1 ca + cb
≥ 1 − bc + ca
b 3 ( c + a) b 4 , c 3 ( a + b) ≥ ·
Adding the above inequalities yields
1 1 1 1 1 1 1
+ + ≥
a 3( b + c) b 3 ( c + a) c 3 ( a + b)
+ + − (ab + bc + ca)
a b c 2
=1
(ab + bc + ca).
Finaly, the AM-GM inequality leads us to the2
1 1 1

1 3 3
≥ ( ab + bc + ca) ≥ a 2b 2c 2 = ·
a3 (b + c) + b3 (c + a) + c3(a + b) 2 2 2


3

Proof 4. Inequality (1) is equivalent to


b2c2
ca22 a2b2 3
+ + ≥ ·
a(b + c) b(c + a) c(a + b) 2
On the other hand, we have for λ > 0,
b2c2 + λb(c +
+ λa(b + c) ≥ 2 ab)(c≥+ 2a)
a(b2+ 2c)
c a

λbc, √
√ λca,
a2b2 λab.
+ λc(a + b) ≥ 2
c ( a + b)
Adding the above inequalities yields
b2c2 c2 a2 b2 √
+ a 2 + ≥ (2
a ( b + c) b ( c + a ) c ( a + b)
By AM-GM inequality, we have
λ − 2λ)(ab + bc + ca).
b2c2 + c2 a2 + a2b≥2 (2 √
a ( b + c) b ( c + a ) c ( a + b) λ − 2λ)(ab + bc + ca)

1
Choosing λ = gives √ √
≥ 6( λ − a2b2c2 = 6( λ − λ).
4


λ) 3

b2c2 c2 a2b2 3
+ a +
2 ≥ ·
a(b + c) b(c + a) c(a + b) 2
Proof 5. Lemma 1. Let α, β be real numbers and x, y be positive real num-
bers.
Then, we have
α2 β2 (α + β)2
+ ≥ · (*)
x y x+y
We see that, inequality (∗) equivalent to
α2y(x + y) + β 2x(x + y) ≥ (α + β)2xy ⇔ α2y2 + β2x2
2
≥ 2αβxy ⇔ (αy − βx) ≥ 0.
Let α, β, γ be real numbers and x, y, z be positive real numbers.

Then, by inequality (∗) we have


2 2 2 (α + β)2 + γ2≥ (α + β + γ)2
α β γ
x+y z
+ + ≥ · (**)
x y z x+y+z
Using the inequality (∗∗) and AM-GM inequality, we have
1 1 1 b2c2 c2 a2 a2b2
a3(b + c)
+ (+abc3+(abc++b)ca=)2a(b= + 1c) + b(c + a) + c(a + b) ≥
b3(c +≥a) 2(ab + bc + ca) 3 2 2 2 3
√3 a b c = ·
2
(ab + bc + ca) ≥ 2
2
Cch 6. Lemma 2. Let real numbers a1 a1 and b1 b 2.

Then, have (aa1
wenumbers
Let real a2)(b1 a and
b2) b 0, or a1b1 + a2b2 a1 b 2 + a 2 b 1 .
1 − a2 −3 ≥1 b2 b3.
Then, we have (a2 a3)(b1 ≥b3) 0, or a2b1 + a3b3 a2 b 3 + a 3 b 1 ,
−≥ ≥− ≥ ≥
≥by above a1b1 + a2b2 a 1b2 + a 2b 1 ,
≥ ≥
lead to a1b1 + a2b2 + a3b3 a 1b2 + a 2b3 + a 3 b 1.

On other hand (a1 a3)(b2 b 3) 0, or a1b2 + a3b3 a1 b 3 + a 3 b 2 ,
− − ≥
by above a1b1 + a2b2 a 1b2 + a 2b 1 ,
≥ ≥
lead to a1b1 + a2b2 + a3b3 a 1b3 + a 2b1 + a 3 b 2.
Put x = bc > 0, y = ca > 0, z = ab > 0, since abc = 1,

a2b2c2 = 1, xyz = 1. The inequality (1) equivalent to


1 1 1 b2c2 ca22 a2b2
+ + = + +
a3(b + c) b3(c + a) c3(a + b) a(b + c) b(c + a) c(a + b)
= x2 + y2+ 2
z≥ · 3
y+z z+x x+y 2
By symmetry, we may assumex that x ≥yy ≥ z. Then
z
.

y+z ≥
z+x x+y
Using the above lemma 2 gives
x2 + y2 + z2≥ x · y z x
+y· +z· ,
y+z z+x x+y z+x x+y y+z
x 2 +
y 2+
z 2≥ x · z
+y· x
+z· ·y
y+z z+x x+y x+y y+z z+x
Adding the above inequalities yields
x2 + y2 z2 1 3√ 3
y + z z + x + x + y ≥ (x + y + z) ≥ 3 xyz = ·
Proof 7. Put x = bc > 0, y = ca > 0, 2z = ab > 0, since2 abc = 21,
a2b2c2 = 1, xyz = 1. The inequality (1) equivalent to
1 1 1 b2c2 ca22 a2b2
+ + = + +
a3(b + c) b3(c + a) c3(a + b) a(b + c) b(c + a) c(a + b)
= x2 + y2+ z≥
2 · 3
y+z z+x x+y 2
By symmetry, we may assume that x 1≥ y ≥ z. Then
x2 y2 z2, 1 1
≥ ≥ .
≥ z+x x+y
y+z
Using the above lemma 2 gives
x2 + y2 + z2≥ x+2 +y
2
, z
2

y+z z+x x+y z+x x+y y+z


x2 + y2 + z2≥ x+2 +y2 · z2
y+z z+x x+y x+y y+z z+x
Adding the above inequalities yields
x2 y2
z2 1 . x2 + y2 + z2 zz2 ++ xx2 Σ .
+ + ≥ + +
y+z z + x x + yy2 2 x+y y+z
α +β
Clearly, we have 2 2 α+β
≥ for α > 0, β 0. Then
α+β 2 >
2 2
x +y y2 + z2 z2 + x 2 √
+ +
x+y y+z +
≥ x + y + z ≥ 3 3 xyz = 3.
z x
Proof 8. Since abc = 1, a2b2c2 = 1. The inequality (1) equivalent to
b2c2 + c2 a2 + a2b2= b2c2 + c2 a2 + a2b≥
2 · 3
a ( b + c) b ( c + a ) c ( a + b) ab + ac bc + ba ca + cb 2
Consider the function bc ca
√ √
f (x) = . √ ab + ac x − ab + acΣ2 + . bc + ba x − bc + baΣ2


ab √
+ .√ x − ca + cbΣ2 =
ca + cb
. b2c 2 c2 a 2 + a2b 2 Σ 2
+
= ab + ac bc + ba ca + cb x
− 2(ab + bc + ca)x + 2(ab + bc + ca).
Clearly f ( x ) ≥ 0, ∀ x ∈ R. From there, we have ∆J ≤ 0, or
2 . b2c 2 c2 a 2 + a 2b2 Σ
+
(ab + bc + ca) − ab + ac bc + ba ca + cb
(ab + bc + ca) ≤ 0.
2
HOMC 2019 - Scientific seminar, Hanoi, April 3, 2019

Inferred
3
b2c2 c2 a2b2 1 3 √3 2 2 2
a
ab +
a2 ac (ab b= ·
bc ++ 2 c 2
ba + bc +
ca +≥
cb ca) ≥

2
Proof 9. Since abc = 1, a2b2c2
= 1. The inequality (1)
equivalent to
b2c2 a2b2
c +
a(b + c) +
b(c + a)
c≥(a +·b)
2
On space Oxyz consider
vectors
O˙ A ( . a ( b + c ) ,

.b ( c + a), .c( a +

b)),
b ca
, , . O˙B ( √
a(b √ a)
+
c)

b(

84
HOMC 2019 - Scientific seminar, Hanoi, April 3,
2019
√abc a ) ab + a
( b)+ | ·| 32 ac (ab b= ·
√ bc ++ 2 c 2
We have O O˙ B | ba + bc +
3 ca +≥
O˙ A · ˙ cb ca) ≥
=
O˙B = ab 1
.
B c a 2 1 1 1
2 2
+ bc + ca b2c2 Proof −1Put −1+
−1 10. + =
| b a + b + c = t > 0, we
and 2 have
+ a
O ab + ac +
· Σ b a− b− c
˙ I bc + ba −
= ca + cb 2
n c
A f
c e [2 1
r 1
o + +
· r a3(b + c) + b−1 + + c−1−1 c−1
s e (a b (c + a) = +−1
3 a−1 a +
3
c ( a + b) b
d
O a−2 b−2 c−2
˙ A =
b
^
B
a
O +
B −
=
bc +

| ≤
+ c
O
˙ | Consid x2
er the ( = , ( < x<
O ca t t
A functio
˙ n f
| −0
)] 2t2 x
A
| x
· 2 )
b ca2b2 1
85
HOMC 2019 - Scientific seminar, Hanoi, April 3, 2019

Inferred

). he (

W a J x=

e vf J )

>
(tot 0,
− lead3
f (xx))is
convex
function.

86
Using Jensen’s inequality, we have
1 1 1 1
a−1 + b−1 + c−1
−1
f (a−1) + f (b ) + f (c−1 ) ≥ 3 f ( )= ( + + )
3 2 a b c
≥ .3 1
3
abc
So we have to prove the inequality.2
= ·
2
Proof 11. Put x = bc > 0, y = ca > 0, z = ab > 0, s = ab + bc + ca =
x + y + z.

We have xyz = 1 and inequality (1) equivalent to


1 1 1 x2 y2 = z2
+ + = + +
a3(b + c) b3(c + a) c3(a + b) y + z z + x x + y
x2 y2 z2 3

Clearly
= + + ≥ ·
s− x s− y s− z 2
y+z s− z+x
s− < 1, 0 < =
0<
x < 1,
= y 2(x + y + z)
2s 2(x + y + z)
x+y 2s
s− = s− x s− y s − z = 1.
z0 <
< 1, + +
2s 2(x + y + z) 2s 2s 2s
Consider random variable X with
x s− x y s− y z s− z
P(X = )= , P(X = )= , P(X = )= .
s− x 2s s− y 2s s− z 2s
The, we have
x s− x y s− y z s− z 1
EX = s − x · 2s + s − y · 2s + s − z · 2s = 2,
. x Σ2 s − x . y Σ2 s − y . z Σ2 s − z
2
E(X ) = · + · 2s + · 2s =
s− x s− s−
y 1 . x2 z z
2 Σ
2s = ,
2 s− x+s−
y2 +
y s−
s
) = + + −
)−
s − xz s− y s− z · 4
0 Var X 1 . x2 y2 z2 Σ 1
≤ ( ) = E( X2 X
2 2s
x2 y2 z2 x+y+z 3√ 3
s

+ + ≥ =
s− x s− y s− z 2 3
xyz = ·
3 Generalizations 2 2 2

Problem 1. Let a, b, c be positive real numbers with abc = 1 and let


m ≥ 3.
Prove that
1 + 1 + 1 ≥ 3
a m ( b + c) b m ( c + a ) c m ( a + b)
· (2)
1 1 1 2
Proof. Put x = , y = , z = . We have xyz = 1 and inequality (2)
a b c
equivalent to
1 + 1 + 1= xm−1 + ym−1 ≥ zm−1
+ · 3
am(b + c) bm(c + a) cm(a + b) y + z z + x x + y 2
By symmetry, we may assume that x ≥ y ≥ z. Then

1 1 1 xm−2 ym−2 zm−2


xm−2
≥ ≥ ≥ ≥ , ≥ ≥ ·
y+z z+x x+y y+z z+x x+y
ym−2 zm−2,
Using the above lemma 2 gives
xm−1 m−1 m−1
+ y + z ≥ x· ym−2
+y· zm−2
+z· x,m−2
y+z z+x x+y z+x x+y y+z
x m−1 +
y m−1+
z ≥ x·
m−1
z +y·
m−2
x+ z ·
m−2
y· m−2
y+z z+x x+y x+y y+z z+x
Adding the above inequalities yields
xm−1 + ym−1+ zm−1 ≥ 1 m−2
y+z z+x x+y 2 (x + ym−2 + zm−2)
≥ .3
3
2 32
xm−2ym−2zm−2 = ·
The inequality (2) is proved. In special case when m = 3, the inequality
(2) would to the inequality (1).
Problem 2. Let a1, a2, ..., an (n ≥ 3) be positive real numbers with

a1a2...an = 1 and let m ≥ 3. Prove that

∑ 1 n(n − 1)
. Σ .a
m−1 ∑ Σ≥ (3)
1≤k<l≤n ∏ i
aiaj
1≤i≤n,iƒ=k, 1≤i<j≤n,iƒ=k, · (n + 1)(n − 2)

l l
Hint. Applying the generalized Radon’s inequality.
a special case when n = 3, a = a, a2 = b, a3 = c and m = 3 the inequality
(3) In
would to the inequality (1). 1
HOMC 2019 - Scientific seminar, Hanoi, April 3, 2019

SOME INTERNATIONAL OLYMPIAD PROBLEMS


IN GEOMETRY
Nguyen Dang Phat Hanoi
University of Education

We introduce some known International Olympiad problems in geometry with short


comments.

1 Some remarks
Problem 1. In the plane we are given two circles intersecting at X and
Y. Prove that there exist four points with the following property :
For every circle touching the two given circles at A and B, and meet- ing the
line XY at C and D, each of the lines AC, AD, BC, BD passes through one of
those four points.
COMMENT. The problem was received in the following formulation Suppose
m is the radical axis of the given circles Γ1 and Γ2 in the
internally
ν is the setorconsisting
both externally and intersecting m.Γν =and
Λ touching ∅Γ : Suppose Λ ν, plane and
of all circles 1 2 both touching Γi in Ai
and intersecting m in Bi, i 1, 2 . ƒ
Prove, there exists a set W of four points in∈the { plane, such that for every
Λ ν every∈line AiBj , i, j 1, 2 is incident
∈ { with at least one point in W.
}
Problem 2. Two circles Γ1 and Γ2 intersect at M and N. Let AB be the line
tangent to these circles at A and B, respectively, so that M lies closer to AB than
N. Let CD be the line parallel to AB and passing through M, with C on circle Γ1
and D on Γ2. Lines AC and BD meet at E; lines AN

and CD meet at P; lines BN and CD meet at Q. Show that EP = EQ.


COMMENT. The proposer has also suggested an alternative version of the
problem: Under the same assumptions, prove that NE is the bisector of the angle
CND.

88
Problem 3. Let O be the circumcentre and H the orthocentre of an acute
triangle ABC. Show that there exist points D, E, F on sides BC, CA, AB
respectively, such that OD + DH = OE + EH = OF + FH and the lines
AD, BE, CF are concurrent.
sition
(by theofproposer).
P for which is a+ variable
If POP PH is a point
minimum is given
on BC, po-= D. COMMENT
by P
then the

Problem 4.n is
A1 A2 . . . A Letcyclic
A1 Aif2 .and
. . Aonly
n beifato
convex polygon,
each vertex n can ≥
Aj one 4. aProve
assign pair that

(bj; cj) of real numbers, j = 1, 2, . . . , n, so that

Ai Aj = bjci − bicj for all i, j with 1 ≤ i < j ≤ n. (*)


COMMENT. The numbers bj , c j are not uniquely determined. Here is one
more proof of the necessity. Let A1, A2, . . . , An be arranged counterclock-
wisely around a circle Γ with centre O and radius R. Choose a system of
rectangular coordinates with origin at O so that the positive x-axis in- tersects Γ
at a point U between An and A1. Let 2αj be the measure of the
oriented angle UOA j (j = 1, 2, . . . , n), that is : the angle through which
incides with Ain j . For any pair of indices i, j with i < j, we have Ai A j = U has
to be rotated counterclockwise direction around O so that it co- angle.
Note
1 that ∠ A i OA j is equal to 2(αj − αi), if 0 < 2(αj − αi) ≤ 180◦, 2R sin
∠A
and toi OA − 2(α∠
360j ,◦ where A i OA j is the
◦ respective central (non-oriented)

j − αi), if 180 < 2(αj − αi) < 360 . In both cases,

2Ai Aj = 2R sin(αj − αi) = 2R sin αj cos αi − 2R cos αj sin αi.


So we can define bj = u sin αj, cj = v cos αj for j = 1, 2, . . . , n, where u

and v are arbitrary numbers such that uv = 2R.


Problem 5. The tangents at B and A to the circumcircle of an acute-
angled triangle ABC meet the tangent at C at T and U respectively. AT meets
BC at P, and Q is the midpoint of AP; BU meets CA at R, and S is the midpoint
of BR.
Prove that ∠ ABQ = ∠BAS. Determine, in terms of ratios of side-
lengths, the triangles for which this angle is a maximum.
COMMENT (by the proposer). An alternative method for finding ymax is
: From (1), we obtain 9(1 − sin2 C) = (4 − y sin C)2, sin2 C(y2 + 9) −
8y sin C + 7 = 0. Since this √equation in sin C has real roots,
6√4y2 ≥ 4.7(y2 + 9), so y2 ≥ 7, y ≥ 7 as before; equality when sin C =
7 3

, cos C = .
C4OMMENT. 4 The original formulation of the problem statement did not
assume that triangle ABC was acute, and required that the size of the maximum
angle be computed.
CD, and let X be a point inside◦ ABCD such that ∠ ADX = ∠BCX < 90◦
and ∠ DAX = ∠CBX < 90 . If Y is the point of intersection of the
Problem 6. Let ABCD be a convex quadrilateral with AB not parallel to
perpendicular bisectors of AB and CD, prove that ∠AYB = 2 ∠ADX.
COMMENT. The proposer has also supplied one more proof of the fact that
the point W coincides with Y, using inversion.
COMMENT. In its original formulation, the problem statement allowed X to be
an arbitrary point of the plane, not necessarily inside ABCD, and did not
require that the angles ADX, BCX, DAX, CBX be acute. Without these
assumptions the conclusion of the problem requires considering oriented
angles, possibly exceeding 360◦ .
Problem
= DE, EF 7. Prove thatbe a convex hexagon such that AB = BC, CD
Let ABCDEF
= FA.
BC DE FA 3
+ + ≥ .
BE DA FC 2
When does the equality occur?
COMMENT. The problem was proposed for a cyclic hexagon with the use of
Ptolemy’s equality.

Problem 8. Four different points A, B, C, D are chosen on a circle Γ so that


the triangle BCD is not right-angled. Prove that:
a) The perpendicular bisectors of AB and AC meet the line AD at certain points
W and V, respectively, and that the lines CV and BW meet at a
certain point T.
b) The length of one of the line segments AD, BT and CT is the sum of the
lengths of the other two.
COMMENT. The problem was slightly changed and a geometric solution to it
found. The author’s solution makes use of trigonometry.

Original formulation. In triangle ABC the angle at A is the smallest. A line


through A meets the circumcircle again at the point U lying on the arc BC
opposite to A. The perpendicular bisectors of CA and AB meet AU at V and W,
respectively and the lines CV, BW meet at T. Show that

AU = TB + TC.
Problem 9. Let AH1, BH1, CH3 be the altitudes of an acute-angled trian-
gle ABC. Its incircle touches the sides BC, CA, AB at T1, T2, T3, respec- tively.
Consider the symmetric images of the lines H1 H2, H2 H3, H1 H3 with respect to
the lines T1T2, T2T3, T1T3. Prove that these images form a triangle whose vertices
lie on the incircle of the triangle ABC.
COMMENT. The last proof is motivated by a simple observation. It is easy
to compute the angles between AB and the lines H1 H2 , T1 T2 , which are 2 α
β and α | β , respectively. It follows that the mirror image of H1 H2 across
| − | −
T1 T2 is |parallel to AB. A similar conclusion holds for the other two mirror
images, so the triangle . formed by the three reflec- tions has sides parallel
to those of ABC. Hence there is a homothety O h taking ABC to . Now,
the claim is that O incircle of
. is inscribed in the ABC. This can be true
only if O h takes theOcircumcircle of ABC to its incircle, which suggests
defining
O h as in theO solution above.
Problem 10. Let ABCDEF be a convex hexagon such that

∠B + ∠D + ∠F = 360◦ and
AB CD EF

· · = 1.
Prove that BC DE FA
BC AE FD
·
CA
· = 1.
EF DB
COMMENT. Considering the arguments of the complex numbers on both sides
of the equality

b− c a− e f− d
a − c · f − e ·b − d = 1,

it shows that ∠ BDF = ∠ AEF + ∠ACB.


2 Other problems
Problem 11. Let A1 be the center of the square inscribed in acute tri- angle
ABC with two vertices of the square on side BC. Thus one of the two
remaining vertices of the square is on side AB and the other is on AC. Points
B1, C1 are defined in a similar way for inscribed squares with two vertices on
sides AC and AB, respective1y. Prove that lines AA1, BB1, CC1 are concurrent.
Problem
AP, 12. In acute triangle ABC with circumcenter O and altitude

∠C “ ∠B + 30◦. Prove that ∠ A + ∠COP < 90◦.


proof,
BP.BG the positionis of
+ CP.CG the point and
a minimum, P inexpress
the plane
this of ABC such
minimum thatin AP.AG
value +
Problem
13. Let ABC be a triangle with centroid G. Determine, with
terms of the side lengths of ABC.
Problem 14. Let M be a point in the interior of triangle ABC. Let AJ lie on
BC with MAJ perpendicular to BC. Define BJ on CA and CJ on AB
similarly. Define
MAJ .MBJ .MCJ
p( M) = MA.MB.MC .
Determine, with proof, the
µ ( ABC ) denote this maximumlocation
value.of that p(MABC
such triangles
ForMwhich ) is maximal.
is the valueLet
of
µ ( ABC ) maximal?
Problem 15. Let ABC be an acute triangle. Let DAC, EAB, and FBC

be isosceles triangles exterior to ABC, with DA = DC, EA = EB and


FB = FC, such that

∠ADC = 2∠BAC, ∠BEA = 2∠ABC, ∠CFB = 2∠ACB.


Let D be the intersection of lines DB and EF, let EJ be the intersection
J

of EC and DF, and let FJ be the intersection of FA and DE. Find, with
proof, the value of the sum.
DB EC FA
+ + .
DDJ EEJ FFJ
Problem 16. Let ABC be a triangle and P an exterior point in the plane
of the triangle. Suppose AP, BP, CP meet the sides BC, CA, AB (or ex-
tensions thereof) in D, E, F, respectively.Suppose further that the areas of
triangles PBD, PCE, PAF are all equal. Prove that each of these areas is equal
to the area of triangle ABC itself.

Problem 17. Let O be an interior point of acute triangle ABC. Let A1 lie on
BC with OA1 perpendicular to BC. Define B1 on CA and C1 on AB similarly.
Prove that O is the circumcenter of ABC if and only if the perimeter of
A1B1C1 is not less than any one of the perimeters of AB1C1, BC1 A1, and CA1 B1 .
Problem 18. Let ABC be a triangle with ∠ BAC = 60◦. Let AP bisect

AQBAC andwhat
+ QB, let BQ
are bisect ∠ABC,
the angles of thewith P on ?BC and Q on AC. If AB + BP =
triangle
Problem 19. Ten gangsters are standing on a flat surface, and the dis- tances
between them are all distinct. At twelve o’clock, when the church bells start
chiming, each of them shoots at the one among the other nine gangsters who
is the nearest, At least, how many gangsters will be killed?

Problem 20. Let ABC be a triangle and M be an interior point. Prove that

min{MA, MB, MC} + MA + MB + MC < AB + AC + BC.


Problem 21. A set S of points from the space will be called completely
symmetric if it has at least three elements and fulfils the condition :
for every two distinct points A, B from S the perpendicular besector plane of the
segment AB is a plane of simmetry for S.
Prove that if a completely symmetric set is finite then it cosists of the vertices
of either of a regular polygon, or a regular tetrahedron or a reg- ular octahedron.
Problem 22. For a triangle T = ABC we take the point X on the side
AX 4
(AB) such that XB = the point Y on the segment (CX) such that CY =
5
C ^ = 180 ◦ ^BC. We denote by Σ the set of all triangles T for
−ifApossible,
which XZ2YX and, the point Z on the ray (CA such that
X^YZ = 45◦ .
Prove that all the triangle form Σ are similar and find the measure of
their smallest angle.

circles orthogonal ABCpassing


23. Let toΩ through Ω( B,itsC )incircle
, ( A, Cand
) andΩ (,A, B) respec-
Problem be a triangle, J a Ω b , Ω c three
tively. The circles Ω a and Ωb meet again in C : in the same way we
obtain the points BJ and AJ . Prove that the circumradius of AJ BJ CJ is half the
radius of Ω.
Problem 24. Two circles Ω1 and Ω2 touch internally the circle Ω in M and N
and the center of Ω2 is on Ω1. The common chord of the circle Ω1 and Ω2
intersects Ω in A and B. MA and MB intersects Ω1 in C and D. Prove that Ω2 is
tangent to CD.

Lemma. The circle k1 touches internally circle k at A and touches one of


2
not contain
chords MN A. at Then
B. LettheC points
be theA, B, C areofcollinear
mid-point k’s arc and
MNCA.CB = CMProof.
which does . k’s
[Proof of the lemma] (fig 89a) The homothety with center A which
transforms k1 into k transforms MN into a tangent at k parallel to MN,
i.e. For
intothe
thesecond
tangentpart
at Cnotice
to k, so
thatA, B, C are collinear.
N^ C^AM, therefore O ACM ~
O MCB, whence CA.CB = CM 2 . MC ≡
Problem 25. The point M is inside the convex quadrilateral ABCD, such that

MA = MC, ^AMB = M^AD + M^CD and C^MD = M^CB +

^MAB.

Prove that AB.CM = BC.MD and BM.AD = MA.CD (Fig 90a).


Problem 26. Points A, B, C divide the circumcircle Ω of the triangle ABC
into three arcs. Let X be a variable point on the arc AB and O1, O2 be the
incenter of the triangles CAX and ABX. Prove that the circumcircle of the
triangle XO1O2 intersects Ω in a fixed point.
Problem 27. A convex quadrilateral ABCD has perpendicular diago- nals. The
perpendicular bisectors of AB and CD meet at a unique point P inside ABCD.
Prove that ABCD is cyclic if and only if triangles ABP and CDP have equal
areas.
points onpoint
P be the the sides and CD,EFrespectively,
ABsegment
on the such that PEsuch that= AE
: PF AB : :EB = Problem
CD. CF : FD. Let
28.
Let ABCD be a cyclic quadrilateral. Let E and F be variable
Prove that the ratio between the areas of triangles APD and BPC does
not depend on the choice of E and F.
Problem 29. Let I be the incenter of triangle ABC. Let K, L and M be the
points of tangency of the incircle of ABC with AB, BC and CA respec- tively.
The line t passes through B and is parallel to KL. The lines MK and ML
intersect t respectively at the points R and S. Prove that ∠ RIS is acute.
Problem 30. Let M and N be points inside triangle ABC such that

∠ MAB = ∠ NAC and ∠ MBA = ∠ NBC.


Prove that AM.AN BM.BN CM.CN
+ + = 1.
AB.AC BA.BC CA.CB
Problem 31. Let ABC be a triangle. H its orthocenter, O its circumcenter,
and R its circumradius. Let D be the reflection of A across BC, E be that of B
across CA, and F that of C across AB. Prove that D, E and F are

collinear if and only if OH = 2R.


Problem 32.the
the point on Letside
ABC BCbe a triangle
such that CDsuch that ∠
= 2BD. ACB
The = 2∠AD
segment ABC. Lel D be
is extended to
E so that AD = DE. Prove that
∠ECB + 180◦ = 2∠EBC.

the ∠ Let∠EBbe
Problem
The tangent33.at Let
A toABC be a triangle
its circumcircle such that
ω meets = 90
A BC
line at D.and < ∠C.
the reflection of A across BC, X the foot of perpendicular from A to BE, and
Y the midpoint of AX. Let the line BY meet ω again at Z. Prove that the line
BD is tangent to the circumcircle of triangle ADZ.
Problem 34. Let A1 A2 A3 be a non-isosceles triangle with the incentre I. Let
(the = 1, 2, 3,
Ci, iaddition be the smaller
of indices circle
being mod 3).through
Let Bi, iI =tangent
1, 2, 3,tobeAthe
i Ai+1 and Ai Ai+2
second point of
intersection of Ci+1 and Ci+2. Prove that the circumcen- tres of the triangles A1B1
I, A2B2 I, A3B3 I are collinear,
Problem 35. In an acute-angled triangle ABC, let AD.BE be altitudes and AP,
BQ internal bisectors. Denote by I and O the incentre and the circumcentre of
the triangle, respectively. Prove that the points D, E and I are collinear if and
only if the points P, Q and O are collinear.
Problem 36. The altitudes through the vertices A, B, C of an acute-angled
triangle ABC meet the opposite sides at D, E, F, respectively. The line through
D parallel to EF meets the lines AC and AB at Q and R, re- spectively. The line
EF meets BC at P. Prove that the circumcircle of the triangle PQR passes
through the midpoint of BC.
Problem 37. Let D be an internal point on the side BC of a triangle ABC.
The line AD meets the circumcircle of ABC again at X. Let P and Q be the
feet of the perpendiculars from X to AB and AC, respectively, and let γ be
the circle with diameter XD. Prove that the line PQ is tangent to

γ if and only if AB = AC.


Problem 38. Let ABCD
tion of its diagonals be aBD.
AC and convex
If quadrilateral and O be the intersec-

OA sin ∠ A + OC sin ∠C = OB sin ∠B + OD sin ∠D,


prove that ABCD is cyclic.
Problem 39. The bisectors of angles A, B, C of a triangle ABC meet its
circumcircle again at the points K, L, M, respectively. Let R be an internal point
on the side AB. The points P and Q are difined by the conditions: RP is parallel
to AK and BP is perpendicular to BL, RQ is parallel to BL and AQ is
perpendicular to AK. Show that the lines KP, LQ, MR have a point in common.
Problem 40. Let triangle ABC have orthocenter H, and let P be a point on its
circumcircle. Let E be the foot of the altitude BH, let PAQB and PARC be
parallelograms, and let AQ meet HR in X. Prove that EX is parallel to AP.
Problem 41. Let P be a point inside OABC such that

∠APB − ∠C = ∠APC − ∠B.


O APC, respectively. Show that AP,
Let D, E be the incenters of APB,
BD and CE meet in a point.
Problem 42. Let ABC be an acute-angled triangle with BC > CA. Let O be
its circumcenter, H its orthocenter, and F the foot of its altitude CH. Let the
perpendicular to OF at F meet the side CA at P. Prove that

∠FHP = ∠BAC.
Problem 43. Let ABC O be an equilateral triangle and let P be a point
in its interior. Let the lines AP, BP, CP meet the sides BC, CA, AB at the points
A1, B1, C1 respectively. Prove that

A1B1.B1C1.C1 A1 ≥ A1B.B1C.C1 A.
Problem 44. Let ABCDEF be a convex hexagon such that AB is parallel to
DE, BC is parallel to EF and CD is parallel to AF. Let R A , RC, RE denote the
circumradii of triangles FAB, BCD, DEF respectively, and let P denote the
perimeter of the hexagon. Prove that

P
RA + RC + RE ≥ 2.
Problem 45. Let the sides of two rectangles be a, b{ and} c, d {respec- tively,
with }
a < c d≤< b and ab < cd. Prove that the first rectangle can be placed within
the second one if and only if

(b2 − a2)2 ≤ (bd − ac)2 + (bc − ad)2.


Problem 46. Let ABC be an acute-angled triangle with circumcenter O
and circumradius R. Let AO meet the circle BOC again in AJ , let BO
meet the circle COA again in BJ and let CO meet the circle AOB again in CJ .
Prove that
OAJ .OBJ .OCJ ≥ 8R3.
When does equality hold ?
Problem
RD denote47.
theLet ABCD beofa the
circumradii convex quadrilateral
triangles DAB, ABC,and letBCD
R A , Rand
B , RCDA
C and

respectively. Prove that RA + RC > RB + RD if and only if ∠A + ∠C >

∠B + ∠D.
Problem 48. On the plane are given a point O and a polygon F (not
necessarily convex). Let P denote the perimeter of F , D the sum of the
distances from O to the vertices of F , and H the sum of the distances
from O to the lines containing the sides of 2.
F . Prove that D2 − H2 ≥ P4
SELECTED PROBLEMS FROM MATHEMATICS
AND YOUTH MAGAZINE 2018

Tran Ngoc Nam Mathematics


and Youth Magazine

1 For Secondary School


Problem 1 (1/488 (For 6th grade)). Given a pentagon ABCDE. Assume that BC
is parallel to AD, CD is parallel to BE, DE is parallel to AC, and AE is parallel
to BD. Show that AB is parallel to CE.
Problem 2 (2/488 (For 7th grade)). Prove that
1 1 1
1+ + +· · · + 2019

3 5 4035 >
4036 ·
12 1 3 12018
1+ + +· · · +

Problem 3 (3/488). Given two triples (a, b, c); (x, y, z), none of them
contains all 0’s, such that

a + b + c = x + y + z = ax + by + cz = 0.
Prove that the expression
P= (b + c) 2 + ( y + z) 2
ab + bc + ca xy + yz + zx

is a constant.

Problem 4 (4/488). Outside a triangle ABC , draw triangles ABD, BCE,


CAF such that 0

Prove that DF = AE.


A^ DB = ^BEC = CˆFA = 90
, A^BD = C^B C^

E = AF =

α.
Problem 5 (5/488). Show that the following sum is a positive integer
. Σ
2
1 1 1 1 +
S=1+ +· · · + + 1+ +· · · +
2 .2017 2 .
2017 Σ2
1 1 1

22
+···+ 2017 ·
2017 +···
Problem 6 (1/489 (Grade 6)). Find all pairs of integers (x, y) satisfying

x2 + x = 32018y + 1.
^
Problem 7 (2/489 (Grade 7)). Given a triangle ABC with B = 450,
0 ^
C^ = 30 . Let BM be one of the medians of ABC. Find the angle AMB.
Problem 8 (3/489).
Find the minimum Given
value of thereal numbers x, y satisfying 0 < x, y < 1. ).
expression
2xy − x 4xy − y+1
F = x2 + y2 +
·
Problem 9 (4/489). Given a circle ( O ) with a diameter
point C (C is different from A and B). Draw CH perpendicular AB. Onto (AB
O) pick a
at H. Choose M and N on the line segments CH and BC, respectively, such that
MN is parallel to AB. Through N draw a line perpendicular to BC. This line
intersects the ray AM at D. On the line DO choose two points F and K such
that O is the midpoint of FK. The lines AF and AK
respectively intersect (O) at P and Q. Prove that D, P, Q are colinear.
3 2
Problem
f ( x) 10 (5/489).
≥ Suppose
α(x that
− the polynomial
3
a) , f
∀x( x ) = x ≥+ ax +0.bxc +has
that3
non-negative real solutions. Find the maximal real number α so
Problem 11 (1/490 (For 6th grade)). The natural number a is coprime with
210. Dividing a by 210 we get the remainder r satisying 1 < r <
120. Prove that r is prime.

Problem 12 (2/490 (For 7th grade)). Given non-zero numbers a, b, c, d


satisfying

b2 = ac; c2 = bd; b3 + 27c3 + 8d3 ƒ= 0.


Show that a a3 + 27b3 + 8c3
= ·
d b3 + 27c3 + 8d3
5yz + all natural
(3/490). Find 3x +
solutions of 3z = 3xyz5. Problem 13
the equation
Problem 14 (4/490). Given a half circle with the center O, the diameter −
AB, and the radius OD perpendicular to AB. A point C is moving on the
^
arc BD. The line AC interdects OD at M. Prove that the circumcenter I of
the triangle DMC always belongs to a fixed line.
3 3
Problem 15 (5/490). Let x, y be real numbers such9 that x + y = 2. Find the
minimum value of the expression P = x2 + y2 +
2017 2018
x+y·
which are greater than 1 and satisfy x = y . Find
Problem 16 (1/491 (For 6th grade)). Consider all pairs of integersthe pair
x, ywith the
smallest
possible value for y.

Problem 17 (2/491 (For 7th grade)). Given 0 an isosceles triangle ABC


with the apex A. Suppose that A^ = 108 , BC = a, AC = b. Outside ABC,
construct the isosceles triangle ABD with the apex A and B^AD = 360.
Find the perimeter of the triangle ABD in terms of a and b.

integer
(3/491).aFind
is a divisor
positiveofintegers + 2isthat
n then na such also positiveof n + 2. Problem 18
a divisor
if the
Problem 19 (4/491). Given a triangle ABC inscribed the circle (O) with
diameter AC. Draw a line which is perpendicular to AC at A and inter- sect BC
at K. Choose a point T on the minor AB (T is different from A and B). The line
KT intersects (O) at the second point P. On the tangent line to the circle (O) at
the point T choose two points I and J such that KIA and KAJ are isosceles
triangles with the apex K. Show that
ˆ
a) T IP = TK J.
b) The circle (O) and the circumcircle of the triangle KPJ are tangent
ˆ
to each other.
Problem 20 (5/491). Find integral solutions of the equation

y2 + 2y = 4x2y + 8x + 7.
Problem 21 (1/492 (For 6th grade)). Let

1 1 1 1 1 1
M= + + + + + +...
10 20 35 56 84 120
1
a) Is the fraction a term of M? Why?
15400
b) Compute the sum of the 8 first terms of M?
Problem 22 (2/492 (For 7th grade)). Given a triangle ABC with AB ¡ AC.
The angle bisector of B^AC intersects the perpendicular bisector of BC at
M. Let H, K, and I respectively be the perpendicular projections of M on AB,
AC, and BC. Prove that H, I, K is collinear.
Problem 23 (3/492). Find integral solutions of the equation

x3 = 4y3 + x2y + y + 13.


with the diameter
24 (4/492). Given AC. Let M beABCD
a quadrilater the point on AB such
inscribed in thethat = AD.
AMThe
circle linesProblem
DM and
BC intersects at N, and the AN intersects the circle at
K. Let H be the perpendicular projection of D on AC. Assume that AB intersects
NH and CK P and Q. Show that
1 1 1
= + ·
MP MA MQ
Problem 25 (5/492). Solve the system of equations
 3x3 + 6x + 2 = 2y3
 3y3 + 6y + 2 = 2z3

3z3 + 6z + 2 = 2x3
Problem 26 (1/493 (For 6th grade)). Find 2018 numbers so that each of
them is the square of the sum of all remaining numbers.
Problem 27 (2/493 (For 7th grade)). Find the following sum

S = (1 + 2.3 + 3.5 + · · · + 101.201) + (12 + 22 + 32 + · · · + 1002).


Problem 28 (3/493). Find all pairs of positive integers (m, n) such that

n3 − 5n + 10 = 2m.
Problem 29 (4/493). Given a triangle ABC with BC = a , AC = b , AB =
5
0 a.
^ ^
c , and 3B + 2C = . Prove that b + c ≤ 4

180
Problem 30 (5/493). Solve
2 the √
2 system of equations
2 0
.
x − y + √ x − y√ + = √
x + 8y + 4 x − 8 y − 4 xy = 0
m
primes p satisfying
31 (1/494 each of the
(For 6th grade)). Findfollowing numbers1)m,p n +
equalities
all natural pn = pm+n; Problem
and
2) pm + pn = pmn.
Problem 32 (2/494 (For 7th grade)). Given a triangle ABC. Let M, N,
and P respectively be the midpoints of AB, AC, and BC. Let O be the
intersection between CM and PN, I be the intersection between AO and BC, and
D be the intersection between MI and AC. Show that AI, BD, MP are
concurrent. 1− 4 x
Problem 33 (3/494). Solve the equation √ 2x
= 2 − 2.
x +1
2x + 1
Problem 34 (4/494). Given a right triangle ABC with the right angle
arbitrary
AH be thepoint D (DOn
altitude. =H ). opposite
the Through ray
D draw
of thethe
rayline
HAperpendicular
pick an BD.toThat
A. line
Let
intersects AC at E. Let ƒK be the perpendicular projection
of E on AH. Show that DK has a fixed length when D varies.
Problem
minimum 35
and(5/494).
maximum Given realofnumbers
values x, y such that x2 + y2 = 1. Find the
the expression


T = 4 + 5x + 4 + 5y.
Problem 36 (1/495 (For 6th grade)). Show that it is impossible to write
292018 as a sum of n consecutive positive integers for any n ∈ N, n ≥ 2.
Problem 37 (2/495 (For 7th grade)). Find the last twelve digits of the number
51040.
Problem 38 (3/495). Find integral solutions of the following systems of
equations
3a2 + 2ab
2 +2 3b22= 12 ) . (z 3)(x2 + y2) 2xy = 0
a) a +b =c − x+y=z
. ;
Problem 39 (4/495). Given a right triangle ABC with the right angle
− 1
A. In the angle B^AC draw the rays Ax, Ay such that C^Ax = A^BC;
b
1 2
^ ^
BˆAy = ACB .The ray Ax intersects the angle bisector of ACB at Q, the
2 AK
ray Ay intersects BC at K. Compute the ratio ·
AQ
x2 2x + 2
Problem 40 (5/495). Solve the equation 3.
3 −
+ 2x = 5.
2x − 1
Problem 41 (1/496 (For 6th grade)). Find 3-digit numbers so that each
of them is 9 times the sum of the squares of its digits.

Problem 42 (2/496 (For 7th grade)). Given a triangle ABC with


0 0

^ ^
D
B^ = 45 , DC^ = CB 30 .=Outside
150 . Prove
ABC that
wethe triangle
choose D soABD
thatis equilateral.
BC =
that [(x + 43
Problem y)((3/496). + x)]23 ≥
y + z)(zGiven xyz(2x + ynumbers
non-negative + z)(2yx,+ y,z and
+ x)( + x + y) .
z.2zShow
Problem 44 (4/496). Given a semicircle (O; R) with the diameter AB and the
tangent Ax. On the ray Ax choose a point K so that AK = R. The circle with
center K and radius R intersects KB at I. The circle with center B and radius BI
intersects (O) at E. The ray BE meets Ax at C. Show that the perpendicular
bisector of BC is tangent to (O).
Problem 45 (5/496). Solve the system of equations
 √
 x2 (y − 2x2y).+ y ( xy + 1)(3 − xyΣ) = y5

 x (1 −
3
2 4 = x2y2
1 1
x2 ) + x + . + ( x 2 − y2 )2
p2 (1/497
+ q2
1.2.3 .1! +. 2!. + 3!
n. +
that there exist
+ 5895 prime numbers
. Notice that n!p, q=satisfying + n! =46
Problem
(For 6th grade)). Find all positive integers n such ··
Problem 47 (2/497 (For 7th grade)). Given an acute angle ABC · with the
altitude AH, the median BM, the angle bisector CK. Show that if HMK is an
equilateral triangle then so is ABC.
Problem 48 (3/497). Suppose that n is a positive integer so that 3n + 7n is
divisible by 11. Find the remainder in the divison of 2n + 17n + 2018n2 by 11.

Problem 49 (4/497). Given a circle (O) and suppose that BC is a fixed


chord of (O). Let A be a point moving on the major arc BC; and M, N
respectively the midpoints of AB and AC. Show that each of the altitudes
MMJ and NN J of ∆AMNcontains some fixed point.
Problem 50 (5/497). Solve the equation

13x − 5x − 13 = (16x − 11) 2x2 − 3.
2

Problem 51 (1/498 (For 6th grade)). Let A = 12016 + 22016 + 32016 + +


··
·
20152016 + 20162016. Show that A is not a perfect square.
that
52 (2/498 = m!
2.k! (For 7th−grade)).
2.n! Find n! = 1.2.3
whereall natural . . k,. m,
numbers so = 1. Problem
n; n0!
Problem 53 (3/498). Find integral solutions (x; y) of the equation

√ √
.y − .y 2 + 2Σ . x + 4 + 2x = 2.
Problem 54 (4/498). Given a triangle ABC with the angles satisfying

A^ : B^ : C^ = 8 : 3 : 1. Let AD 4 of the angle A (D is on


1 1be the angle bisector
BC). Assume furthermore that + BD2 BC2 3
AD. = · Compute the length of
√3
3
Problem 55 (5/498). Solve the equation 2x + 20 = 9x x3 − 7.
2 For High School
Problem 56 (6/488). Solve the system of equations

.
2x5(− 1)3 y√− x2 y + 10x3 +2y2 − 5y = 0
x +2x
y − 5 = y − 3x + x −
2π 8π 10π
Problem 57 (7/488). Prove that x02= cos + cos + cos
21 21 21 is a solu-

tion of the equation 4x3 + 2x2 − 7x − 5 = 0.


Problem 58 (8/488). In any triangle ABC, show that
1 a+b b+c c+a
cos ( A − B) + cos (B − C) + cos (C − A) ≤ . + + Σ·
x y z
that x + Given
(9/488). y + z6 =positive
1. Show + x,
that a,axb, c,
numbers cz ≥
byy,+z and a .b .c . Problem 59
assume

2 c a b
Problem 60 (6/489). Solve the equation (1 2 sin x)(cos 2x + sin 2x) =
1
· −
2
Problem 61 (7/489). Given the following system of equations

 xyz+(yy++zz− zxx(z)
+ x − y) x + = a
 y + z xy (x + y
− z) x + y +
z =b

=c

where a, b, cthat
a) Show are the
positive
systemparameters.
always has a positive solution.

b) Solve the system when a = 2, b = 5, c = 10.


Problem 62 (8/489). Suppose that a, b, c are the lengths of 3 sides of a
ab bc ca 1 2r
triangle a, b, c. Prove that 2 2 + 2 2 + 2 2 ≥ + Rwith R, r
a +b b +c c +a 2
respectively are the inradius and the circumradius of the triangle.
Problem 63 (9/489). For any integer n which is greater than 3, let P =
√ √8
6
√0 12
√0 n − 1. Show that (24n
2+24n
2
) 3n +n−12 ≤P< 3.
3. 4 ...

(n3 −n
√)

Problem 64 (6/490). Given 3 positive numbers a, b, c satisfying abc = 1.


Prove that
1 + 1 + 1 ≤ 1.
a5 + b5 + c2 b5 + c5 + a2 c5 + a5 + b2

π √
Problem
a cos 65 (7/490). Find all positive integers a, b such that .8 + √32 + 768 =
b
·Problem 66 (8/490). Given a triangle ABC. Let (K) be the circle pass- ing
through A, C and is tangent to AB and let (L) be the circle passing through A, B
and is tangent to AC. Assume that (K) intersects (L) at another point D which is
different from A. Assume that AK, AL respec- tively intersect DB, DC at E, and
F. Let M, N respectively be the mid- points of BE, CF. Prove that A, M, N are
colinear.
Problem 67 (9/490). Given real numbers a, b, c such that

2(a2b2 + b2c2 + c2a2) ≥ a4 + b4 + c4.


Prove that
|b + c − a| + |c + a − b| + |a + b − c| + |a + b + c|

= 2(|a| + |b| + |c|).


Problem 68 (6/491). Sove the system of equations

. √ log2 x + log√2 y + log2 z =√ 3


x2 + 4 + y2 + 4 +
Problem 69 (7/491). Show that z2 + 4 = 2(x + y + z)
n→∞ , √
lim ‚. 1 + 2.1 + 3.1 + · · · + .1 + (n − 1) 1 + n = 3.
Problem 70 (8/491). Letma, mb, mc be the lengths of the medians of the triangle
with the perimeter 2. Show that
max{1 ; 3
√3 √ ,
3 r } ≤ m a + mb + m c < 2
where r is the inradius of the triangle.
Problem 71 (9/491). Assume that a, b, c are three non-negative numbers

su√ch that a +√b + c = 3. F√ind the maximum value of the expression P =

a b3 + 1 + b c3 + 1 + c a3 + 1.
Problem 72 (8/492). Given a triangle ABC inscribed in the circle (O).
The tangent lines of (O) at B and C intersect at P. The line which goes through A
and is parallel to BP intersects BC at M. The line which goes through A and is
parallel to BC intersects BP at N. Suppose that I is the intersections between AP
and MN. Prove that four points B, I, O, C lie on a circle.
Problem 73 (9/492). Given the equation

x3 + mx2 + n = 0. (∗)
Find m, n so the the equation (*) has three distinct non-zero real roots
u, v, t satisfying
u4 + v4 + t4 = 3.
u3 − 2n v3 − 2n t3 − 2n
Problem 74 (6/493). Given three positive numbers a, b, c satisfying a + b + c
= 3. Show that
1 + 1 + 1≤ · 3
2 2 2 2
(a + b) + c (b + c) + a (c + a) + b2 2 5
. the system
Problem 75 (7/493). Solve √ of equations
x − 1 = 4 9 + 12y − 6y2
√4
y− 1= 9 + 12x − 6x2
Problem 76 (8/493). Given a right prism with equilateral bases ABC.AJ BJ CJ .
Let α be t√he angle between the line BC’ and the plane (A’BC). Prove that
sin α ≤ 2 3 − 3.
Problem 77 (9/493). Given positive numbers a, b. Show that
1 min a, b 2
Σ1 −
2 maxΣ(a, b) b− a
ln b + ln a
( ) a −

≤ Σ − 1 Σ2 .
2 min ( a,
1 max(a, b) b )
Problem 78 (6/494). Find conditions on a, b besides a > b ≥ 1 so that
the system −
. x2 = (a − y)(a + y + 2)

has unique solution.


y2 = (b − x)(b + x + 2)
Problem 79 (7/494). Given positive real numbers a, b, c. Prove that
b ( 2 a − b ) c( 2b − c) a (2 c − a ) 3
+ + ≤ ·
a ( b + c) b ( c + a) c ( a + b) 2
Problem 80 (8/494). Let A, B, and C denote the angles of an arbitrary
triangle ABC. Show that
sin A sin B sin C 3
+ + ≥ ·
tan B tan C tan A 2
Problem 81 (9/494). Find all pairs of positive integers (x; y) satisfying

x3 + y3 = x2 + 72xy + y2.
Problem 82 (6/495).xGiven2 real numbers
y x, z that 2xyz =
2 y, z such 3 1, show that
+ +
1 + xy 1 + zy ≥ .
. . .
1 + xz 4
Σ Σ Σ
Problem 83 (7/495). Suppose that the polynomial P(x) = x2018 − ax2016 +
a (a is a real parameter) has 2018 real solu√tions. Show that there exists at
least one solution x0 of P(x) with |x0| ≤ 2.
Problem 84 (8/495). Given a pyramid S.ABCD with the base ABCD is a

parallelogr√am, and SA = SB = SC = a, AB = 2a, BC = 3a. Let SD = x


(0 < x < a 14). Find x in terms of a so that the product AC.SD obtains
its maximum value.
isfying + y + z =that
(9/495).xSuppose 1. x,
Find
y, zthe
aremaximum valuenumbers
non-negative of the expression
sat- Problem 85
2
P = xy + yz + zx + ( M − m)3
9

where M = max{x, y, z} and m = min{x, y, z}.


Problem
tions 86 (6/496). Find real solutions of the following system of equa-

 1 1 1 1

x + y + z =8 xyz
  (x + y + z)3
x + y + z = 27
xyz +

Problem 87 (7/496). Solve the equation log2 x = log5 −x3.


Problem 88 (8/496). Show that the sums of the squares of the distances
from the midpoints of the edges of a triangle to its orthocenter and cir-
cumcenter are equal.
sum of all of its digits by f1(k). Let fn+1(k) = f1( fn(k)) (n = 1, 2, . . .). Problem
89 (9/496). For any positive integer k, denote the square of the Find f2018 21990 .
.
Problem 90 (6/497). Solve the system of equations

Σ  √ 2 √
 4x + 5 + 4y2 + 5 = 6 |xy|
1 2 + 1 2 +
  x 4
y4 + 2 y8 = 16
x8
Problem 91 (7/497). Show that the following inequality holds for all
positive numbers a, b, c

1
a2 + b2 + c2 ≥ (ab + bc + ca)+
2
+
(a + b)(b + c)(c + a)
.2( a + b + c)( a3 b3 + b3 c3 + c3 a3 )
Problem 92 (8/497). Given triangle ABC with the incenter I, the centroid
G. There lines AG, BG, CG respectively intersect the circumcircle of ABC at
A2, B2.C2. Show that
GA2 + GB2 + GC2 ≥ IA + IB + IC.
Problem
(g(x)) = g93
( f ((9/497).
x)) x [Given continuous
a, b]; where functions
a, b are f, g: [a,Show
real numbers. b] [a, ]the
b→
that such that f
equation
f (x) = g(x) has at least one solution.

Problem 94 (6/498). Solve the system of equations
 √ √
2( x + 4y − 8) = . y2 + x − 3 + 1 − yΣ ×
2 2


 × . y2 + x − 3 + 1 + y Σ


2√ y4 + 5 = x
Problem 95 (7/498). Solve the equation
4

√ 3
1 + 2log16 x2 + .4 − log x8 4
4
− = 0.
log2 x2 3
+ log2 x − log x3 + 2
2 ABC2 ( AB < points
AC), B,
Problem 96 (8/498). Given a triangle
given circle (O), with the point A can be varied 3 and2 the inscribed
C are in a
by BC.
fixed, andAtwocircle (OA
points
J
) is
andinternally tangent
O are always to same
on the (O) atsideT determined
(T is outside the
triangle
ABC) and is tangent to the sides AB, AC respectively at P, Q. The
line The lines TB, TC meet again (OJ ) respectively at E,
F ( EPQ intersects
F = T ).BC at R.that
= T, Prove
ƒ EF is parallel to the line BC.
a) The line
b) The line RT always passes through a fixed point when A varies.
Problem 97 (9/498). Let a, b, c be positive numbers such that

a + b + c + 2 = 2abc. Prove that


a+2 b+2 c+2
√ +√ +√ ≤ 2.
2
6(a + 2) 2
6(b + 2) 2
6(c + 2)
3 Towards Mathematical Olympiad
Problem 98 (10/488). Given an infinite sequence of positive integers
a1 < a2 < < an < such that ai+1 ai 8 for all i = 1, 3, . . . .
··· ··· −
For n = a1 + a2 +≥ · · ·+ an Show
two each n, letssquares
complete inside the half-open that[sfor
interval each n, there are at least
n , s n+1 ).
Problem 99 (11/488). Given two positive sequences (an)n≥0 and (bn)n≥0

which are determined as follows a0 = 3, b0 = 2 and
 1 + an+1
 a n + bn =

 2
an2 1 an+1
+ 1 = bn
for al n = 0, 1, 2,..........Show that they converges and find the limits.
Problem 100 (12/488). Given a triangle ABC with AB = AC ƒ .A circle
(O) passing through B, C intersects the line segments AB and AC at M
and N respectively. Let P be the intersection between BN and CM. Let Q
^
be the midpoint of the arc BC which does not contain M, N . Let K be the
incenter of PBC. Show that KQ always goes through a fixed point when
(O) varies.
Problem 101 (10/489).
a perfect square Find natural
for any natural numbers
number m. n so that 4m + 2n + 29 can- not be

Problem 102 (11/489). The sequence (an) is 2given as follows:


a 1

+1
( −
a1 = 12
; an = n , n = 1, 2, . . . .
a)Find lim an . 2− a √
a1 + a2 + · · · + an 2
b) Show that
n ≥ 1− for n = 1, 2, . . . .
2
Problem 103 (12/489). Given a triangle ABC inscribed in a cirle (O). A
point P varies on (O) but is different from A, B, and C. Choose M, N
respectively on PB, PC so that AMPN is a parallelogram.
a) Prove that there exists a fixed point which is equidistant from M and N.
b) Prove that the Euler cirle of AMN always goes through a fixed point.
a b c
(10/490). Find 2all triples of+positive 5integers (a, =
that b, c) such7 . Problem 104
Problem 105 (11/490). The sequence (un) is determined as follows
u1 = 14, u2 = 20, u3 = 32, un+2 = 4un+1 − 8un + 8un−1

with n ≥ 2. Show that u2018 = 5.22018.


Problem 106 (12/489). Given a triangle ABC with (O) is the circumcircle
and I is the incenter. Let D be the second intersection of AI and (O). Let E be
the intersection between BC and the line passing through I and perpendicular to
AI. Assume that K, L respectively are the intersections
Prove that through I and
DE and the line passing KI perpendicular = to OI. KL. between BC,
Problem 107 (10/491). For every n ∈ N ,let Fn = 22n + 1. For eachn ∈ N,
let q be a prime divisor of Fn, show that 2n+1 | q − 1. Furthermore, if

n ≥ 2,show even more that 2n+2 | q − 1.


Problem 108 (11/491). Find all functions f : R → R satisfying
f (x) f (y) + f (xy) + f (x) + f (y) = f (x + y) + 2xy for
all x, y ∈ R.
Problem 109 (12/491). Given a convex hexagon ABCDEF circumscrib- ing a
circle (O). Assume that O is the circumcenter of the triangle ACE. Prove that the
circumcircles of the triangles OAD, OBE and OCF has another common point
besides O.
Problem 110 (6/492). Solve the inequality


x3 + 6x2 + 9x ≤ x + 4 − 2.
Problem
is equal to111 (7/492).
1, find For a, b, value
the minimum c are positive numbers with the product
of the expression

1 1 1
P= + + .

a2017 + a2015 + 1 b2017 + b2015 + 1 c2017 + c2015 + 1


Problem 112 (10/492). Let p be an odd prime and a1, a2, . . . , ap is an arithmetic
progression with the common difference d which is not di- visible by p. Prove
that
p . 2
∏ ( a i + a 1 a 2 . . . a p ) .p .
i=1
Problem 113 (11/492). Find all functions f : R → R such that

f (x + f (y)) = f (x + y2018) + f (y2018 − f (y)), ∀x, y ∈ R.


Problem 114 (12/492). Given an acute triangle ABC with the altitudes
BE, CF. Let ST be a chord of the circumcircle of AEF. Two circles which go
through S, T is tangent to BC respectively at P and Q. Prove that the intersection
between PE, QF lies on the circumcircle of AEF.
Problem 115 (10/493, Daniel Sitaru and Leonard Giugiuc (Romania)).
Find the maximal positive number k so that the following inequality

2
+ bpositive
holds true fora all
2
k(a + b +a,c)b,≥c 3satisfying
+ c2 + numbers + k(ab + abc
bc +=ca1.)
Problem 116 (11/493). Given the sequence (xn)
x2 = x3 = 1

. (n + 1)(n − 2)xn+1 = (n3 − n2 − n)xn − (n − 1)3xn−1 ∀n ≥ 3


Find all indices n so that xnis an integer.
Problem 117 (12/493). Given a cyclic quadrilateral ABCD. Let K be the
intersection between AC and BD. Let M, N, P, and Q respectively be the
perpendicular projection of K on AB, BC, CD, and DA. And then, let X, Y, Z,
and T respectively be the perpendicular projection of K on MN, NP, PQ, QM.
Prove that AXCZ and BYDT have equal areas.
Problem 118 (10/494). For any integer n, show that

an = √ √ + √ √
3 +10 5 .7 + 3 2 5 Σn 10 5 .7 − 32 5 Σn + 2
3−
5
is a perfect square.
Problem 119 (11/494). A class has n students attending n 1 clubs.−Show that
we can choose a group of at least two students so that, for each club, there are
an even number of students in that group attend it.
on
120the(12/494). Let (Ia1),square
side AB.Given (I2)respectively
ABCD and be Ptheisincircles of ADP,
an arbitrary CBP.
point Problem
Assume that
DI 1, CI2intersect AB respectively at E, F. The line through
E which is parallel to AC intersects BD at M and the line through F which is
parallel to BD intersects AC at N. Show that MN is a common tangent
to (I1) and (I2). √3
Problem 121 (10/495). Consider the sequence ( an ): an = n + [ n] , n
√ √
positive integers ( [ 3 n] is the integral part of 3 n ). Suppose there ex-
ists a positive
consecutive integer
natural k such
numbers withthat
p =the terms
6015 2006a+k; 1.ak+1 . ; ak+p are p + 1
; . . that
Show
×
k > 8.10 + 6.10 .
9 7

Problem 122 (11/495). Find the least number k so that in any subset of k
elements of 1, 2, . . . , 25 we can always find at least a Pythagorean triple.
Problem 123 (12/495). Let ABC be a triangle inscribed in a circle (O). Suppose
that AH is the altitude and the line AO intersects BC at D. Let K be the second
intersection of the circumcircle of ADC and the circum- circle of AHB. Suppose
that the circumcircle of KHD intersects (O) at M and N. Let X be the
intersection of MN and BC.
Show that XA = XK.

.
Σxn
Problem 124 (10/496).
x1 = 0, xn+1 = 1The, ∀ n ∈ N ∗ .( xn ) (n ∈ N ∗ ) is defined as follows
sequence
4
Show that the limit of the sequence exists and find it.
Problem 125 (11/496). Find all functions f : R → R such that

4
f (x) f (y) − 9 xy = f (x + y), ∀x, y ∈ R.
Problem 126 (12/496). Given a quadrilateral ABCD inscribed in a circle
(O). AC intersects BD at E, AD intersects BC at F. Let (O1) be the circle which
is tangent to the rays EA, EB and is internally tangent to (O); (O2)
the circle which is tangent to the rays FA, FB and is externally tangent to
(O) at a point on the arc AB which does not contain C, D. Prove that the
intersection of two external tangents to (O1) and (O2) lies on the circle (O).
Problem 127 (10/497). Find all pairs of integers (m, n) so that both

m5n + 2019 and mn5 + 2019 are cubes of integers.

Problem 128 (11/497). Let S = 1, 2, { 3, . . . , 2019 and suppose that


} that for every i S, there exist
S1, S2, . . . , S410 are subsets of S such ∈
exactly 40 of them containing i. Prove that there exist different indices
j, k, l ∈ { 1, 2, . . . , 410 } such that Sj.∩ Sk ∩ Sl ≤ 1.
.
Problem 129 (12/497). Given a cyclic quadrilateral ABCD with E is the
intersection between AC and BD. The circumcircles of EAD and EBC intersect
at another point F (besides E). The perpendicular bisectors of BD, AC
respectively intersect FB, FA at K, L. Show that KL goes through the midpoint
of AB.
Problem 130 (10/498). a) Given rational numbers a1, a2, . . . , an
(n ∈ Z+). Show that if, for any positive integer m, am + am + · · · + am is
1 2 n
an interger then a1, a2, . . . , an are integers.
b) Is the above conclusion still correct if we only assume that
a1, a2, . . . , an are real?
Problem 131 (11/498). people come and sit on the bench, one by one. The first
person can choose any seat. The next people firstly try to avoid to sit next to the
previous ones. If they cannot avoid, then they can choose any empty seat. How
many such arrangements are there?
be the lengths
(12/498). Givenofathe medians
triangle ABC.corresponding the sides BC = a, Problem 132
Let ma, mb, mtoc respectively
AC = b, AB = c. Show that 1 1 1 √
( ab + bc + ca) . + + Σ ≥ 2 3(ma + mb + mc ).
a b c
When does the equality hold?
SELECTED VIA MAIL COMPETITION QUESTIONS FROM
CHILDREN’S FUN MATHS JOURNAL
Nguyen Ngoc Han Children’s
Fun Maths Journal
1 1 1
Problem 1 (Nguyen Ngoc Hung, CFMJ 1(187)). Let A = + + +
1 2017 1! 2! 3!

···+ . Compare A with . (Notation: n! = 1.2.3 . . . n).


2018! 1152
Problem 2 (Ta Thap, 2(187)). Two boxs contain 100 marbles and 123
marbles respectively. Hong and Ha play taking arbitrary number of marbles in
one box. The winner is the one who gets the last marble. Hong takes the marbles
first. Show the way helping Hong to win the game.
that
(Nguyen Duc a2Tan, 3(187)).
+ +
5ab prime numbers
Find ba,2 b, c, such
= 7c. Problem 3
triangle.
ProblemM4 (Luuis inside triangleCFMJ
ly Tuong, such that
ABC4(187)). LetMA : MB : MC = 3 : 4 : 5.
AHABC be an equilateral Draw
AH perpendicular to BM at H. Find the ratio .
CM
x
Problem 5 (Lai Quang Tho, CFMJ 1(188+189)). Find the integer x such that
−1
is the square of a fraction.
9x + 7
Problem 6 (Truong Quang An, CFMJ 2(188+189)). Find digits x, y, z,
2
t, u such that (xy + ztu) = xyztu.
Problem 7 (Nguyen Duc Tan, CFMJ 3(188+189)). Find the smallest pos-
itive integer equal to 3 times the cube of a whole number and 4 times the
exponent of fourth of another whole number.
Problem 8 (Thai Nhat Phuong, CFMJ 4(188+189)). Let ABC and ADC
be triangles such that B B D D^CA are acute angles and
^ ^ ^
AC,of AC.
B, D are on the opposite side CA, AC,

Given that AB2 + CD2 = AD2 + BC2, prove AC ⊥ BD.


positive integers (a; b) such that 9a2b2 − 5a + 2018
5b be a square number and a2019
= 2020 × b . Problem 9 (Nguyen Ngoc
Hung, CFMJ 5(188+189)). Find all pairs of
given lines with
(Lai Truong Sinh,equation y = (m 1)x +On
CFMJ 6(188+189)). 2mOxy
3. Inco-ordinate, we Problem
these lines, choose 10
3 arbitrary

lines. In these angles formed by two lines of these 3
lines which are no common interior points, we choose the angle with the greatest
measure x. Find the smallest value of x.
Problem 11 (Vu Dinh Hoa, CFMJ 7(188+189)). A bench has 4 seats. 4 people
sit in this bench. The first person sits randomly. The next three people choose the
seats with no adjacent people. If they cannot find the seat they want, they will
choose the vacant seats. How many ways to arrange the seats for these four
people are there?
right-angled
12 (Le Trantriangle AC =8(188+189)).
at C withCFMJ
Quoc Canh, b, CB = a; ABLet= c.
ABC BFaare Problem
AE, be
medians of triangle ABC. AE = m; BF = n. Let r be a radius of the r2
.
inscribe circle of triangle ABC. Find the greatest value of A = m2 + n 2
Problem 13 (Le Son Tung, CFMJ 1(190)). Find all prime numbers a, b, c

such that ac − b + c and ca + b are also prime numbers.


such that a14< (Vo
Problem b < Xuan b − a,CFMJ
c and Minh, c − b,2(190)).
c − b +Find
a are also numbers
prime prime numbers.
a, b, c
Problem 15 (Cao MInh Quang, CFMJ 3(190)). Find all positive integers
m, n such that
a) 3m − n! = 1;

b) 3m − n! = 2.
les triangle
16 (Tran Quang AB = CFMJ
with Hung, and Q are
AC. P 4(190)). inside
Let ABCtriangle ABC such that Problem
be an isosce-
∠APB = ∠AQC and AP = AQ. Prove that PQ ǁ BC.
Problem 17 (Nguyen NGoc Hung, CFMJ 5(190)). Given the sum
1 1 1 1
A= + + +···+ .
23 + 3 33 + 4 43 + 5 20183 + 2019
1
Compare A
and .
6 Nam, CFMJ 6(190)). Solve the following equa- tion
Problem 18 (Mai Van

3
2x + 2 = x3 + 9x2 + 26x + 28.
Problem 19 (Vu Dinh Hoa, CFMJ 7(190)). Find all natural numbers n ≥
3 such that we can fill in an n n grid
× with real numbers satisfying the following
conditions simultaneously:
1) The sum of all numbers in an arbitrary 2 × 2 grid is positive.
2) The sum of all numbers in an arbitrary 3 × 3 grid is negative.
lateral
Problem inscribed
20 (Nguyen Tan(O
in circle Ngoc, that ∠
) suchCFMJ BCD <
8(190)). Let90ABCD
o
, the diagonal AC
be a quadri-
intersects BD at the midpoint M of BD. Draw line b through D, perpen-
dicular to DC that cuts the perpendicular bisector of BD at E. AB cuts
CD at F. Show that BC is perpendicular to EF.
Problem 21 (Nguyen Ha Ha Uyen, CFMJ 1(191)). Given that

M = 11 + 22 + 33 + · · · + 9999 + 100100.
Show that M has 201 digits and find the first 2 digits of M.
Problem 22 (Cao Ngoc Toan, CFMJ 2(191)). Given that

1 2 3 4 2019
S =2018 .1 + 2 + 3+ · · · + 2018Σ .
Prove that S is not a natural number.
having
Problemthe23form ab such
(Le Son Tung,that ba, (a +Find
ab, 3(191).
CFMJ 1)b,all(bthe
+ 2-digit
1)a arenumbers
all 2-digit prime
numbers.
Problem 24 (Tran Quang Hung, CFMJ 4(191). Given acute triangle ABC, we
construct two right-angled isosceles triangles ACE with hypotenuse AC and ABF
with hypotenuse AB (both of them are outside triangle ABC). Then, we construct
another right-angled isosceles DEF with hy- potenuse EF such that A, D and are
on the same plane side of EF. Show that AD is perpendicular to BC.
Problem 25 (Nguyen Ngoc Hung, CFMJ 5(191). Given real numbers x
and y satisfying
. x3 + y3 = 1948

(x + y)(x + 1)(y + 1) = 2017.

Find the sum x + y.


b such that the equation x4 + ax 3
+ +3x2 +2b2bx + ≥2 = 0 12.
has at least one real
root. Prove that a2 2
Problem 26 (Vo
Quoc Ba Can, CFMJ 6(191). Given real numbers a and
Problem 27 (Thai Nhat Phuong, CFMJ 7(191). Solve the following sys- tem of
equations

. √
x y+y + zy + x z= = 3xyz + 2.
xz +
√ √ √
Problem 28 (Doan Van Truc, CFMJ 8(191). Given an acute triangle ABC
(AB =< CE.
AC).Prove that
E the
are centroid
BD G of triangle ADE would move alonga
D and on side AB and side AC respectively so that
fixed line when points D and E move along the segments AB and AC,
respectively.
Problem
integer 29n (Truong
so Quangthat An, CFMJ
S(n).S1(192+193).
(n + Let 1) S(=n) be87.
theofsum
the
digits of positive integer n. Find the smallest value of positive
2
doublets of natural
30 (Nguyen numbers
Ngoc Hung, (a; b)2(192+193).
CFMJ satisfying 1000
Finda all b = 1001a.b2 + Problem
+ possible
×
Problem 31 (Nguyen Duc Tan, CFMJ 3(192+193). A box contains 219
marbles, Hong and Ha take turns to pick up the marbles which are ex- ponents
of 2 (20; 21; 22; . . . ). The person to pick up the last marble is the winner.
If Hong is the first one to pick, give a scenario in which Hong is the winner
with the fewest number of turns.
Problem 32 (Tran Quang Hung, CFMJ 4(192+193). Triangle ABC, we
construct an equilateral triangle APB and an isosceles triangle ACE
o
(EC = EA) outside ∆ABC such that ∠CEA = 120 o . Then, we construct
isosceles triangle BCD (DB = DC) with ∠BDC = 120 and A and D are on the
same side of BC.
Finally, we construct another isosceles triangle DEF (DE = DF) with
o
∠EDF = 120
that PF = CE.and F and B are on the same side of DE. Show
Problem 33 (Lai Quang Tho, CFMJ 5(192+193). Solve the following system of
equations
. 2x2 − y3 + 2xy + 2xy2 = 3

Where x < 0, y > 0.


x2 − y3 + xy = 1.
Problem 34 (Vo Quoc Ba Can, CFMJ 6(192+193). Given three non-zero
real numbers a, b and c. Show that
a2 − bc b2 − c2 − ab
+ 2 ca + ≥ 0.
a + 2b + 3c b + 2c2 + 3a2 c2 + 2a2 + 3b2
2 2 2
Problem 35 (Vu Dinh Hoa, CFMJ 7(192+193). How many natural 9-
digit numbers are there such that its decimal representation does not contain any
of these digits
Problem 1, 2,Cong
36 (Vu 3, 4, Minh,
5 and 6? CFMJ 8(192+193). Isosceles triangle ABC

(CA = CB) with ∠C = 15o. Suppose that there exists point D inside
∆ABC such th√at ∠ ADB = 105o and AD = 2BD.
Show that 5.AD.BC = 2CD.AB.
Problem 37 (Ta Thap, CFMJ 1(194). There are two types of trucks: 4-
tonne trucks and 11-tonne trucks. How many of each types of trucks would be
needed to transport 58 tonnes of goods if they are all fully loaded?
Problem 38 (Nguyen Ngoc Hung, CFMJ 2(194). Given that
A 33 53 73 93 113 133 153 173 1993
− + +
= − + 6 10 15 21 28 +··· + .
1 3 − − 36 4950
Compare A with 814.
Problem 39 (Thai Nhat Phuong, CFMJ 3(194). Five numbers 1, 2, 3, 4, 5
are arbitrarily grouped into 2 smaller groups. Prove that there always exists one
group in which the difference between two members of the group is equal to
another number in that group.
Problem 40 (Nguyen Ba Dang, CFMJ 4(194). Given triangles ABC and BCD
with common side BC. A and D are on different sides of BC and it is known
that:
∠ ABC = 36o, ∠CBD = 30o, ∠ BAD = 81o, ∠ CAD = 27o.
Find the measure of angles in triangle ACD.
Problem 41 (Nguyen Tien Lam, CFMJ 5(194). A positive integer n is named
interesting number if there exists other xpositive
2
+ y2 integers x, y, z, t satisfying:
=
n z2 + t2 .
a) Prove that there are an infinite number of interesting numbers.
b) Is 2019 an interesting number? Give an explanation.
Problem 42 (Luu Ly Tuong, CFMJ 6(194). Solve the equation

(x + 3) −x2 − x + 48 = x − 24.
Problem 43 (Luu Ly Tuong, CFMJ 7(194). n is a positive integer (n

3).
a) Find
positive the greatest
integers positive integer
not exceeding k(n) so
k(n), there that among
always exists n3 arbitrary
num- bersdistinct
(not
necessarily
to the other distinct)
one. in which the sum of two of them is equal

b) Find
positive the greatest
integers positivehinteger
not exceeding h(nalways
(n), there ) so that among
exists n arbitrary
3 dis- distinct
tinct numbers in
which the sum of two of them is equal to the other one.

Problem 44 (Huynh Thanh Tam, CFMJ 8(194). Two circles of different


sizes with centers O and OJ intersect at points A and B. We construct a
common external tangent passing the two circles at C and D respec- tively so
that B is inside triangle ACD, then, draw AM perpendicular to
CD (where M is the point of intersection). From B, we construct a line
perpendicular to AB meeting AM at H. Prove that H is the orthocenter of
triangle ACD.
HOMC 2019 - Scientific seminar, Hanoi, April 3, 2019

ON THE S HAPIRO ’ S CYCLIC INEQUALITY AND SOME


RELATIVE ISSUES

Le Quy Thuong, Nguyen Minh Tuan

In this paper we prove an interesting extension of the Shapiro’s cyclic in- equality
for four, and five variables, and formulate a generalization of the well-known
Shapiro’s cyclic inequality. The method used in the proofs of the theorems in the
paper concerns the positive quadratic forms. Some stud- ied open problems which
can be seen as a real generalization of the Shapiro’s
proofs of P(3)
inequality are provided
are given in Section
at the end 3, and
of Section the relativesome
2. Moreover, inequalities
differentare cyclic
given in
Sections 4, 5.

1 Introduction
In 1954 Harold Seymour Shapiro proposed the inequality for a cyclic sum in
n variables as follows
x1
xn−1 n
x2 · · xn
+ + xn + x1 + ≥ ,
x2 + x3 + · 2
(1)
x3 + x4 x1 + x2

where xi 0,≥xi + xi+1 > 0 and xi+n = xi for i N. Although the prob-
lem (1) was settled in 1989 by Troesch [13], the history of long year proofs
of this inequality was interesting, and the certain problems remain (see [1, 2, 3,
4, 5, 6, 7, 13]). Motivated by the directions of generalizations and proofs of (1),
we consider the following inequality
x1
P(n, p, q) := x2 xn
+ xn−1 +
+ · · + pxn + qx1
px2 + qx3 px3 + qx4 · px1 + qx2
n

≥ , (2)
p+q

132
where p, q 0 and p + q > 0. It is clear that the inequality (2) is true for

n = 3. Indeed, by the Cauchyxinequality, we have x2
1
( x + x + x )2 = x ( px + qx ) +
..1 2 3 . .1
px2 + qx3 px3 + qx1 .x2 ( px3 + qx1 )
. .
+ x3 2 3
px1 + qx2
Σ
2 ≤ P(3, p, q)(p + q)(x1x2 + x2x3 + x3x1).
x3(px1 + qx2)
It follows that (x1 + x2 + x3)2 3
.

( p + q)(x1x2 + x2x3 + x3x1) p+q
P(3, p, q)


Obviously, (2) is true for every n 4 if p = 0 or q = 0.
it isIntrue
thiswhen
note, pby studyingq,the inequality
and (2) pin <
false when theq.case n = 4, we
Moreover, we show
give athat
case n = 5. It is worth ≥ if p <the
sufficient condition of p,saying thatwhich
q under then the inequality
q, inequality (2) is true(2)
in is
thefalse
the
next section. For short, for any n 3 and p = q = 1, let P(n) stand for every
even n 4. Two open questions are discussed at the end of


simply for P(n, 1, 1) (see [8, 9, 11, 12, 14]).
2 Main result
Without loss generality of (2) we assume that p + q = 1. The inequal-

ity (2) for n = 4 nowxis1 of the form


P(4, p, q) = x2 x3 x4
+ + + ≥ 4. (3)
px2 + qx3 px3 + qx4 px4 + qx1 px1 + qx2
Theorem 15. The inequality (3) is true for p ≥ q, and it is false for p < q.
Proof. By the Cauchy inequality we have

(x1+x2 + x3 + x4)2 ≤
Σ
P(4, p, q) x1(px2 + qx3) + x2(px3 + qx4) + x3(px4 + qx1) + x4(px1 + qx2) .
Σ
Hence
(x1 + x2 + x3 + x4)2
.
P(4, p, q) px1x2 + 2qx1x3 + px1x4 + px2x3 + 2qx2x4 + px3x4


It is an equality if and only if

px2 + qx3 = px3 + qx4 = px4 + qx1 = px1 + qx2. (4)


Consider the following quadratic form

ω(x1, x2, x3, x4) = (x1 + x2 + x3 + x4)2

− 4(px1x2 + 2qx1x3 + px1x4 + px2x3 + 2qx2x4 + px3x4).


By a simple calculation we obtain the canonical quadratic form ω as
follows
4q(2p − 1) 2
ω(t1, t2, t3, t4) = t2 + 4pqt2 + t, (5)
1 2 3
p
where

t1 = x1 + (1 − 2p)x2 + (1 − 4q)x3 + (1 − 2p)x4,


1 − 2p q
t2 = x2 + p x 3 − px4,

t3 = x3 − x4.
≥ We thus≥have
This implies that ω is positive. P(4, p, ≥ q) 4. It is
easily seen that if p q, i.e. p ∈ 12, then ω 0 for all t1, t2, t3 R.
Now let us consider the cases when ω vanishes. This ≥
depends con-
siderably at t1 = x1 of p with q. If p x=3 q,= i.e.
on the0 comparison
form ω attains p =t 1 ,=then
x2 the quadratic
0 and 2 2x4 = 0. By

(4) we

assert that P(4, p, q) = 4 whenever x1 = x3 and x2 = x4. Also, if
p > 12, then ω vanishes if and only if

t1 = x1 + (1 − 2p)x2 + (1 − 4q)x3 + (1 − 2p)x4 = 0,


1 − 2p q
t2 = x2 + p x 3 − xp4 = 0,

t3 = x3 − x4 = 0.
Combining these facts with (4) we conclude that P(4, p, q) = 4 when

x1 = x2 = x3 = x4.
Now we give a counter-example to the inequality (3) in the case p < q,

i.e. p < 1. Let x1 = x3 = a, x2 = x4 = b, and a ƒ= b. We shall prove that


2
a b a b a b
=2. + Σ< 4.
+ + +
pb + qa pa + qb
pb + qa pa + qb pb + qa pa + qb (6)
It is obvious that

(6) ⇔ p(2q − 1)(a2 + b2) + 2(p2 + q2 − q)ab > 0 ⇔ p(1 − 2p)( a − b)2 > 0.

The last inequality is evident as a ƒ= b and p < 1, so2 (6) follows.


The theorem is proved.
Remark 2. Let A denote the matrix of the quadratic form ω in the canonical
base of the real vector space R4. Namely,
 
1 1 − 2p 1 − 4q 1 − 2p
 1 − 2p 1 1 − 2p 1 − 4q 
A 1 − 4q 1 − 2p 1 1 − 2p .
1 − 2p 1 − 4q 1 − 2p 1
= 
Let D1, D2, D3 and D4 be  the principal minors of orders 1, 2, 3 and 4 respec-
tively of A. By direct calculation we obtain
D1 = 1, D2 = 4pq, D3 = 16q2(2p − 1), D4 = 0.
Then ω isofpositive
first part Theorem if and
15. only if Di 0 for every i = 1, 2, 3, 4. We find the

(2) in the case n = 5. It is sufficient to consider the case p +q


= 1.inequality
By the Cauchy inequality we reduce our work to the following Thanks to
the idea of using positive quadratic form we now study the
inequality
5

ϕ(x1, . . . , x5) = ∑ x2 i+ (2 − 5p)x1x2 + (2 − 5q)x1x3 + (2 − 5q)x1x4


i=1
+ (2 − 5p)x1x5 + (2 − 5p)x2x3 + (2 − 5q)x2x4 + (2 − 5q)x2x5 + (2 − 5p)x3x4

+ (2 − 5q)x3x5 + (2 − 5p)x4x5 ≥ 0.
The matrix of ϕ in an appropriate system of basic vectors is of the form
 
 2 2 − 5p 2 − 5q 2 − 5q 2 − 5p 

2 − 5p 2 2 − 5p 2 − 5q 2 − 5q
1
 
B = 2 2− 5q 2 − 5p 2 2 − 5p 2 − 5q
2 − 5q 2 − 5q 2 − 5p 2 2 − 5p
 
− 5p
which has the principal2minors
 2 − 5q 2 − 5q 2 − 5p 2

D 1, D
5p(4 − 5p) 25q(5pq − 1) 125(1 − 5pq )2
,D ,D , D 0.
1 = 2 = 3 = 4 = 5 =
4 4 16
This implies that the necessary and sufficient condition for the positivity
of the quadratic form ϕ is
5− √
√ 5+ 5
5 ≤p≤ .
10 10
We thus obtain a sufficient condition under which the inequality (2)

holds for n = 5. √
5− 5 5+ 5
Theorem 16. If √
10 ≤p≤ , then (2) is true for n = 5.
10
Remark
According3.toConsider
the proofthe inequality
of the (2) in
second part of the case n≥
Theorem 154,this
n isinequality < q.
even, andispfalse.

Indeed, we choose x1 = x3 = · · · = a, x2n = x4 = · · · = b. By the above

counter-example we conclude P(n, p, q) p+q


.
<
Open Problem. (a) Find pairs of non-negative numbers p, q so that the in-
equality (2) is true for every n ≥ 4.
(b) For certain n ≥ 5, which sufficient condition of the pair p, q so that the
inequality (2) is true.

andWe observe
Theorem 16that p = 0 oranswer
if detailed
is the q = 0, for
thenn it=is5.the case for every n ≥ 4,
3 Proof of P(3)

In this section we present some different proofs for P(3).


Problem 1 (Nesbitt’s inequality). Let a, b, c > 0 be given. Prove the inequal-
ity
a + b + c≥ 3
b+c a+c a+b
. (7)
2
Proof 1. Consider the following expressions

a b c
S= + + ,
b+c a+c a+b
b c a
M = b + c+ a + c+a + b,
c a b
N= + + .
b+c a+c a+b

Evidently, M + N = 3. Using the AM-GM inequality we have


a+b b+c c+a
M+S= + + ≥ 3,
c + a a + b b + ac + b ≥ 3.
N + S = b + c+ c + +a

Hence,

b+c c+a a+b

This implies

M + N + 2S ≥ 6.
2S ≥ 3.
The equality occurs if and only if a = b = c.
Proof 2. By calculating and applying the AM-GM inequality, we have
a b c
a b c
=. + 1Σ + . + 1Σ + . + 1Σ − 3
+ +
b+c a+c a+b b+c c+a a+b
1 1 1
= ( a + b + c) . + + Σ− 3
b+c c+a a+b
1 1 1 1
= .(b + c) + (c + a) + ( a + b)Σ . + + Σ− 3
2 b+c c+a a+b
3
≥ 3.3 (b + c)(c + a)( a + b) · 3. 1 1
9 3 +

= − 3= , c+a a+b
2 2

as desiered. The equality occurs if and only if a = b = c.


Proof 3. Without loss of generality, we can assume that a ≥ b ≥ c. Then
1
1 1
≥ .
b+c ≥
c+a a+b
By the Chebyshev’s inequality, we have
a b c
1 1 1 1
≥ (a + b + c) . + + Σ
+ +
b+c a+c a+b 3 b+c c+a a+b
1 1 1 1
= ((b + c) + (c + a) + (a + b)) . + + Σ
6
1 3
≥ .9 = . b+c c+a a+b
6 2

The inequality is proved. The equality occurs if and only if a = b = c.


Proof 4. Theasleft-hand
of variables follows side of (7) can be seen as a homogeneous function
x y z
f (x, y, z) = + + .

y+z z+x x+y


It is easily seen that f is a homogeneous function of zero-degree, i.e.
f (tx, ty, tz) = t0 f (x, y, z) = f (x, y, z),
for every0 x,
follows < y, z and
a, b, c < for
1. The > 0. Then,now
all tinequality onecan
canbe
assume that
seen as + b + c = 1. It
thea following
type
a b c 3

Consider the function


+ + ≥ .
1− a 1− b 1− c 2
x

We have .
1− x
f ( x) =
−2
f J (x) =
> 0,
(x − 1)3
for all x ∈ (0, 1). We deduce that f is a convex function defined on (0, 1).
By the Jensen’s inequality, we have a+b+c
f ( a ) + f ( b) + f ( c) ≥ 3 f . Σ= 3 f
1 3

3 ( )= .
3 2
The inequality is proved. The equality occurs if and only if a = b = c.
Proof 5. Put
xy = + a,
= bc + c,
z = a + b.

y+z− x
It follows a= 2 ,
z+x− y
b= 2 ,
x+y− z
c= 2 .
The inequality is now equivalent to the following ones:
y+z− x z+x− y x+y− z 3
+ + ≥ ,
y + 2z − x z + x2 − y x + y2− z 2
x + y + z ≥ 3.
x y z
Put
x
u=
,
y
y
v=z,
w= .
The last inequality becomes z x
1 1 1

Equivalently, +w− 1+ +u− 1+ + v − 1 ≥ 3.


u 1 v 1 w1

( + u) + ( + v) + ( + w) ≥ 6,
u numbers.
where u, v, w are positive v This inequality
w holds as
1
t + ≥ 2,
t
for all t > 0. Thus, P(3) is proved. The equality occurs if and only if

a = b = c.
Proof 6. Put
a + b + c = k.
It follows 0 < a, b, c < k. We shall prove the following inequality
a
k
+b+ c
1
k−b k−c
3 ≥ .
T= −a 2
Indeed, consider the function
t
f (t) = .
k− t
As Proof 4 we
inequality, havethat f is a convex function on interval (0, k). By the Jensen’s
shows
a+b+c
k
3 3 1
T 2 = .
≥ a+3b+c
= 3
k 2
k−
The inequality is proved. The equality occurs if and only if a = b = c.
Proof 7. By the Cauchy-Schwarz inequality, we have

( a + b + c) 2
Σ √a Σ2
. √ . √
. b c
= √ a ( b + c) + √ . b(c + a) + √ . c ( a + b)
b+c c+a a+b
a b c
≤ 2. + + Σ (ab + bc + ca).

Hence,
b+c a+c a+b ( a + b + c) 2 3
a + b + c≥ ≥ .
b + c a + c a + b 2(ab + bc + ca) 2

The inequality is proved. The equality occurs if and only if a = b = c.


Proof 8. Put
a b c
f (a, b, c) = + + ,

b+c
a+b a+c a+b
t= 2 .
We see that
a2 + b2 + c(a + b) c
f
(a, b, c) = 2t2 + 2tc c
≥ + = f (t, t, c).
t2 + c2 + 2ct 2t
This implies
3
f ( a, b, c) ≥ f (tJ , tJ , tJ ) = ,
2
where
a+b+c
tJ = 3 .

The inequality is proved. The equality occurs if and only if a = b = c.


Proof 9. By the Cauchy-Schwarz inequality, we have
a + b + c= a2
+ b2 + c2
b+c a+c a+b ab + ac bc + ba ca + cb
( a + b + c) 2

It is easy to prove that ≥ .


2(ab + bc + ac)

Then, (a + b + c)2 ≥ 3(ab + bc + ac).


≥ . 3
a b c
+ +
b+c a+c a+b 2

The inequality is proved. The equality occurs if and only if a = b = c.


Proof 10. It is easy to prove the following inequality for every x > 0
8x − 1
x .

4x + 1
By this, we have −1
≥ =
b+c b+c
8a +1 4−(ab+−b c+ c) 8a
a b+c
4a .

Similarly, ≥ −1 4−(ac+−b a+ c) 8b
c+a
8b
c+a c+a = .
b 4b
+ 1
− 4−(aa+−b b+ c) 8c
≥ a+b
8c
= .
a+b a+b
4c +11
c

We deduce
a + b + c≥ 6(a + b +=c) . 3
b + c a + c a + b 4(a + b + c) 2
The inequality is proved. The equality occurs if and only if a = b = c.
HOMC 2019 - Scientific seminar, Hanoi, April 3,
2019
Proof 11. We shall prove that
a ≥ 3
b+c 3a 2
3 3 . (8)
3

2(a 2 + b 2 + c 2 )
Equivalently, 3 3 3 1

2(a 2 + b 2 + c 2 ) ≥ 3a 2 (b + c).
Indeed, by the AM-GM inequality, we have
3 3 3 1
a 32 + b 32 + b 23≥ 3a 21b
a 2 + c 2 + c 2 ≥ 3a 2 c.
Hence, the above inequality holds. Similarly, we have two more inequal-
ities:
b 3b 32

c+a 3 3 3
(9)

c ≥ 2(a 2 +3cb 322 + c 2 )


a+b 3 3 . (10)
3

2(a 2 + b 2 + c 2 )
From the inequalities (8), (9), (10) we derive
a b c 3(a 32 + b 32 + c 23 ) 3
+ + ≥ 3 3 = .
b+c a+c a+b 3
2
2(a 2 + b 2 + c 2 )

The inequality is proved. The equality occurs if and only if a = b = c.


Proof 12. The inequality is of the following form

2a(a + b)(a + c) + 2b(b + c)(b + a) + 2c(c + a)(c + b)

≥ 3(a + b)(b + c)(c + a).


Equivalently,
a3 + b3 + c3 ≥ a2b + b2c + c2a.
By the AM-GM inequality, we have

143
HOMC 2019 - Scientific seminar, Hanoi, April 3, 2019

a3 + a3 + b3 ≥ 3a2b.

144
b3 + b3 + c3 ≥ 3b2c.

It follows c3 + c3 + a3 ≥ 3c2a.

a3 + b3 + c3 ≥ a2b + b2c + c2a.

The inequality is proved. The equality occurs if and only if a = b = c.


Proof 13. Without loss of generality, we can assume that

a + b + c = 1.
By the AM-GM inequality, we have
a 9 a ( b + c)

+ . a 9a(b + c) = 3a,
b+c 4 ≥2 b + c. 4
b 9b(c + a)

+ . b 9b(c + a) = 3b,
c+a 4 ≥2 c+ a. 4
c 9 c ( a + b)
+ . c 9c( a + b) = 3c.
≥2 .

This implies a+b 4 a+b 4


. Σ . Σ
a 9 a ( b + c) b 9b (c +
4 a)
+ + +
. Σ
b+c 4 c+a c 9 c (a + b)
+ + ≥ 3.

Equivalently,
a b c
a+b 4
9
+ + ≥ 3− (ab + bc + ca). (11)
+ c prove
In other side, web can a + cthat a + b 2
1 1
ab + bc + ca ≤ 3(a + b + c)2 = . 3 (12)
HOMC 2019 - Scientific seminar, Hanoi, April 3,
2019
From (11) and (12) it follows
a + b + c≥ . 3
b+c a+c a+b 2

The inequality is proved. The equality occurs if and only if a = b = c.


Proof 14. As above we can assume a ≥ b ≥ c. Put
a
x= ,
c
b
y= .
c
It follows x ≥ y ≥ 1. The inequality becomes
a b 1 3
+a ≥ .
b c c
+a
b

Equivalently, +c 2
c
+1
+1 c
c
x + 3
y ≥ − 1.
y+1 x+1 2 x+y
Applying the AM-GM inequality, we have
x+1 y+1

+ ≥ 2.
Or, y+1 x+1
x + y ≥ 2− 1− 1.
y+1 x+1 x+1 y+1
We shall prove that
1 1 3 1
2− − .

Equivalently,
≥ −
x+1 x+y y+1 2
1 1 1 1
− ≥ − ,
2 y+1 x+1 x+y
y− 1

145
HOMC 2019 - Scientific seminar, Hanoi, April 3, 2019
y− 1
.

2(y + 1) (x + 1)(x + y)

146
only if since xa
inequality is true, = ≥ y b 1. The equality
= c. The
occurs last
if and
Proof
assume15.
thatAsa the
≥ babove
≥ 1. proof, oneprove
We shall that that c = 1. Moreover, we can
can assume

a b 1 3

+ + ≥ .
Put b+1 a+1 a+b 2

A = a + b, B = ab.
The inequality becomes
a2 + b2 + a + b 1 3

+ ≥ .
Equivalently, (a + 21)(b + 1) a + b 2
A 2B + A 1 3
− + ≥ ,
A+B+1 A 2

Hence,
2A3 − A2 − A + 2 ≥ B(7A − 2).
and
7A − 2 > 2(a + b − 1) > 0,

A2 = (a + b)2 ≥ 4ab = 4B.


We now prove the following inequality

4(2A3 − A2 − A + 2) ≥ A2(7A − 2).


Or,
A3 − 2A2 − 4A + 8 ≥ 0.
Equivalently,

( A − 2)2(A + 2) ≥ 0.
occurs if
inequality holds, and
as A only ≥ 2.if The proof
a is=completed.
b =The equality
c. The last
Proof 16. Put
a b c
M( 3) : = + + .
b+c a+c a+b
By the Cauchy-Schwarz inequality, we have

( a + b + c) 2
Σ. . . Σ2
. . .
a
. a(b + c) + . b b ( c + a) + c.
= c ( a + b)
b+c c+a a+b
≤ M(3)Σa(b + c) + b(c + a) + c(a + b)Σ. It
follows
( a + b + c )2 a2 + b2 + c2
M =1+ .
2(ab + bc + ca) 2(ab + bc + ca)
(3) ≥
We can prove that
a2 + b2 + c2 1
≥ .
This follows 2(ab + bc + ca) 2
3
M( 3) ≥ .
2

The equality occurs if and only if a = b = c.


Proof 17. (Nesbitt, 1903). The inequality is equivalent to the following
inequality
a+b+c a+b+c a+b+c 9
+ + a + b ≥ 2,
b+c a+c
1 1 1 9
( a + b + c) . + + Σ≥ ,
b+c a+c a+b 2

1b+c 1 1
Σ(b + c) + ( a + c ) + ( a + b ) Σ Σ + a + c + a + b Σ ≥ 9.
The last inequality holds by the Cauchy-Schwarz inequality. In particu-
lar, we have

1 1 1
Σ(b + c ) + ( a + c) + ( a + b ) Σ Σ + + Σ
b+c a+c a+b

= Σ.(b + c)2 + .( a + c)2 + .( a + b)2 Σ×

Σ. (b +1 c)2 + . (a +1 c)2 + . (a +1 b)2 Σ ≥


Σ . . . Σ2
. + . + .
(b +1c)2 (a +1c)2 1
(b + c)2.
(a + c)2. (a + b)2. ( a + b) 2
= 32 = 9.

The equality occurs if and only if a = b = c.


Proof 18. This proof is based on the following:
a b c

= + Σ + + Σ.
32 21 (a +(ac)(
−b + c) (a + b()(a b−+ c) (a + b)((ab+−c)c)2

b) 2 c )2
and
we deduce only if
that the inequality a
is proved. = equality
The b occurs= if c. By this
Proof 19. (Nesbitt, 1903). Put
xy =
= ab +
+ b,c,
z = c + a.

The inequality becomes


x+z− y y+z− x x+y− z 3
2y + 2x + 2z ≥ 2.
Equivalently,
x+z y+z x+y 6
+ + ≥ .
y x z 1
Or,
x y y z x z
. +
y xΣ + . + Σ
z y + . + zΣ ≥x6.
The last inequality holds, since :
a b
+ ≥ 2,
b a

for every a, b > 0. The equality occurs if and only if a = b = c.


Proof 20. By the inequality of AM-HM we have
3
x+y+z ≥ ,
1 1 1
3 + +
x y z

for every x, y, z > 0. Equivalently,


1 1 1
(x + y + z) . + + Σ≥ 9.
x y z
Put
xy = + a,
= bc + c,
z = a + b.
equality occurs
in the above if we
inequality andderive
only if = bThe = c. Insert this
a proof.
the expected
Proof 21. The inequality becomes
a b c 9
+1+ +1+ +1≥ .
b+c c+a a+b 2
Without loss of generality, we can assume that a + b + c = 1. We shall
prove

a+b+c + b+c+a + c+a+b 9


Equivalently, b+c c+a
≥ .
a+b 2
1 + 1+ 1≥ . 9
b+c c+a a+b 2
The last inequality can be proved by applying the AM-HM inequality as
follows:
1 1
≥ + + 1a+b 3 3
b+c c+a = .
3 a+b+b+c+c+a 2

The proof is completed. The equality occurs if and only if a = b = c.

4 Proofs of P(4), P(5), and P(6)


This section presents the Shapiro’s cyclic inequality for n = 4, 5, 6.
The known proofs for other nJ s are very complicated, and these are out
of the scope of this work. However, the reader can refer the references of this
work.

Proposition P(4). Assume that a, b, c, d are arbitrarily non-negative real


numbers. Then,

a + b + c+ d≥ 2. (13)
b+c c+d d+a a+b

We shall present two proofs for P(4).


Proof 1. Consider the following expression

a b c d
S= + + + ,
b+c c+d d+a a+b
b c d a
M = b + c+ c + d+ d + a+ a + b,
c d a b
N= + + + .

Evedently,
b+c c+d d+a a+b

M + N = 4.
By the AM-GM inequality we have

a+b b+c c+d d+a


M + S = b + c+ c ++d d ++ a , b
da +
+b
a+c b+d c+a
N + S = b + c+ c + +d d+a + a+b,
a+c c+a b+d d+b
= + + +
b+c d+a c+d a+b
≥ 4(a + c) + 4(b + d) = 4.
a+b+c+d a+b+c+d
We deduce

Thus,
M + N + 2S ≥ 8.
S ≥ 2.
The equality occurs if and only if a = b = c = d.
Proof 2. Put
a b c d
M (4 ) := + + + .
b+c c+d d+a a+b
proof
applying of M(3) as
the Cauchy-Schwarz presented above,
inequality, which we to obtain
is similarly the By

M ( a + b + c + d )2
. (14)
We then have
(4) ≥
ab + ac + bc + bd + cd + ca + da + db

ab + ac + bc + bd + cd + ca + da + db = (a + c)(b + d) + 2(ac + bd)


1 1
≤ (a + c)(b + d) + (a + c)2 + (b + d)2
1 2 2
= ( a + b + c + d ) 2. (15)
2
Combining
and only if a(14)
= b =and d. we receive M(4)
c =(15) ≥ 2. The equality occurs if
Proposition P(5). Let a, b, c, d, e be arbitrarily real numbers. Then
a + b + c+ d+ ≥e 5
b+c c+d d+e e+a a+b
Proof. By the Cauchy-Schwarz inequality, we have . (16)
a b c d e 2
+ + + +
b+c c+d d+e e+a a+b
2
a b2 c 2
d2 e2

(a + b + c + d + e )2
≥ .
that+ bc + cd + de + ea + ac + ce + eb + bd + da
We shall prove ab
( a + b + c + d + e) 2 5
≥ .
ab + bc + cd + cd + de + ea + ac + ce + eb + bd + da 2
Indeed, this inequality is equivalent to the followings:

2(a2 + b2 + c2 + d2)

≥ (ab + bc + cd + cd + de + ea + ac + ce + eb + bd + da),
(a − b)2 + (b − c)2 + (c − d)2 + (d − e)2 + (e − a)2 + (a − c)2+
+ (c − e)2 + (e − b)2 + (b − d)2 + (d − a)2 ≥ 0.
The equality occurs if and only if a = b = c = d = e.

Proposition P(6). Let a, b, c, d, e, f be arbitrarily real numbers. Then


a + b + c+ +d e f
b+c c+d d+e e+f + ≥ 3. (17)
Proof. Applying the Cauchy-Schwarz inequality,f we + ahavea+b
a b c
+ + + +d +e f
b+c c+d d+e e+f f+a a+b
a2 b2 c2 d2 e2 f2
(a + b + c + d + e + f )2
≥ . (18)
ab + bc + cd + de + e f + f a + ac + ce + ea + bd + d f + f b
Denote

S := ab + bc + cd + de + e f + f a + ac + ce + ea + bd + d f + f b.
We then have

2S = (a + b + c + d + e + f )2
− (a2 + b2we
By the Cauchy-Schwarz inequality, + chave
2
+ d2 + e2 + f 2 + 2ad + 2bd + 2c f ).

a2 + b2 + c2 + d2 + e2 + f 2 + 2ad + 2bd + 2c f

= (a + d)2 + (b + e)2 + (d + f )2
1 . (19)
Hence, ≥ (a + b + c + d + e + f ) 2
3
2
2S ≤ ( a + b + c + d + e + f ) 2.
3
Combining
and only if a = b = c = d = e = receive
(18) and (19) we f . M( 6) ≥ 3. The equality occurs if

5 Generalizations of P(3)
Problem 2. Let a, b, cr be arbitrarilyr real numbers. 3
Prove that
a + b + c r , (20)
pbr + qcr pcr + qar

where p, q > par + qbr p+q
0.
Proof. By the Cauchy-Schwarz inequality, we have
ar + b r
c r
c2r
pbr + qcr + b2r
pcr + qar +
par + qbr pbrcr + qbrar pcrar + qbrcr
= r r a2r rr r +r r 2
pa b + (qa
a c+ b + c ) 3
= ≥ .
(p + q)(arbr + brcr + crar )
p+q

The equality occurs if and only if a = b = c.


Corollary 7. Let a, b, c > 0 be given. Prove that
ar + br + cr 3
r
pb + qc r r
pc + qa r
≥ , (21)
par + qbr 2

where p, q are positive numbers satisfying p + q = 2.


Proof. By the Cauchy-Schwarz, we have
r
ar r + br + cr
pb + qc pcr + qar
a2r par + qbr
= r r + b2r c2r
pa b + qa c r r +
pbrcr + qbrar pcrar + qbrcr
= ( a r + b r + cr ) 2 ≥ . 3
2(arbr + brcr + cr ar ) 2
The equality occurs if and only if a = b = c.

We remark that P(3) can be rewritten in the following form


1 a1 1 + 1 b11 + 1 1c1≥ 0 0 + a0 0 0 + b00 0
, c0 (22)
b 0 + c 0 c 0+ a a + b b +c c +a a + b
where a = b = c = 1. Let
F at bt ct
(t ) = t t + t t + t t , (23)
b +c c +a a +b

stand for the function of variable t (0, +∞). Then, F(t) is monotone.
We then have
F(1) ≥ F(0).
Figure 1 is the curve of F(t) defined by (23) for the case a = 2, b =

3, c = 4. We see that the function is monotone on the interval (0, +∞).


We have the following problems.
Problem 3. Let a, b, c be given positive numbers, and α > β ≥ 0. Prove that
α≥
α aα α + α bαα + c aβ + bβ + cβ . (24)
b +c c +a aα + bα
bβ + cβ cβ + aβ aβ + bβ
y

F(0)
O x

Figure 1: Function F(t) is defined by (23)


Proof. Calculating the derivative of F as
FJ t t t t 2ct + at + bt
(t ) = a b (a
t t t t
2at + bt + ct
+ b c (b
t t t t
2bt + ct + at

+ c a (c
It follows F (t) ≥ 0 for every t ≥ 0. This implies that F is monotone on the
J
interval (0, ∞). We deduce that if α ≥ β, then
aα + bα + cα≥ aβ + bβ + cβ .
b α + cα cα + aα aα + b α
The inequality is proved.

bβ + cβ cβ + aβ a β + bβ
Corollary 8 below is an inequality which can be seen as an immediate
consequence of the above.
Corollary 8. If a, b, c are non-negative numbers, then
a2 + b2 + 2
c≥ +a + b . c
2 2 2
b +c c +a a +b 2 2 2 b+c c+a a+b
However, we can prove the following problem by using Problem 3.
Problem 4. If a, b, c ≥ 0, then

3 a3 3 + 3 b33 + 3 c ≥
3 +a2 2 + b22 . c2
b +c c +a a + b3 b 2 + c2 c + a2 a + b2
A further derection of the generalization of P(3) as follows.
Problem 5. Let a, b, c > 0 be given, and k ≥ 1. We have
ak + b k + ck ≥ ak−1 + bk−1 + ck−1 .
b+c c+a a+b 2
In particular, this inequality is homogeneous of (k − 1)-degree, and
we leave the proof of it to the reader.
A generalization for n in this derection is the following problem.
Problem 6. Let a1, . . . , an be n positive numbers with n ≥ 3, and p, q > 0.
Then, for every k ≥ 2 we have:
k
k k
+ 2 akn−1 n
a1
pa2 + qa3 a a
+··· pan + qa1 + pa1 + qa2
pa3 + qa4

+
a1k−1 + a2k−1 + k−1
· · · + an−1 + ak−1
n
.

Proof. Put
p+q
ak ak
ak + ak2 n−1
pa2 +1qa3 n
+ .
M :=
pa3 + qa4
+··· pa1 + qa2
We denote pan + qa1

+
an+1 := a1, an+2 := a2.
For each one of ak−1i , i = 1, 2, . . . , n we have
. .
k−1
ak−1 a i
qa i+2) .
= ak−1 i+1 +
i pi+1 + q
a a i+2 · pa i (
By the Cauchy-Schwarz inequality, we have
. Σ2
a1k−1 + ak−1
2 +···+ ak−1
n−1 + ak−1
n
≤ MΣp(a1k−2 a2 + ak−2 k−2 k−2
2 a3 + · · · + a n−1 an + a n a1)

1 2 n−1 n
k−2 k−2 k−2 k−2
+ q (a a3 + a a4 + · · · + a a1 + a a2).Σ (25)
By proving similar to the homogeneous inequalities as above, we obtain the
following inequalities:
k−2 k−2 k−2 k−2
a1 a2 + a 2 a3 + · · · + a n−1 an + a n a1

1 2 n−1 n
k−1 k−1 k−1 k−1
≤a +a +···+a +a , (26)
k−2 k−2 k−2 k−2
a1 a3 + a 2 a4 + · · · + a n−1 a1 + a n a2

≤ a1k−1 + ak−1
2
+ · · · + ak−1
n−1
+ ak−1
n
. (27)
Combining (25), (26), and (27) we receive

. Σ2
a1k−1 + ak−1
2 +···+ ak−1
n−1 + ak−1
n
≤ M(p + q)(ak−1 + ak−1 + · · · + ak−1 + ak−1).
1 2 n−1 n

Therefore, the inequality is proved.

Remark
Shapiro’s 4.
cycle one.k =Thus
When 1 and p = qnot
it does = 1,true
thefor
inequalityin
every n Problem 6 turns out
3. However, the
it may
hold for some n which can be seen as an open problem. ≥
We have the following corollary.
Corollary 9. Assume that a1, a2, . . . , an are positive numbers (n ≥ 3) and
k ≥ 2. Then
k k k
1 2 n
a + a akn−1 + a
a2 + a3 a3 + a4 + · · · + n + a1 a1 + a2

ak−1
1
+ aak−1
2
+ · · · + ak−1 + ak−1n
2 n−1
≥ .

We can provide now a generalization of P(3).


Problem 7. Let a, b, c be given
α
positive numbers,
α
and αα ≤ 0. Then
. a + . b + . c
b+c c+a
Σ Σ Σ
3 2α
≥ .
a+b
α
Proof. Put . a
α
. b
α
. c
+ + .
b+c c+a Σa + b
Σ Σ
M :=
By the AM-GM inequality, we have
Σ Σα
. abc
M≥ .
3
(a + b)(b + c)(c + a)
In other side, we can prove that

(a + b)(b + c)(c + a) ≥ 8abc.


Equivalently, abc 1
≤ .
(a + b)(b + c)(c + a) 8
From the assumption α ≤ 0 it follows α
Σ abc . Σα
1
≥ .
Σ (a + b)(b + c)(c + a) 8
α α α
Hence, . a . b . c
+ + 3
Σb + c Σc + a Σa + b 2α.

The inequality is proved.


α let α ≤ 1. Then
Problem 8. Let a, b, c > 0α be given, and α
. a . b . c
+ + 3
b+c c+a 2α.
Σ Σ Σa + b

Proof. By the Bernoulli’s inequality,


. we Σhaveα
2a 2a α

.b + c
. 2b c+a
Σ
Σ
2c + α − 1 ≥ α 2b ,
α b+c
+α− 1≥ α ,
c 2+c a
a+b
+α− 1≥ α ,
b + c2a c +2ba 2c
a+b a + b
( α − 1) . + + Σ ≥ (α − 1)3.
Combining the sides of the above inequalities we deduce the expected proof.
Problem 9. Assume that a, b, c > 0 and which satisfy abc = 1. Let α
be given. Then 1

aα + bα + cα≥ . 3
b+c c+a a+b 2
Proof. Without loss of generality, we assume that a ≥ b ≥ c. Then
a b c
,
≥ ≥
b+c c+a a+b

aα−1 ≥ bα−1 ≥ cα−1.


By the Chebyshev’s inequality, we have
aα + b α + cα
b+c c+a a+b
13 . α−1 α−1 α−1
Σ. a
b + c c b+ a a c+ b Σ
≥ a +b +c + + .
Applying the AM-GM inequality and having in mind that abc = 1, we
obtain √
aα−1 + bα−1 + cα−1 ≥ 3 aα−1bα−1cα−1 = 3.
By the Shapiro’s cycle inequality P(3), we have
a + b + c≥ . 3
b+c c+a a+b 2
The proof is completed.
An another proof of Problem 9 is the following. We have:
. aα
1+α

a
2 = .b +α c .a ( b + c) ,
1+α
b
c+a
b 2 = . .b ( c + a ),
1+α α
c
c
2 = a+b

.c ( a + b.
Therefore, we apply the Cauchy-Schwarz inequality to receive

2 2 2

. Σa b 1+ α Σ 2
1+ α 1+ α Σ . aα cα
Σ.
a +b +( c+
c ) + b( c + ) + c( a +

a b ) + +
b+c c+a a+b
The proof would be completed by the following inequality

2 2 2
.a 1+α + b 1+α + c 1+α Σ2 ≥ 3( ab + bc + ca ). (28)
For the proof of (28), we 1have
+α 1+α
1+α
a
≥ a + b + c. b c (29)
2 + 2 + 2

Indeed, by the AM-GM inequality and note that abc = 1 we have

a+b+c≥ abc = 3. (30)


√3
3
In other side, by the Bernoulli’s inequality it holds
1+α
1+α
1
a 2 + ≥ −1 + α .a (31)
1+α 2 2
1+α
b 2 + ≥ − 1 1+α
1+α 2 .b (32)
1+α 2
c
2 + − 1 1+α

2 .c. (33)
Combining the1+α inequality
1+α (31), (32), and (33), and2 using (30) we obtain
1+2α

a 2 + b1 + α+ c
2
3 + 3α
≥ .a + b + cΣ + 3 − 2
2
α−1 3 + 3α
≥ ( a 2+ b + c) + α − 1.3 + 3 − 2 =3a ++ 3α
b + c.
= ( a + b + c) + Σ .a + b + c ΣΣ + 3 −
2 2
Thus, (29) is proved. It follows that
2 2 2
.a 1+α + b 1+α + c 1+α Σ2 ≥ ( a + b + c)2 . (34)
In other side, we have

By the inequalities (34) and (35) we≥deduce


2
( a + b + c ) 3(ab +(28).
bc + The
ca). problem is proved. (35)

Problem 10. Let a, b, c > 0 be satisfied abc = 1. Assume that α ≥ 2. Then


1 + α 1 + α 1 ≥ .3
α
a (variables
b + c) b (c + a) c (a + b) 2
Proof. By changing the
1
x= ,
a
1
y= ,
b
1
z= ,
c
and
α = αJ + 1.
the inequality can be seen as an immediate consequence of Problem 9.
Note that we also have xyz = 1. The inequality is proved.
Furthermore, the inequality (36) below is a special case of the above
one.
condition
11 (IMO, 1995). Let x, y,=z be given
xyz 1. positive
Prove numbers
the satisfying the Problem
inequality
1 + 3 1 + 3 1 ≥ 3
3
a ( b + c) b ( c + a ) c ( a + b) . (36)
2
References
[1]P. J. Bushell. Shapiro’s cyclic sum. Bull. London Math. Soc., 26:564–
574, 1994.
[2]P. J. Bushell and J. B. McLeod. Shapiro’s cyclic inequality for even n.
J. of Inequal. & Appl., 7(3):331–348, 2002.
[3]P. H. Diananda. On a cyclic sum. Proc. Glasgow Math. Assoc., 6:11–13,
1963.
[4]V. G. Drinfeld. A cyclic inequality. Math. Notes, 9:68–71, 1971.
[5]A. M. Fink. Shapiro’s inequality. In G. V. Milovanovic, editor, Recent
Progress in inequality, Mathematics and Its Applications, part 13, pages 241–
4248. Kluwer Academic Publishers, Dordrecht/ Boston/London, first edition,
1997.
n
[6]L. J. Mordell. On the inequality ∑ r=1 xr/(xr+1 + xr+2) ≥ n/2 and
some others. Abh. Math. Se. Univ. Hamburg, 22:229–240, 1958.
n
[7]L. J. Mordell. Note on the inequality ∑r=1 xr/(xr+1 + xr+2) ≥ n/2
and some others. J. London Math. Soc., 37:176–178, 1962.
[8]B. G. Pachpatte, Mathematical Inequalities, North-Holland Mathemat- ical
Library, V. 67 (first ed.). Amsterdam, The Netherlands: Elsevier, 2005.
[9]R. A. Rankin, A cyclic inequality, Proc. Edinburgh Math. Soc. (2), V. 12
(1961), pp. 139-147.
[10]R. T. Rockafellar, Convex analysis, Princeton University Press, Princeton,
1970.
[11]H. S. Shapiro, Advanced problem # 4603, Amer. Math. Monthly, 61 (1954),
p. 571.
[12]J. M. Steele, The Cauchy-Schwarz Master Class: An Introduction to
the Art of Mathematical Inequalities, Cambridge University Press, Cam-
bridge, 2004.
[13]B. A. Troesch. The validity of Shapiro’s cyclic inequality. Math.
Comp., 53:657–664, 1989.
[14]N. M. Tuan and L. Q. Thuong, On an extension of Shapiros cyclic in-
equality, J. Inequalities Appl. (2009), no. 12, 1–5.

You might also like